Monday, October 10, 2022

Indian Air Force (IAF) 2022 :- 90th Anniversary Celebration


 

(About Indian Air Force)


Indian Air Force Day (IAF) is celebrated on October 8 every year.


Also known as "भारतीय वायु सेना", the IAF was officially established on October 8, 1932, by the British Empire.


The Indian Air Force was established in 1932 during World War II to support the Royal Air Force of the United Kingdom in its war against Japan.


2022 :- This year the Indian Air Force (IAF) marks its 90th anniversary.


:- President of India is the Supreme Commander of the Armed Forces.


:- As of 8th October 2022, The fourth-largest air force in the world is the Indian Air Force.


Headquarters :- New Delhi


Motto of India Air Force :- Touch the sky with Glory

It was taken from the eleventh chapter of the Bhagavad Gita.


:- The Chief of Air Staff, an air chief marshal is responsible for the operational command of the air force.


:- The major operations undertaken by the IAF are commonly known as  Operation Vijay, Operation Meghdoot, Operation Cactus and Operation Poomalai. 


As of 8th October 2022 Current Air Chief Marshal :-  Vivek Ram Chaudhari (V R Chaudhari)

June 2019 (Policies, Governance, and Administration) (NTA UGC NET Paper 1)

(1) Which of the following is/are true? निम्नलिखित में से कौन-सा सत्य है / हैं?
(1) India has given freedom to its citizens for following personal law of their own religion. भारत ने अपने नागरिकों को अपने धर्म के व्यक्तिगत कानून का पालन करने की स्वतंत्रता दी है।
(2) Divorce granted by ecclesiastical tribunal under Christian personal law is valid as it can override the law of the land like Muslim Personal Laws. ईसाई पर्सनल लॉ के तहत सनकी ट्रिब्यूनल द्वारा दिया गया तलाक वैध है क्योंकि यह मुस्लिम पर्सनल लॉ की तरह भूमि के कानून को खत्म कर सकता है।

Code :-
A. Only 1
B. Only 2
C. Both 1 and 2
D. Neither 1 nor 2

(A) Only 1
Explanation :-
Articles 25 to 28 provide freedom of religion to citizens of India.
Supreme Court has said that divorce granted by ecclesiastical tribunal under Christian personal law are not valid as it cannot override the law of the land.
The apex Court also rejected a PIL that sought according legal sanction to such separations granted by the Church Court.
A bench comprising Chief Justice J S Khehar and Justice D Y Chandrachud dismissed the plea saying the issue has been settled by it in its 1996 verdict delivered in the case of Molly Joseph versus George Sebastian.
The apex court had then ruled that Canon Law (personal law of Christians) can have theological or ecclesiastical implications to the parties.
But after the Divorce Act came into force, a dissolution or annulment granted under such personal law cannot have any legal impact as statute has provided a different procedure and a different code for divorce or annulment.
However, when it comes to personal issues (marriage, divorce, inheritance, custody of children, etc.), Muslims in India are governed by the Muslim Personal Law.
अनुच्छेद 25 से 28 भारत के नागरिकों को धर्म की स्वतंत्रता प्रदान करता है।
सुप्रीम कोर्ट ने कहा है कि क्रिश्चियन पर्सनल लॉ के तहत सनकी ट्रिब्यूनल द्वारा दिया गया तलाक वैध नहीं है क्योंकि यह जमीन के कानून को खत्म नहीं कर सकता है।
शीर्ष अदालत ने एक जनहित याचिका को भी खारिज कर दिया, जो चर्च कोर्ट द्वारा दी गई इस तरह के अलगाव के लिए कानूनी मंजूरी के अनुसार मांगी गई थी।
मुख्य न्यायाधीश जे एस खेहर और न्यायमूर्ति डी वाई चंद्रचूड़ की पीठ ने यह कहते हुए याचिका को खारिज कर दिया कि यह मसली जोसेफ बनाम जॉर्ज सेबेस्टियन के मामले में 1996 के फैसले में दी गई है।
शीर्ष अदालत ने तब फैसला सुनाया था कि कैनन लॉ (ईसाइयों का पर्सनल लॉ) पार्टियों के लिए धार्मिक या सनकी निहितार्थ हो सकता है।
लेकिन तलाक अधिनियम के लागू होने के बाद, इस तरह के व्यक्तिगत कानून के तहत दिए गए एक विघटन या विलोपन का कोई कानूनी प्रभाव नहीं हो सकता है क्योंकि क़ानून ने एक अलग प्रक्रिया और तलाक या विलोपन के लिए एक अलग कोड प्रदान किया है।
हालांकि, जब व्यक्तिगत मुद्दों (विवाह, तलाक, विरासत, बच्चों की हिरासत, आदि) की बात आती है, तो भारत में मुस्लिम मुस्लिम पर्सनल लॉ द्वारा शासित होते हैं।

(2) Which of the following is/are true regarding National Commission for Minorities? राष्ट्रीय अल्पसंख्यक आयोग के संबंध में निम्नलिखित में से कौन-सा सत्य है / हैं?
(1) The Chairperson shall hold office for a term of five years. अध्यक्ष पाँच वर्ष की अवधि के लिए पद धारण करेगा।
(2) The Union Government set up the National Commission for Minorities (NCM) under the National Commission for Minorities Act, 2001. केंद्र सरकार ने राष्ट्रीय अल्पसंख्यक आयोग अधिनियम, 2001 के तहत राष्ट्रीय अल्पसंख्यक आयोग (NCM) की स्थापना की।

Code :-
A. Only 1
B. Only 2
C. Both 1 and 2
D. Neither 1 nor 2

(D) Neither 1 nor 2
Explanation :-
The Union Government set up the National Commission for Minorities (NCM) under the National Commission for Minorities Act, 1992.
Six religious communities, viz; Muslims, Christians, Sikhs, Buddhists, Zoroastrians (Parsis) and Jains have been notified in Gazette of India as minority communities by the Union Government all over India.
Original notification of 1993 was for Five religious communities Sikhs, Buddhists, Parsis, Christians and Muslims.
The NCM adheres to the United Nations Declaration of 18 December 1992 which states that "States shall protect the existence of the National or Ethnic, Cultural, Religious and Linguistic identity of minorities within their respective territories and encourage conditions for the promotion of that identity.”
The Commission shall consist of a Chairperson, [a Vice Chairperson and five] Members to be nominated by the Central Government from amongst persons of eminence, ability and integrity; Provided that five Members including the Chairperson shall be from amongst the Minority communities.
The Chairperson and every Member shall hold office for a term of three years from the date he assumes office.
The Chairperson or a Member may, by writing under his hand addressed to the Central Government, resign from the office of Chairperson or, as the case may be, of the Member at any time.
No act or proceeding of the Commission shall be questioned or shall be invalid merely on the ground of the existence of any vacancy or defect in the constitution of the Commission.
केंद्र सरकार ने राष्ट्रीय अल्पसंख्यक आयोग अधिनियम, 1992 के तहत राष्ट्रीय अल्पसंख्यक आयोग (NCM) की स्थापना की।
छह धार्मिक समुदाय, अर्थात; मुस्लिम, ईसाई, सिख, बौद्ध, पारसी (पारसी) और जैनों को पूरे भारत में केंद्र सरकार द्वारा अल्पसंख्यक समुदायों के रूप में भारत के राजपत्र में अधिसूचित किया गया है।
1993 की मूल अधिसूचना पांच धार्मिक समुदायों सिख, बौद्ध, पारसी, ईसाई और मुस्लिम के लिए थी।
NCM 18 दिसंबर 1992 के संयुक्त राष्ट्र घोषणापत्र का पालन करता है जिसमें कहा गया है कि "राज्य अपने संबंधित क्षेत्रों के अल्पसंख्यकों की राष्ट्रीय या जातीय, सांस्कृतिक, धार्मिक और भाषाई पहचान के अस्तित्व की रक्षा करेंगे और उस पहचान को बढ़ावा देने के लिए शर्तों को प्रोत्साहित करेंगे।"
आयोग में एक अध्यक्ष, [एक उपाध्यक्ष और पाँच] सदस्य होंगे जिन्हें केंद्र सरकार द्वारा प्रतिष्ठित, क्षमता और सत्यनिष्ठा वाले व्यक्तियों में नामित किया जाएगा; बशर्ते अध्यक्ष सहित पांच सदस्य अल्पसंख्यक समुदायों में से हों।
अध्यक्ष और प्रत्येक सदस्य पद ग्रहण करने की तारीख से तीन वर्षों के लिए पद धारण करेंगे।
अध्यक्ष या सदस्य, केंद्र सरकार को संबोधित अपने हाथ से लिखकर, अध्यक्ष के पद से इस्तीफा दे सकते हैं या जैसा भी मामला हो, सदस्य के किसी भी समय हो सकता है।
आयोग के किसी भी कार्य या कार्यवाही पर सवाल नहीं उठाया जाएगा या आयोग के संविधान में किसी रिक्ति या दोष के अस्तित्व के आधार पर अमान्य होगा।

(3) Which of the following is/are true regarding political party symbols? निम्नलिखित में से कौन राजनीतिक पार्टी के प्रतीकों के बारे में सच है / हैं?
(1) Election Commission allots party symbols to all except National and State parties. चुनाव आयोग राष्ट्रीय और राज्य दलों को छोड़कर सभी को पार्टी के प्रतीक आवंटित करता है।
(2) In case of divide in a party, both sections are allowed to decide on their own as to who will get the original party symbol. किसी पार्टी में विभाजित होने की स्थिति में, दोनों वर्गों को यह तय करने की अनुमति है कि मूल पार्टी का प्रतीक किसे मिलेगा।

Code :-
A. Only 1
B. Only 2
C. Both 1 and 2
D. Neither 1 nor 2

(B) Only 2
Explanation :-
The Election Commission registers political parties for the purpose of elections and grants them recognition as national or state parties on the basis of their poll performance.
The other parties are simply declared as registered-unrecognised parties.
The recognition granted by the Commission to the parties determines their right to certain privileges like allocation of the party symbols, etc.
Every national party is allotted a symbol exclusively reserved for its use throughout the country.
Similarly, every state party is allotted a symbol exclusively reserved for its use in the state or states in which it is so recognised.
Thus the election commission allots the symbols even to national and state parties but keeps the symbols reserved for those parties.
A registered-unrecognised party, on the other hand, can select a symbol from a list of free symbols.
In other words, the Commission specifies certain symbols as ‘reserved symbols’ which are meant for the candidates set up by the recognised parties and others as ‘free symbols’ which are meant for other candidates.
In case a party gets divided into two or more factions, the factions can decide amongst themselves as to who will lay claim to the original party symbol.
If all the parties are interested, then the Election Commission allots the symbol to the faction having majority.
In the recent scenario of Samajwadi Party (symbol-bicycle), the poll body applied the test of majority supported as approved by the Supreme Court while deciding the case in favour of Akhilesh Yadav, whose group enjoys overwhelming majority support both among the legislative and organizational wing of the party.
चुनाव आयोग राजनीतिक दलों को चुनाव के उद्देश्य के लिए पंजीकृत करता है और उन्हें उनके चुनाव प्रदर्शन के आधार पर राष्ट्रीय या राज्य दलों के रूप में मान्यता प्रदान करता है।
अन्य दलों को केवल पंजीकृत-गैर-मान्यता प्राप्त दल के रूप में घोषित किया जाता है।
पार्टियों को आयोग द्वारा दी गई मान्यता कुछ विशेषाधिकारों जैसे पार्टी प्रतीकों के आवंटन, आदि के लिए उनके अधिकार को निर्धारित करती है।
प्रत्येक राष्ट्रीय पार्टी को पूरे देश में इसके उपयोग के लिए विशेष रूप से आरक्षित एक प्रतीक आवंटित किया जाता है।
इसी तरह, हर राज्य पार्टी को राज्य या राज्यों में इसके उपयोग के लिए विशेष रूप से आरक्षित एक प्रतीक आवंटित किया जाता है जिसमें इसे मान्यता प्राप्त है।
इस प्रकार चुनाव आयोग राष्ट्रीय और राज्य दलों को भी प्रतीक आवंटित करता है लेकिन उन दलों के लिए प्रतीकों को आरक्षित रखता है।
एक पंजीकृत-गैर-मान्यता प्राप्त पार्टी, दूसरी ओर, एक प्रतीक को मुक्त प्रतीकों की सूची से चुन सकती है।
दूसरे शब्दों में, आयोग कुछ प्रतीकों को ’आरक्षित प्रतीकों’ के रूप में निर्दिष्ट करता है जो कि मान्यता प्राप्त दलों द्वारा स्थापित उम्मीदवारों और अन्य लोगों के लिए and मुक्त प्रतीकों ’के रूप में होते हैं जो अन्य उम्मीदवारों के लिए होते हैं।
यदि कोई पार्टी दो या दो से अधिक गुटों में विभाजित हो जाती है, तो गुट आपस में यह तय कर सकते हैं कि मूल पार्टी चिन्ह के लिए कौन दावा करेगा।
यदि सभी पक्ष रुचि रखते हैं, तो चुनाव आयोग बहुमत वाले गुट को प्रतीक आवंटित करता है।
समाजवादी पार्टी (सिंबल-साइकिल) के हालिया परिदृश्य में, अखिलेश यादव के पक्ष में मामले का फैसला करते हुए सुप्रीम कोर्ट द्वारा अनुमोदित बहुमत के परीक्षण के लिए पोल बॉडी ने आवेदन किया, जिसका समूह विधायी और संगठनात्मक दोनों के बीच भारी बहुमत का समर्थन प्राप्त करता है। पार्टी का विंग।

(4) Which of the following is/are true regarding Public Accounts Committee (PAC)? लोक लेखा समिति (पीएसी) के बारे में निम्नलिखित में से कौन-सा सच है / हैं?
(1) PAC has the powers to summon Prime Minister or any other Minister with regard to any issue before it. पीएसी के पास किसी भी मुद्दे के संबंध में प्रधान मंत्री या किसी अन्य मंत्री को बुलाने की शक्तियाँ हैं।
(2) PAC was set up first under the provisions of the Government of India Act of 1909. पीएसी की स्थापना 1909 के भारत सरकार अधिनियम के प्रावधानों के तहत की गई थी।

Code :-
A. Only 1
B. Only 2
C. Both 1 and 2
D. Neither 1 nor 2

(D) Neither 1 nor 2
Explanation :-
Ministers shall not be called before the Committee either to give evidence or consultation in connection with the examination of estimates of accounts.
However, [the] chairperson, when [it is] considered necessary but after its [committee’s] deliberations are concluded, may have an informal interaction with the Minister.
The issue came up in a meeting of the Public Accounts Committee (PAC) of Parliament, where chairperson and former Union Minister K.V. Thomas had declared that the PAC was within its rights to summon Mr. Modi as the committee was examining monetary policy and the issue of demonetisation.
Public Accounts Committee was set up first in 1921 under the provisions of the Government of India Act of 1919 and has since been in existence.
At present, it consists of 22 members (15 from the Lok Sabha and 7 from the Rajya Sabha).
खातों के अनुमानों की जांच के संबंध में साक्ष्य या परामर्श देने के लिए मंत्रियों को समिति के समक्ष नहीं बुलाया जाएगा।
हालाँकि, [] चेयरपर्सन, जब [यह] आवश्यक माना जाता है, लेकिन इसके [समिति के] विचार-विमर्श के समापन के बाद, मंत्री के साथ एक अनौपचारिक बातचीत हो सकती है।
यह मुद्दा संसद की लोक लेखा समिति (पीएसी) की बैठक में सामने आया, जहां चेयरपर्सन और पूर्व केंद्रीय मंत्री के.वी. थॉमस ने घोषणा की थी कि पीएसी अपने अधिकारों के भीतर श्री मोदी को बुलाने के लिए थी क्योंकि समिति मौद्रिक नीति और विमुद्रीकरण के मुद्दे की जांच कर रही थी।
लोक लेखा समिति की स्थापना 1921 में भारत सरकार अधिनियम 1919 के प्रावधानों के तहत पहली बार की गई थी और तब से यह अस्तित्व में है।
वर्तमान में, इसमें 22 सदस्य (लोकसभा से 15 और राज्यसभा से 7) शामिल हैं।

(5)  According to the Supreme Court decision, appealing for votes on basis of which of the following will amount to corrupt practice? सुप्रीम कोर्ट के फैसले के अनुसार, निम्नलिखित में से किस आधार पर वोटों के लिए अपील करने से भ्रष्ट प्रैक्टिस होगी?
(1) Sex, Gender
(2) Caste
(3) Community
(4) Religion

Code :-
A. 2, 3, 4
B. 1, 2, 3
C. 1, 4
D. All of the above

(A) 2, 3, 4
Explanation :-
A seven-judge Bench of the Supreme Court held that an appeal for votes during elections on the basis of religion, caste, race, community or language, even that of the electorate, will amount to a ‘corrupt practice’ and call for disqualification of the candidate.

Section 123(3) of the Representation of the People Act, defines a corrupt electoral practice as follows: “The appeal by a candidate or his agent or by any other person with the consent of a candidate or his election agent to vote or refrain from voting for any person on the ground of his religion, race, caste, community....”

The question before the Supreme Court was deceptively simple: did the underlined word “his” qualify only the electoral candidate (and his agent, or persons speaking with his consent)? Or did it also qualify the person to whom the appeal was addressed (the elector)?

That means did it meant a bar on appeals made in the name of the candidate or does the word ‘his’ also extend to soliciting votes on the basis of the religion, caste, community, race, language of the electorate as a whole.

The latter would mean a blanket ban on any appeal, reference, campaign, discussion, dialogue or debate on the basis of religion, race, caste, community or language, even if such a debate was on the deprivations suffered by the voters due to these considerations.

सुप्रीम कोर्ट की सात-न्यायाधीशों की खंडपीठ ने कहा कि चुनावों के दौरान वोट के लिए अपील, धर्म, जाति, नस्ल, समुदाय या भाषा के आधार पर, यहां तक ​​कि मतदाता की 'भ्रष्ट प्रथा' के लिए राशि दी जाएगी और अयोग्य घोषित करने का आह्वान किया जाएगा। उम्मीदवार।

जनप्रतिनिधित्व अधिनियम की धारा 123 (3), एक भ्रष्ट चुनावी प्रथा को इस प्रकार परिभाषित करती है: “उम्मीदवार या उसके एजेंट या किसी अन्य व्यक्ति द्वारा उम्मीदवार या उसके चुनाव एजेंट की सहमति से वोट देने या मना करने की अपील किसी भी व्यक्ति को उसके धर्म, जाति, जाति, समुदाय के आधार पर वोट देना ... "

सर्वोच्च न्यायालय के समक्ष यह प्रश्न भ्रामक रूप से सरल था: क्या रेखांकित शब्द "उसका" केवल चुनावी उम्मीदवार (और उसके एजेंट, या उसकी सहमति से बोलने वाले व्यक्ति) के योग्य था? या क्या यह उस व्यक्ति को भी योग्य बनाता है जिसे अपील (चुनावी) संबोधित किया गया था?

इसका मतलब यह था कि उम्मीदवार के नाम पर की गई अपील पर एक बार का मतलब था या यह शब्द 'उसके' धर्म, जाति, समुदाय, जाति, मतदाताओं की भाषा के आधार पर वोट मांगने का विस्तार करता है।

उत्तरार्द्ध का मतलब धर्म, जाति, जाति, समुदाय या भाषा के आधार पर किसी भी अपील, संदर्भ, अभियान, चर्चा, संवाद या बहस पर एक कंबल प्रतिबंध होगा, भले ही इस तरह की बहस इनकी वजह से मतदाताओं को हुई क्षति पर हो विचार।

(6) Which new service has been formed to achieve the vision of “Startup India”, “Stand-up India” and “Make in India”? "स्टार्टअप इंडिया", "स्टैंड-अप इंडिया" और "मेक इन इंडिया" की दृष्टि को प्राप्त करने के लिए किस नई सेवा का गठन किया गया है?
A. Enterprise Development Service
B. Indian Enterprise Development Service
C. Indian Enterprise Service
D. Indian Business Service

(B) Indian Enterprise Development Service भारतीय उद्यम विकास सेवा
Explanation :-
The Union Cabinet has given its approval to the Cadre review and formation of a new service in the name of Índian Enterprise Development Service (IEDS)’ in the Office of Development Commissioner (MSME), Ministry of Micro, Small and Medium Enterprises(MSME).

The creation of the new cadre and change in structure will not only strengthen the organization but will also help to achieve the vision of “Startup India”, “Stand-up India” and “Make in India”.

The measure will enhance the capacity and efficiency of the organization and also help in achieving growth in MSME sector through a focussed and dedicated cadre of technical officers.
केंद्रीय मंत्रिमंडल ने सूक्ष्म, लघु और मध्यम उद्यम मंत्रालय (MSME) के विकास आयुक्त (MSME) के कार्यालय में Developmentndian Enterprise Development Service (IEDS) के नाम से कैडर समीक्षा और एक नई सेवा के गठन को अपनी मंजूरी दे दी है। ।

नए कैडर के निर्माण और संरचना में बदलाव से न केवल संगठन मजबूत होगा, बल्कि "स्टार्टअप इंडिया", "स्टैंड-अप इंडिया" और "मेक इन इंडिया" के दृष्टिकोण को प्राप्त करने में भी मदद मिलेगी।

यह उपाय संगठन की क्षमता और दक्षता को बढ़ाएगा और तकनीकी अधिकारियों के एक समर्पित और समर्पित कैडर के माध्यम से एमएसएमई क्षेत्र में विकास प्राप्त करने में मदद करेगा।

(7) Which of the following schemes are merged into a new "umbrella" scheme called "Rashtriya Yuva Sashaktikaran Karyakram (RYSK)"? निम्नलिखित में से कौन सी योजना "राष्ट्रीय युवा सशक्तीकरण कार्याक्रम (RYSK)" नामक एक नई "छतरी" योजना में विलय कर दी गई है?
(1) National Discipline Scheme
(2) Youth Hostels
(3) Nehru Yuva Kendra Sangathan

Code :-
A. 2, 3
B. 1, 2
C. 1, 3
D. All of the above

(D) All of the above
Explanation :-
The following existing Schemes/ Programmes shall get merged in the Rashtriya Yuva Sashaktikaran Karyakram (RYSK) :-

Nehru Yuva Kendra Sangathan (NYKS)
National Youth Corps (NYC)
National Programme for Youth and Adolescent Development (NPYAD)
International Cooperation (IC)
Youth Hostels (YH)
Assistance to Scouting and Guiding Organisations
National Discipline Scheme (NDS)
National Young Leaders Programme (NYLP)

(8) Which of the following is/are true regarding Gram Nyayalayas? निम्नलिखित में से कौन-सा ग्राम न्याालय के बारे में सही है / हैं?
(1) The Gram Nyayalaya shall not be bound by the rules of evidence provided in the Indian Evidence Act, 1872. भारतीय न्याय अधिनियम, 1812 में उपलब्ध कराए गए साक्ष्य के नियमों से ग्राम न्यायालय बाध्य नहीं होगा।
(2) One cannot appeal against the decision of the Gram Nyayalaya except to the High Court and the Supreme Court. ग्राम न्यायलय के निर्णय के खिलाफ उच्च न्यायालय और सर्वोच्च न्यायालय को छोड़कर कोई अपील नहीं कर सकता है।

Code :- 
A. Only 1
B. Only 2
C. Both 1 and 2
D. Neither 1 nor 2

(A) Only 1
Explanation :- 
Some salient features :- 
the Gram Nyayalaya shall not be bound by the rules of evidence provided in the Indian Evidence Act, 1872 but shall be guided by the principles of natural justice and subject to any rule made by the High Court. 
appeal in criminal cases shall lie to the Court of Session (in civil cases shall lie to the District Court), which shall be heard and disposed of within a period of six months from the date of filing of such appeal. 
the Gram Nyayalaya shall be established for every Panchayat at intermediate level or a group of contiguous Panchayats at intermediate level in a district or where there is no Panchayat at intermediate level in any State, for a group of contiguous Panchayats. 
कुछ मुख्य विशेषताएं: -
ग्राम न्यायलय भारतीय साक्ष्य अधिनियम, 1872 में दिए गए साक्ष्य के नियमों से बाध्य नहीं होगा, लेकिन प्राकृतिक न्याय के सिद्धांतों द्वारा निर्देशित किया जाएगा और उच्च न्यायालय द्वारा बनाए गए किसी भी नियम के अधीन होगा।
आपराधिक मामलों में अपील सत्र की अदालत में (सिविल मामलों में जिला न्यायालय से झूठ होगी), जो इस तरह की अपील दायर करने की तारीख से छह महीने के भीतर सुना और निपटाया जाएगा।
ग्राम न्यायालय मध्यवर्ती स्तर पर प्रत्येक पंचायत के लिए या किसी जिले में मध्यवर्ती स्तर पर सन्निहित पंचायतों के समूह या जहाँ किसी भी राज्य में मध्यवर्ती स्तर पर पंचायत नहीं है, के लिए स्थापित किया जाएगा।

(9) Which of the following is/are true regarding Gram Nyayalayas? निम्नलिखित में से कौन सा ग्राम न्याालय के बारे में सही है / हैं?
(1) The Gram Nyayalaya can try only civil cases and not criminal cases. ग्राम न्यायलय केवल दीवानी मामलों की कोशिश कर सकता है न कि आपराधिक मामलों की।
(2) The presiding officer of the Gram Nyayalaya will be selected by the Gram Sabha by two-third majority. ग्राम न्यायलय के पीठासीन अधिकारी को दो-तिहाई बहुमत से ग्राम सभा द्वारा चुना जाएगा।

Code :- 
A. Only 1
B. Only 2
C. Both 1 and 2
D. Neither 1 nor 2

(D) Neither 1 nor 2
Explanation :- 
The Gram Nyayalayas Act, 2008 has been enacted to provide for the establishment of the Gram Nyayalayas at the grass roots level for the purpose of providing access to justice to the citizens at their door steps.
Some of its salient features are :- 
the Gram Nyayalaya shall be a mobile court and shall exercise the poliwers of both Criminal and Civil Courts.
the Gram Nyayalaya shall try criminal cases, civil suits, claims or disputes which are specified in the First Schedule and the Second Schedule to the Act.
Gram Nyayalayas are aimed at providing inexpensive justice to people in rural areas at their doorsteps.
the Gram Nyayalaya shall be court of Judicial Magistrate of the first class and its presiding officer (Nyayadhikari) shall be appointed by the State Government in consultation with the High Court.
ग्राम न्यालय अधिनियम, 2008 को नागरिकों को उनके दरवाजे के कदमों पर न्याय तक पहुँच प्रदान करने के उद्देश्य से ग्राम न्यालय की स्थापना जमीनी स्तर पर करने के लिए अधिनियमित किया गया है।
इसकी कुछ मुख्य विशेषताएं हैं: -
ग्राम न्यालय एक मोबाइल कोर्ट होगा और आपराधिक और सिविल दोनों न्यायालयों के कार्यवाहियों का पालन करेगा।
ग्राम न्याालय आपराधिक मामलों, सिविल मुकदमों, दावों या विवादों की कोशिश करेगा, जो पहली अनुसूची और अधिनियम की दूसरी अनुसूची में निर्दिष्ट हैं।
ग्राम न्यायालय का उद्देश्य ग्रामीण क्षेत्रों में लोगों को उनके दरवाजे पर सस्ता न्याय प्रदान करना है।
ग्राम न्यालय प्रथम श्रेणी के न्यायिक मजिस्ट्रेट की अदालत होगी और इसके पीठासीन अधिकारी (न्यायाधिकारी) को उच्च न्यायालय के परामर्श से राज्य सरकार द्वारा नियुक्त किया जाएगा।

10.  Which of the following are components of the National Young Leaders Programme (NYLP)? निम्नलिखित में से कौन राष्ट्रीय युवा नेता कार्यक्रम (एनवाईएलपी) के घटक हैं?
(1) National Youth Corps
(2) National Youth Development Fund
(3) Neighbourhood Youth Parliament 

Code :- 
A. 1, 2
B. 1, 3
C. 2, 3
D. All of the above

(C) 2, 3
Explanation :- 
The programme aims at developing leadership qualities among the youth to enable them to realize their full potential and in the process, to contribute to the nation-building process. कार्यक्रम का उद्देश्य युवाओं में नेतृत्व के गुणों को विकसित करना है ताकि वे अपनी पूर्ण क्षमता का एहसास कर सकें और इस प्रक्रिया में राष्ट्र निर्माण की प्रक्रिया में योगदान कर सकें।
The objective of the Scheme is to motivate the youth to strive for excellence in their respective fields and to bring them to the forefront of the development process. It seeks to harness the immense youth energy for national – building. योजना का उद्देश्य युवाओं को अपने क्षेत्रों में उत्कृष्टता के लिए प्रयास करने और उन्हें विकास प्रक्रिया में सबसे आगे लाने के लिए प्रेरित करना है। यह राष्ट्रीय - निर्माण के लिए अपार युवा ऊर्जा का दोहन करना चाहता है।

The National Young Leaders Programme has the following five components :- 
1. Neighbourhood Youth Parliament
2. Youth for Development Programme
3. National Young Leaders Awards
4. National Youth Advisory Council
5. National Youth Development Fund

11. Which of the following is/are true regarding functions of the Standing Committee of the Inter State Council? निम्नलिखित में से कौन-सा अंतर राज्य परिषद की स्थायी समिति के कार्यों से संबंधित है?
(1) They process all matters pertaining to Centre State Relations before they are taken up for consideration in the Inter-State Council. वे राज्य-राज्य संबंध से संबंधित सभी मामलों की प्रक्रिया करते हैं, इससे पहले कि वे अंतर-राज्य परिषद में विचार के लिए उठाए जाएं।
(2) The Standing Committee may, if necessary, invite experts while deliberation upon the related subjects. स्थायी समिति, यदि आवश्यक हो, संबंधित विषयों पर विचार-विमर्श करते हुए विशेषज्ञों को आमंत्रित कर सकती है। 

Code :- 
A. Only 1
B. Only 2
C. Both 1 and 2
D. Neither 1 nor 2

(C) Both 1 and 2
Explanation :- 
The Standing Committee will :- 
Have continuous consultation and process matters for consideration of the Council.
Process all matters pertaining to Centre State Relations before they are taken up for consideration in the Inter-State Council.
Monitor the implementation of decisions taken on the recommendations of the Council and consider any other matter referred to it by the Chairman/Council.
The Standing Committee may, if necessary, invite experts and persons eminent in specific fields to have the benefit of their views while deliberation upon the related subjects.
स्थायी समिति करेगी: -
काउंसिल के विचारार्थ निरंतर परामर्श और प्रक्रिया के मामले रखें।
अंतर-राज्य परिषद में विचार के लिए उठाए जाने से पहले केंद्र राज्य संबंधों से संबंधित सभी मामलों की प्रक्रिया करें।
परिषद की सिफारिशों पर लिए गए निर्णयों के कार्यान्वयन की निगरानी करें और अध्यक्ष / परिषद द्वारा निर्दिष्ट किसी अन्य मामले पर विचार करें।
स्थायी समिति, यदि आवश्यक हो, संबंधित क्षेत्रों में विचार-विमर्श करते समय अपने विचारों का लाभ लेने के लिए विशिष्ट क्षेत्रों में प्रख्यात विशेषज्ञों और व्यक्तियों को आमंत्रित कर सकती है।

12. Who is the Chairman of the Standing Committee of the Inter State Council? अंतर राज्य परिषद की स्थायी समिति के अध्यक्ष कौन हैं?
A. Prime Minister
B. Union Minister of Finance
C. Union Minister of Home Affairs
D. Minister of Defence

(C) Union Minister of Home Affairs केंद्रीय गृह राज्य मंत्री
Explanation :- 
The composition of the Standing Committee of the Inter State Council is :- 
Chairman - Shri Rajnath Singh - Minister of Home Affairs
Members :- 
Smt. Sushma Swaraj - Minister of External Affairs
Shri Arun Jaitley - Minister of Finance & Corporate Affairs
Shri M. Venkaiah Naidu - Minister of Urban Development, Housing & Urban Poverty Alleviation & Information & Broadcasting
Shri Nitin Jairam Gadkari - Minister of Road Transport & Highways and Shipping
Chief Minister, Andhra Pradesh
Chief Minister, Punjab
Chief Minister, Chattisgarh
Chief Minister, Tripura
Chief Minister, Odisha
Chief Minister, Rajasthan
Chief Minister, Uttar Pradesh

13. Recently Inter-State Council has been reconstituted. Which of the following are amongst the six Union Ministers who are nominated by the PM? हाल ही में अंतर-राज्य परिषद का पुनर्गठन किया गया है। निम्नलिखित में से कौन छह केंद्रीय मंत्रियों में से हैं जिन्हें पीएम द्वारा नामित किया जाता है?
(1) Minister of Home Affairs
(2) Minister of External Affairs
(3) Minister of Tribal Affairs 

Code :- 
A. 2, 3
B. 1, 2
C. 1, 3
D. All of the above

(B) 1, 2
Explanation :- 
The Government has reconstituted the Inter State Council (ISC) and the Standing Committee of the Inter State Council under Clause 2 of the Inter State Council order, 1990.
The PM is the Chairman. सरकार ने इंटर स्टेट काउंसिल के 1990 के खंड 2 के तहत इंटर स्टेट काउंसिल (आईएससी) और इंटर स्टेट काउंसिल की स्थायी समिति का पुनर्गठन किया है।
पीएम चेयरमैन हैं।
The six union Ministers appointed by the PM are :- 
1. Shri Rajnath Singh - Minister of Home Affairs
2. Smt.Sushma Swaraj - Minister of External Affairs
3. Shri Arun Jaitley - Minister of Finance, Corporate Affairs
4. Shri M.Venkaiah Naidu - Minister of Urban Development, Housing & Urban Poverty Alleviation and Information & Broadcasting
5. Shri Nitin Jairam Gadkari - Minister of Road Transport & Highways and Shipping
6. Shri Manohar Parrikar - Minister of Defence

Minister of tribal affairs is included amongst the Five Ministers of Cabinet rank / Minister of State (independent charge) nominated by the Chairman of the Council (i.e., Prime Minister) are permanent invitees to the Council. जनजातीय मामलों के मंत्री को मंत्रिमंडल के पांच मंत्रियों / राज्य मंत्री (स्वतंत्र प्रभार) में शामिल किया जाता है, जिन्हें परिषद के अध्यक्ष द्वारा नामित किया जाता है (अर्थात, प्रधानमंत्री) परिषद में स्थायी आमंत्रित होते हैं।

14. Which of the following is true regarding PESA, 1996? PESA, 1996 के बारे में निम्नलिखित में से कौन-सा सही है?
A. All seats of Chairpersons of Panchayats at all levels shall be reserved in rotational manner for the STs, SCs and OBCs. सभी स्तरों पर पंचायतों के अध्यक्षों की सभी सीटें एसटी, एससी और ओबीसी के लिए घूर्णी तरीके से आरक्षित होंगी।
B. All seats of Chairpersons of Panchayats at all levels shall be reserved for the SC s and STs. सभी स्तरों पर पंचायतों के अध्यक्षों की सभी सीटें एससी और एसटी के लिए आरक्षित होंगी।
C. All seats of Chairpersons of Panchayats at all levels shall be reserved for the Scheduled Tribes. सभी स्तरों पर पंचायतों के अध्यक्षों की सभी सीटें अनुसूचित जनजाति के लिए आरक्षित होंगी।
D. All seats of Chairpersons of Panchayats at all levels shall be reserved for the Scheduled Castes. सभी स्तरों पर पंचायतों के अध्यक्षों की सभी सीटें अनुसूचित जाति के लिए आरक्षित होंगी।

(C) All seats of Chairpersons of Panchayats at all levels shall be reserved for the Scheduled Tribes. सभी स्तरों पर पंचायतों के अध्यक्षों की सभी सीटें अनुसूचित जनजाति के लिए आरक्षित होंगी।
Explanation :- 
All seats of Chairpersons of Panchayats at all levels shall be reserved for the Scheduled Tribes.
Every Panchayat at the village level shall be required to obtain from the Gram Sabha a certification of utilisation of funds for the above plans, programmes and projects.
Every Gram Sabha shall be competent to safeguard and preserve the traditions and customs of the people, their cultural identity, community resources and the customary mode of dispute resolution.
The Gram Sabha has power to exercise control over institutions and functionaries in all social sectors and also the power to control local plans and resources for such plans including tribal sub-plans.
सभी स्तरों पर पंचायतों के अध्यक्षों की सभी सीटें अनुसूचित जनजाति के लिए आरक्षित होंगी।
ग्राम स्तर पर प्रत्येक पंचायत को ग्राम सभा से उपरोक्त योजनाओं, कार्यक्रमों और परियोजनाओं के लिए धन के उपयोग का प्रमाण पत्र प्राप्त करना आवश्यक होगा।
प्रत्येक ग्राम सभा लोगों की परंपराओं और रीति-रिवाजों, उनकी सांस्कृतिक पहचान, सामुदायिक संसाधनों और विवाद समाधान के प्रथागत मोड को सुरक्षित रखने और संरक्षित करने के लिए सक्षम होगी।
ग्राम सभा के पास सभी सामाजिक क्षेत्रों में संस्थानों और अधिकारियों पर नियंत्रण रखने की शक्ति है और आदिवासी उप-योजनाओं सहित ऐसी योजनाओं के लिए स्थानीय योजनाओं और संसाधनों को नियंत्रित करने की शक्ति है।

15. Which of the following is/are true regarding PESA, 1996? PESA, 1996 में निम्नलिखित में से कौन सा सच है / हैं?
(1) The state government cannot nominate Scheduled Tribes at the intermediate level or the Panchayat at the district level. राज्य सरकार मध्यवर्ती स्तर पर या पंचायत को जिला स्तर पर अनुसूचित जनजाति के लिए नामित नहीं कर सकती है।
(2) The actual planning and implementation of the development projects in the Scheduled Areas shall be coordinated at the state level. अनुसूचित क्षेत्रों में विकास परियोजनाओं की वास्तविक योजना और कार्यान्वयन राज्य स्तर पर समन्वित किया जाएगा।

Code :- 
A. Only 1
B. Only 2
C. Both 1 and 2
D. Neither 1 nor 2

(B) Only 2
Explanation :- 
The state government may nominate such Scheduled Tribes which have no representation in the Panchayat at the intermediate level or the Panchayat at the district level.
But such nomination shall not exceed one-tenth of the total members to be elected in that Panchayat.
The Gram Sabha or the Panchayats at the appropriate level shall be consulted before making the acquisition of land in the Scheduled Areas for development projects and before resettling or rehabilitating persons affected by such projects in the Scheduled Areas.
However, the actual planning and implementation of the projects in the Scheduled Areas shall be coordinated at the state level.
राज्य सरकार ऐसे अनुसूचित जनजातियों को नामित कर सकती है जिनका मध्यवर्ती स्तर पर पंचायत में कोई प्रतिनिधित्व नहीं है या जिला स्तर पर पंचायत है।
लेकिन ऐसा नामांकन उस पंचायत में चुने जाने वाले कुल सदस्यों के दसवें हिस्से से अधिक नहीं होगा।
विकास परियोजनाओं के लिए अनुसूचित क्षेत्रों में भूमि के अधिग्रहण से पहले और अनुसूचित क्षेत्रों में ऐसी परियोजनाओं से प्रभावित व्यक्तियों के पुनर्वास या पुनर्वास से पहले ग्राम सभा या पंचायतों से उचित स्तर पर परामर्श किया जाएगा।
हालांकि, अनुसूचित क्षेत्रों में परियोजनाओं की वास्तविक योजना और कार्यान्वयन राज्य स्तर पर समन्वित किया जाएगा।

16. Which of the following is/are true regarding PESA, 1996? PESA, 1996 में निम्नलिखित में से कौन-सा सच है / हैं?
(1) The reservation for the Scheduled Tribes shall not be less than one half of the total number of seats. अनुसूचित जनजातियों के लिए आरक्षण कुल सीटों की संख्या के आधे से कम नहीं होगा।
(2) Panchayats at the higher level can assume the powers and authority of any Panchayat at the lower level or of the Gram Sabha. उच्च स्तर पर पंचायतें निचले स्तर पर या ग्राम सभा की किसी भी पंचायत की शक्तियों और अधिकारों को मान सकती हैं।

Code :- 
A. Only 1
B. Only 2
C. Both 1 and 2
D. Neither 1 nor 2

(A) Only 1
Explanation :- 
The reservation of seats in the Scheduled Areas in every Panchayat shall be in proportion to the population of the communities for whom reservation is sought to be given under Part IX of the Constitution.
However, the reservation for the Scheduled Tribes shall not be less than one half of the total number of seats.
The State Legislations shall contain safeguards to ensure that Panchayats at the higher level do not assume the powers and authority of any Panchayat at the lower level or of the Gram Sabha.
प्रत्येक पंचायत में अनुसूचित क्षेत्रों में सीटों का आरक्षण उन समुदायों की जनसंख्या के अनुपात में होगा जिनके लिए आरक्षण संविधान के भाग IX के तहत दिया जाना है।
हालाँकि, अनुसूचित जनजातियों के लिए आरक्षण कुल सीटों की संख्या के आधे से कम नहीं होगा।
राज्य विधानमंडलों में यह सुनिश्चित करने के लिए सुरक्षा उपाय होंगे कि उच्च स्तर पर पंचायतें निचले स्तर या ग्राम सभा की किसी पंचायत की शक्तियों और अधिकारों को न मानें।

17. Who of the following have framed rules as per the PESA Act, 1996?
(1) Maharashtra
(2) Chhattisgarh
(3) Rajasthan 

Code :- 
A. 1, 2
B. 2, 3
C. 1, 3
D. All of the above

(C) 1, 3
Explanation :- 
The provisions of Panchayats with certain modification and exceptions have been extended to the Schedule V areas viz. the ten States where the Panchayats exists in the country including Andhra Pradesh.
Gram Sabhas have been constituted in every State as per the Panchayat Raj Act/PESA Rules of the concerned State.
Only four States have framed their Rules for implementation of PESA. These are, Andhra Pradesh, Himachal Pradesh, Maharashtra and Rajasthan.
कुछ संशोधनों और अपवादों वाली पंचायतों के प्रावधानों को अनुसूची V क्षेत्रों में विस्तारित किया गया है। उन दस राज्यों में जहां आंध्र प्रदेश सहित देश में पंचायतें मौजूद हैं।
संबंधित राज्य के पंचायत राज अधिनियम / पीईएसए नियमों के अनुसार प्रत्येक राज्य में ग्राम सभाओं का गठन किया गया है।
केवल चार राज्यों ने PESA के कार्यान्वयन के लिए अपने नियम बनाए हैं। ये हैं, आंध्र प्रदेश, हिमाचल प्रदेश, महाराष्ट्र और राजस्थान।

18. Which of the following is/are true regarding PESA, 1996? PESA, 1996 में निम्नलिखित में से कौन-सा सच है / हैं?
(1) Gram Sabha or Panchayat has the right to exercise control over money lending to STs. ग्राम सभा या पंचायत को एसटी को धन उधार पर नियंत्रण का अधिकार है।
(2) PESA has been enacted for Sixth Schedule areas. PESA को छठी अनुसूची क्षेत्रों के लिए अधिनियमित किया गया है।

Code :- 
A. Only 1
B. Only 2
C. Both 1 and 2
D. Neither 1 nor 2

(A) Only 1
Explanation :- 
Gram Sabhas or Panchayats at appropriate level shall also have powers to manage minor water bodies; power of mandatory consultation in matters of land acquisition; resettlement and rehabilitation and prospecting licenses/mining leases for minor minerals; power to prevent alienation of land and restore alienated land; regulate and restrict sale/consumption of liquor; manage village markets, control money lending to STs; and ownership of minor forest produce. उचित स्तर पर ग्राम सभाओं या पंचायतों के पास भी मामूली जल निकायों का प्रबंधन करने की शक्तियाँ होंगी; भूमि अधिग्रहण के मामलों में अनिवार्य परामर्श की शक्ति; लघु खनिजों के लिए पुनर्वास और पुनर्वास और पूर्वेक्षण लाइसेंस / खनन पट्टे; भूमि के अलगाव को रोकने और विस्थापित भूमि को बहाल करने की शक्ति; शराब की बिक्री / खपत को विनियमित और प्रतिबंधित करना; गाँव के बाजारों का प्रबंधन, एसटी को पैसा उधार देना; और लघु वन उपज का स्वामित्व।
PESA, 1996 has been enacted for 5th Schedule areas. It includes 10 states. PESA, 1996 को 5 वीं अनुसूची क्षेत्रों के लिए अधिनियमित किया गया है। इसमें 10 राज्य शामिल हैं।

19. Who is a Classified Service Voter? एक वर्गीकृत सेवा मतदाता कौन है?
A. Service voters employed under intelligence agencies. खुफिया एजेंसियों के तहत कार्यरत सेवा मतदाता।
B. A service voter who opts for voting through a proxy appointed by him/her. एक सेवा मतदाता जो उसके द्वारा नियुक्त प्रॉक्सी के माध्यम से मतदान का विरोध करता है।
C. Service Voters employed by intelligence and Indian diplomatic missions.  सेवा वोटर जो खुफिया और भारतीय राजनयिक मिशनों द्वारा नियोजित हैं।
D. Proxy of the service voter. सेवा मतदाता का प्रॉक्सी।

(B) A service voter who opts for voting through a proxy appointed by him/her. एक सेवा मतदाता जो उसके द्वारा नियुक्त प्रॉक्सी के माध्यम से मतदान का विरोध करता है।
Explanation :- 
Service voter belonging to Armed Forces or forces to which provisions of Army Act, 1950 are applicable, has option of either voting through postal ballot or through a proxy voter duly appointed by him/her.
A service voter who opts for voting through a proxy is called Classified Service Voter (CSV).
A service voter may appoint (by applying to Returning Officer in Form 13 F of Conduct of Elections Rules, 1961) any person as his / her proxy to give vote on his / her behalf and in his / her name at the polling station.
The proxy shall have to be ordinary resident of that constituency. He need not be a registered voter but he / she must not be disqualified to be registered as a voter.
सशस्त्र बलों या सेना के जिन मतदाताओं पर सेना अधिनियम, 1950 के प्रावधान लागू होते हैं, उनके पास डाक मतपत्र के माध्यम से या उसके द्वारा नियुक्त एक प्रॉक्सी मतदाता के माध्यम से मतदान का विकल्प होता है।
एक सेवा मतदाता जो प्रॉक्सी के माध्यम से मतदान करने का विरोध करता है, उसे वर्गीकृत सेवा मतदाता (CSV) कहा जाता है।
एक सेवा मतदाता नियुक्ति कर सकता है (चुनाव नियमों, 1961 के फॉर्म 13 एफ में रिटर्निंग अधिकारी के लिए आवेदन करके) किसी भी व्यक्ति को उसकी / उसकी ओर से और मतदान केंद्र पर उसके नाम पर वोट देने के लिए उसके प्रॉक्सी के रूप में।
प्रॉक्सी को उस निर्वाचन क्षेत्र का साधारण निवासी होना चाहिए। उसे एक पंजीकृत मतदाता होने की आवश्यकता नहीं है लेकिन उसे मतदाता के रूप में पंजीकृत होने के लिए अयोग्य घोषित नहीं किया जाना चाहिए।

20.  Which of the following is/are true regarding Service Voters? निम्नलिखित में से कौन-सा सेवा वोटर के बारे में सही है / हैं?
(1) A son/daughter residing ordinarily with a service voter cannot be enrolled as service voter. सेवा मतदाता के साथ सामान्यतया रहने वाले पुत्र / पुत्री को सेवा मतदाता के रूप में नामांकित नहीं किया जा सकता है।
(2) Husband of a female Service Voter cannot be registered as a Service Voter. महिला सेवा मतदाता के पति को सेवा मतदाता के रूप में पंजीकृत नहीं किया जा सकता है।

Code :- 
A. Only 1
B. Only 2
C. Both 1 and 2
D. Neither 1 nor 2

(C) Both 1 and 2
Explanation :-  
The wife of a service voter shall, if she is ordinarily residing with him, be also deemed to be a service voter in the constituency specified by that person.
The service voter has to make a statement to the effect in the relevant Form 2/2A/3 that his wife ordinarily resides with him. The wife will be enrolled as a service voter on the basis of declaration made by her husband in the application form itself submitted by him and no separate declaration / application is required to be made by the wife.
A son / daughter / relative / servant etc. residing ordinarily with a service voter cannot be enrolled as service voter.
Under the existing law, this facility is available only to the wife of a male service voter and is not available to the husband of a female service voter.
एक सेवा मतदाता की पत्नी, यदि वह सामान्य रूप से उसके साथ रहती है, तो उस व्यक्ति द्वारा निर्दिष्ट निर्वाचन क्षेत्र में एक सेवा मतदाता माना जाता है।
सेवा मतदाता को संबंधित फॉर्म 2/2 ए / 3 में इस आशय का विवरण देना होता है कि उसकी पत्नी सामान्य रूप से उसके साथ रहती है। पत्नी को उसके द्वारा प्रस्तुत आवेदन पत्र में पति द्वारा की गई घोषणा के आधार पर सेवा मतदाता के रूप में नामांकित किया जाएगा और पत्नी द्वारा कोई अलग घोषणा / आवेदन करने की आवश्यकता नहीं है।
एक बेटा / बेटी / रिश्तेदार / नौकर आदि को सेवा मतदाता के साथ नियत रूप से रहने वाले को सेवा मतदाता के रूप में नामांकित नहीं किया जा सकता है।
मौजूदा कानून के तहत, यह सुविधा केवल पुरुष सेवा मतदाता की पत्नी के लिए उपलब्ध है और महिला सेवा मतदाता के पति के लिए उपलब्ध नहीं है।

21. Which of the following is true regarding an ordinary voter and service voter? सामान्य मतदाता और सेवा मतदाता के संबंध में निम्नलिखित में से कौन सा सही है?
A. A person having service qualification can get enrolled at either his/her place of posting or his/her native place. सेवा योग्यता रखने वाला व्यक्ति अपनी पोस्टिंग के स्थान पर या अपने मूल स्थान पर दाखिला ले सकता है।
B. A service voter cannot get enrolled at his/her place of posting. सेवा योग्यता रखने वाला व्यक्ति अपनी पोस्टिंग के स्थान पर या अपने मूल स्थान पर दाखिला ले सकता है।
C. If person having service qualification enrolls in his/her place of posting, he/she is still considered as service voter and not general elector. सेवा योग्यता रखने वाला व्यक्ति अपनी पोस्टिंग के स्थान पर या अपने मूल स्थान पर दाखिला ले सकता है।
D. If an ordinary voter migrates to other areas for job/profession, he can register as service voter. यदि कोई साधारण मतदाता नौकरी / पेशे के लिए अन्य क्षेत्रों में जाता है, तो वह सेवा मतदाता के रूप में पंजीकरण कर सकता है।

(A) person having service qualification can get enrolled at either his/her place of posting or his/her native place. सेवा योग्यता रखने वाला व्यक्ति अपने पदस्थापन स्थान पर या अपने मूल स्थान पर दाखिला ले सकता है।
Explanation :-
While an ordinary elector is registered in the electoral roll of the constituency in which his place of ordinary residence is located, person having service qualification can get enrolled as ‘service voter’ at his native place even though he actually may be residing at a different place (of posting).
He has, however, an option to get himself enrolled as general elector at the place of his posting where he factually, at the point of time, is residing ordinarily with his family for a sufficient span of time.
Service voter is a voter having service qualifications which are related to employment of armed forces, central government, etc. No ordinary person can be termed as service voter if he is not into service of armed forces, central government, etc.
जबकि एक साधारण निर्वाचक उस निर्वाचन क्षेत्र के मतदाता सूची में पंजीकृत होता है जिसमें उसका साधारण निवास स्थान स्थित होता है, सेवा योग्यता रखने वाला व्यक्ति अपने मूल स्थान पर 'सेवा मतदाता' के रूप में नामांकित हो सकता है, भले ही वह वास्तव में एक अलग स्थान पर रहता हो (पोस्टिंग का)।
हालांकि, उनके पास अपनी पोस्टिंग के स्थान पर सामान्य निर्वाचक के रूप में खुद को नामांकित करने का एक विकल्प है, जहां वे तथ्यात्मक रूप से, पर्याप्त समय के लिए अपने परिवार के साथ सामान्य रूप से निवास कर रहे हैं।
सेवा मतदाता सेवा योग्यता रखने वाला एक मतदाता है जो सशस्त्र बलों, केंद्र सरकार, आदि के रोजगार से संबंधित है। किसी भी सामान्य व्यक्ति को सेवा मतदाता नहीं कहा जा सकता है यदि वह सशस्त्र बलों, केंद्र सरकार, आदि की सेवा में नहीं है।

22. Which of the following is/are true regarding postal and e-postal ballot? निम्नलिखित में से कौन-सा डाक और ई-पोस्टल बैलेट के बारे में सही है / हैं?
(1) Current system of postal ballot involves one-way transmission of ballot paper by the postal services. पोस्टल बैलेट की वर्तमान प्रणाली में पोस्टल सेवाओं द्वारा बैलेट पेपर का एकतरफा प्रसारण शामिल है।
(2) The e-postal system will provide complete online voting system useful for remote service voters. ई-पोस्टल प्रणाली दूरस्थ सेवा के मतदाताओं के लिए उपयोगी ऑनलाइन मतदान प्रणाली प्रदान करेगी।

Code :-
A. Only 1
B. Only 2
C. Both 1 and 2
D. Neither 1 nor 2

(D) Neither 1 nor 2
Explanation :-
The Election Commission proposed that the categories of voters mentioned at rule 18 of the Conduct of Election Rules, 1961 may be made eligible for e-postal ballot system.
However, on a pilot basis, e-postal ballot system has been introduced by Notification dated 21st October, 2016 for service voters only.
A blank postal ballot could be electronically transmitted to the voter, namely, e-postal ballot system. Voters entitled to postal ballot such as service voters, can download the postal ballot and print the blank postal ballot.
After marking his vote in the blank postal ballot, the same would be returned to the concerned Returning Officer by post as in the present system of postal ballot.
This is one-way transmission of ballot paper by the postal services.
Current system of postal ballot involves two-way transmission of ballot paper by the postal services. This causes delays.
चुनाव आयोग ने प्रस्ताव दिया कि चुनाव आचार संहिता, 1961 के नियम 18 में उल्लिखित मतदाताओं की श्रेणियों को ई-पोस्टल बैलेट प्रणाली के लिए योग्य बनाया जा सकता है।
हालाँकि, पायलट आधार पर, केवल सेवा मतदाताओं के लिए 21 अक्टूबर, 2016 की अधिसूचना द्वारा ई-पोस्टल बैलट प्रणाली शुरू की गई है।
एक रिक्त डाक मतपत्र इलेक्ट्रॉनिक रूप से मतदाता को प्रेषित किया जा सकता है, अर्थात्, ई-पोस्टल मतपत्र प्रणाली। मतदाता डाक मतपत्र के हकदार हैं जैसे कि सेवा मतदाता, डाक मतपत्र डाउनलोड कर सकते हैं और रिक्त डाक मतपत्र प्रिंट कर सकते हैं।
रिक्त डाक मतपत्र में अपना मत अंकित करने के बाद, डाक विभाग की वर्तमान प्रणाली के अनुसार डाक द्वारा संबंधित रिटर्निंग अधिकारी को वापस कर दिया जाएगा।
यह डाक सेवाओं द्वारा बैलेट पेपर का एकतरफा प्रसारण है।
पोस्टल बैलेट की वर्तमान प्रणाली में पोस्टल सेवाओं द्वारा बैलेट पेपर के दो-तरफा प्रसारण शामिल हैं। यह देरी का कारण बनता है।

23. Who of the following are categorized as "Service Voters"? निम्नलिखित में से किसे "सेवा मतदाता" के रूप में वर्गीकृत किया गया है?
(1) Being a member of the armed Forces of the Union. संघ के सशस्त्र बलों का सदस्य होना।
(2) Being a person who is employed under the Government of India, in a post outside his/her cadre. भारत सरकार के अधीन कार्यरत व्यक्ति होने के नाते, उसके कैडर के बाहर एक पद पर।
(3) Being a person who is employed under the Government of India, in a post outside India. भारत के बाहर किसी पद पर कार्यरत व्यक्ति होना।

Code :-
A. 1, 3
B. 2, 3
C. 1, 3
D. All of the above

(C) 1, 3
Explanation :-
Service voter is a voter having service qualification. According to the provisions of Representation of People Act, 1950, service qualification means
A. Being a member of the armed Forces of the Union; or
B. Being a member of a force to which provisions of the Army Act, 1950 (46 of 1950), have been made applicable whether with or     without modification;
C. Being a member of an Armed Police Force of a State, and serving outside that state; or
D. Being a person who is employed under the Government of India, in a post outside India.
सेवा मतदाता सेवा योग्यता रखने वाला मतदाता है। जनप्रतिनिधित्व कानून, 1950 के प्रावधानों के अनुसार सेवा योग्यता का मतलब है
A. संघ के सशस्त्र बलों का सदस्य होने के नाते; या
ख। सेना अधिनियम, १ ९ ५० (१ ९ ५० के ४६) के प्रावधानों के एक बल के सदस्य होने के नाते, यह लागू किया गया है कि संशोधन के साथ या बिना;
C. एक राज्य के सशस्त्र पुलिस बल का सदस्य होने के नाते, और उस राज्य के बाहर सेवा करना; या
D. भारत के बाहर एक पोस्ट में एक व्यक्ति जो भारत सरकार के अधीन कार्यरत है।

24. The underlying objective of "Advantage Health Care India 2016 (AHCI 2016)" is a conglomeration of 5Ts. Which of the following are the 5Ts? "एडवांटेज हेल्थ केयर इंडिया 2016 (AHCI 2016)" का अंतर्निहित उद्देश्य 5Ts का समूह है। निम्नलिखित में से कौन सी 5Ts हैं?
(1) Tradition परंपरा
(2) Tourism पर्यटन
(3) Trade व्यापार

Code :-
A. 1, 3
B. 2, 3
C. 1, 2
D. All of the above

(D) All of the above
Explanation :-
The objective of this international summit is to promote India as a Premier Global Healthcare Destination and to enable streamlined medical services exports from India. This underlying objective is a unique conglomeration of 5Ts, namely.

Tradition – promoting AYUSH and wellness industry
Technology – promoting technologically advanced medical treatment in India
Tourism – encouraging people to travel to India for medical value reasons
Talent – showcasing globally renowned Indian talent in healthcare sector
Trade – creating unique B2B institutionalized linkages.
इस अंतर्राष्ट्रीय शिखर सम्मेलन का उद्देश्य भारत को प्रीमियर ग्लोबल हेल्थकेयर डेस्टिनेशन के रूप में बढ़ावा देना और भारत से सुव्यवस्थित चिकित्सा सेवाओं के निर्यात को सक्षम बनाना है। यह अंतर्निहित उद्देश्य 5Ts का एक अनूठा समूह है, अर्थात्।

परंपरा - आयुष और कल्याण उद्योग को बढ़ावा देना
प्रौद्योगिकी - भारत में तकनीकी रूप से उन्नत चिकित्सा उपचार को बढ़ावा देना
पर्यटन - चिकित्सा मूल्य कारणों से लोगों को भारत की यात्रा के लिए प्रोत्साहित करना
प्रतिभा - स्वास्थ्य सेवा क्षेत्र में विश्व स्तर पर प्रसिद्ध भारतीय प्रतिभा का प्रदर्शन
व्यापार - अद्वितीय बी 2 बी संस्थागत लिंकेज बनाना।

25.  Who headed the committee on Indian Diaspora which proposed the setup of a centre for linkages with Indian diaspora? भारतीय डायस्पोरा पर समिति का नेतृत्व किसने किया, जिसने भारतीय डायस्पोरा के साथ संबंध के लिए एक केंद्र की स्थापना का प्रस्ताव रखा?
A. Narsimhan Committee
B. LM Singhvi
C. Rao Committee
D. Yashpal Committee

(B) LM Singhvi
Explanation :-
A High Level Committee on Indian Diaspora, headed by Shri LM Singhvi, had recommended in January 2002 that the Government must renew and strengthen linkages of overseas Indians to their place of origin and with each other.
The committee recommended that a Pravasi Bharatiya Bhavan should be set up to emerge as the focal point for networking between India and its overseas Indian community; and as a suitable place which would commemorate the trials, tribulations, the evolution and achievements of the Indian Diaspora.
श्री एल एम सिंघवी की अध्यक्षता में भारतीय प्रवासी पर एक उच्च स्तरीय समिति ने जनवरी 2002 में सिफारिश की थी कि सरकार को अपने मूल स्थान और एक दूसरे के साथ प्रवासी भारतीयों के संबंधों को नवीनीकृत और मजबूत करना होगा।
समिति ने सिफारिश की कि भारत और उसके विदेशी भारतीय समुदाय के बीच नेटवर्किंग के लिए केंद्र बिंदु के रूप में उभरने के लिए एक प्रवासी भारतीय भवन स्थापित किया जाना चाहिए; और एक उपयुक्त स्थान के रूप में जो भारतीय डायस्पोरा के परीक्षणों, क्लेशों, विकास और उपलब्धियों की प्रशंसा करेगा।

26.  Which of the following is/are objective/s of Pravasi Bhartiya Kendra? निम्नलिखित में से कौन / प्रवासी भारतीय केंद्र का उद्देश्य / है?
(1) Forum to leverage the skills, knowledge and resources of the Indian Diaspora.
(2) Centre for diaspora studies.
(3) Permanent exhibition illustrating the history of the Diaspora.

Code :-
A. 1, 3
B. 2, 3
C. 1, 2
D. All of the above

(D) All of the above
Explanation :-
Objectives of Pravasi Bhartiya Kendra -
Symbolise the common aspirations of all members of the Diaspora to maintain their cultural identity and civilizational ethos.
Be an ideal forum to leverage the skills, knowledge and resources of the Indian Diaspora for accelerating the development process in India.
Serve as a permanent exhibition illustrating the history of the Diaspora in various parts of the world.
The Kendra would be developed into a Centre for diaspora studies to run short term courses and orientation programmes for the youth of the India Diaspora.
The Kendra would lend a strategic dimension to India’s engagement with its diaspora and provide informal channels of communication, which would help in promoting understanding of issues of vital concern to India.
प्रवासी भारतीय केंद्र के उद्देश्य -
अपनी सांस्कृतिक पहचान और सभ्यता के लोकाचार को बनाए रखने के लिए प्रवासी भारतीयों के सभी सदस्यों की सामान्य आकांक्षाओं को चिह्नित करें।
भारत में विकास प्रक्रिया में तेजी लाने के लिए भारतीय प्रवासी के कौशल, ज्ञान और संसाधनों का लाभ उठाने के लिए एक आदर्श मंच हो।
दुनिया के विभिन्न हिस्सों में प्रवासी भारतीयों के इतिहास को दर्शाती एक स्थायी प्रदर्शनी के रूप में परोसें।
भारत डायस्पोरा के युवाओं के लिए अल्पकालिक पाठ्यक्रम और अभिविन्यास कार्यक्रम चलाने के लिए केंद्र को प्रवासी अध्ययन केंद्र के रूप में विकसित किया जाएगा।
केंद्र अपने प्रवासी भारतीयों के साथ भारत के जुड़ाव के लिए एक रणनीतिक आयाम प्रदान करेगा और संचार के अनौपचारिक चैनल प्रदान करेगा, जो भारत के लिए महत्वपूर्ण चिंता के मुद्दों को समझने में मदद करेगा।

27.  Pravasi Bhartiya Kendra is at :- प्रवासी भारतीय केंद्र इस प्रकार है: -
A. Mumbai
B. New Delhi
C. Bengaluru
D. Gurgaon

(B) New Delhi
Explanation :-
The Indian Diaspora constitutes a significant economic, social and cultural force in the world today.
To commemorate the trials and tribulations, as well as the subsequent evolution and achievements of the diverse Indian Diaspora, it has been decided to establish a Pravasi Bharatiya Kendra at Chanakyapuri, New Delhi.
The Kendra will serve as the focal point of the institutional framework to benefit from networks with and among overseas Indians. Over time, the Kendra is expected to develop into the hub of activities for sustainable, symbiotic and mutually rewarding economic, social and cultural engagement between India and its Diaspora.
भारतीय प्रवासी आज दुनिया में एक महत्वपूर्ण आर्थिक, सामाजिक और सांस्कृतिक बल का गठन करता है।
परीक्षणों और क्लेशों, साथ ही साथ विविध भारतीय प्रवासी के विकास और उपलब्धियों के लिए, नई दिल्ली के चाणक्यपुरी में एक प्रवासी भारतीय केंद्र की स्थापना करने का निर्णय लिया गया है।
केंद्र भारतीयों के साथ और विदेशों में नेटवर्क से लाभ के लिए संस्थागत ढांचे के केंद्र बिंदु के रूप में काम करेगा। समय के साथ, केंद्र को भारत और उसके डायस्पोरा के बीच टिकाऊ, सहजीवन और पारस्परिक रूप से पुरस्कृत आर्थिक, सामाजिक और सांस्कृतिक जुड़ाव के लिए गतिविधियों के केंद्र में विकसित होने की उम्मीद है।

28.  When funds to meet the proposed expenditure on a new service can be made available by reappropriation, then which grant is passed? जब एक नई सेवा पर प्रस्तावित व्यय को पूरा करने के लिए धन पुन: प्राप्ति द्वारा उपलब्ध कराया जा सकता है, तो कौन सा अनुदान पारित किया जाता है?

A. Additional grant
B. Token grant
C. Excess grant
D. Reappropriation grant

(B) Token grant
Explanation :-
Token Grant - It is granted when funds to meet the proposed expenditure on a new service can be made available by reappropriation.
A demand for the grant of a token sum (of Re 1) is submitted to the vote of the Lok Sabha and if assented, funds are made available.
Reappropriation involves transfer of funds from one head to another. It does not involve any additional expenditure.
टोकन ग्रांट - यह तब दिया जाता है जब किसी नई सेवा पर प्रस्तावित व्यय को पूरा करने के लिए धनराशि को पुन: विनियोजन द्वारा उपलब्ध कराया जा सकता है।
एक टोकन राशि के अनुदान की मांग (1 का पुन:) लोकसभा के वोट के लिए प्रस्तुत किया जाता है और यदि आश्वासन दिया जाता है, तो धन उपलब्ध कराया जाता है।
पुनर्संरचना में एक सिर से दूसरे तक धन का हस्तांतरण शामिल है। इसमें कोई अतिरिक्त खर्च शामिल नहीं है।

29. Vote of credit is :- साख का आधार है: -
A. Granted for meeting an unexpected demand upon the resources of India. भारत के संसाधनों पर एक अप्रत्याशित मांग को पूरा करने के लिए दी गई।
B. Granted for a special purpose and forms no part of the current service of any financial year. एक विशेष उद्देश्य के लिए दी गई और किसी भी वित्तीय वर्ष की वर्तमान सेवा का कोई हिस्सा नहीं है।
C. Granted when money has been spent on any service during a financial year in excess of the amount granted for that service. जब उस सेवा के लिए दी गई राशि से अधिक की धनराशि किसी वित्तीय वर्ष के दौरान किसी सेवा पर खर्च की गई हो।
D. It is for raising money in case of emergency through a loan from the RBI. यह RBI से ऋण के माध्यम से आपातकाल के मामले में धन जुटाने के लिए है।

(A) Granted for meeting an unexpected demand upon the resources of India. भारत के संसाधनों पर एक अप्रत्याशित मांग को पूरा करने के लिए दी गई।
Explanation :-
Excess Grant - It is granted when money has been spent on any service during a financial year in excess of the amount granted for that service in the budget for that year. It is voted by the Lok Sabha after the financial year. Before the demands for excess grants are submitted to the Lok Sabha for voting, they must be approved by the Public Accounts Committee of Parliament.
Vote of Credit - It is granted for meeting an unexpected demand upon the resources of India, when on account of the magnitude or the indefinite character of the service, the demand cannot be stated with the details ordinarily given in a budget. Hence, it is like a blank cheque given to the Executive by the Lok Sabha.
Exceptional Grant - It is granted for a special purpose and forms no part of the current service of any financial year.
अतिरिक्त अनुदान - यह उस समय दिया जाता है जब उस वर्ष के लिए बजट में उस सेवा के लिए दी गई राशि से अधिक राशि किसी वित्तीय वर्ष के दौरान किसी सेवा पर खर्च की गई हो। यह वित्तीय वर्ष के बाद लोकसभा द्वारा मतदान किया जाता है। मतदान के लिए लोकसभा में अतिरिक्त अनुदान की मांग प्रस्तुत करने से पहले, उन्हें संसद की लोक लेखा समिति द्वारा अनुमोदित किया जाना चाहिए।
वोट ऑफ क्रेडिट - यह भारत के संसाधनों पर एक अप्रत्याशित मांग को पूरा करने के लिए दिया जाता है, जब परिमाण या सेवा के अनिश्चित चरित्र के कारण, मांग को बजट में दिए गए विवरण के साथ नहीं बताया जा सकता है। इसलिए, यह लोकसभा द्वारा कार्यपालिका को दिए गए एक खाली चेक की तरह है।
असाधारण अनुदान - यह एक विशेष उद्देश्य के लिए दिया जाता है और किसी भी वित्तीय वर्ष की वर्तमान सेवा का कोई हिस्सा नहीं बनता है।

(30)  Which article deals with "Vote on account"? "वोट ऑन अकाउंट" किस अनुच्छेद से संबंधित है?
A. 112
B. 117
C. 116
D. 120

(C) 116
Explanation :-
Article 116 of the Indian Constitution deals with the Vote on Account.
It states that - The House of the People shall have power to make any grant in advance in respect of the estimated expenditure for a part of any financial year pending the completion of the procedure prescribed in article 113 for the voting of such grant and the passing of the law in accordance with the provisions of article 114 in relation to that expenditure.
Vote of account has been a necessary feature since the adoption of the Constitution of India.
By advancing of budget presentation by a month and completion of Budget related legislative business before 31st March, it would pave the way for early completion of Budget cycle and will also preclude the need for seeking appropriation through "Vote on Account".
भारतीय संविधान का अनुच्छेद ११६ वोट ऑन अकाउंट से संबंधित है।
इसमें कहा गया है कि - लोक सभा के पास किसी भी वित्तीय वर्ष के भाग के लिए अनुमानित व्यय के संबंध में अग्रिम रूप से कोई अनुदान देने की शक्ति होगी, जो इस तरह के अनुदान के मतदान के लिए अनुच्छेद 113 में निर्धारित प्रक्रिया को पूरा करने और पारित करने के लिए लंबित है उस व्यय के संबंध में अनुच्छेद 114 के प्रावधानों के अनुसार कानून।
भारत के संविधान को अपनाने के बाद से वोट ऑफ़ अकाउंट एक आवश्यक विशेषता रही है।
एक महीने तक बजट प्रस्तुति को आगे बढ़ाने और 31 मार्च से पहले बजट से संबंधित विधायी व्यवसाय पूरा करने से, यह बजट चक्र के जल्दी पूरा होने का मार्ग प्रशस्त करेगा और "वोट ऑन अकाउंट" के माध्यम से विनियोग की आवश्यकता को भी समाप्त करेगा।

31. Which is true about "Vote on account"? जो "वोट ऑन अकाउंट" के बारे में सच है?
A. It is generally one-fourth of the total estimation. यह आम तौर पर कुल अनुमान का एक-चौथाई है।
B. It is generally one-sixth of the total estimation. यह आम तौर पर कुल अनुमान का एक-छठा हिस्सा होता है।
C. It is generally one-fifth of the total estimation. यह आम तौर पर कुल अनुमान का पांचवां हिस्सा है।
D. It is generally granted for a period of 3 months. यह आम तौर पर 3 महीने की अवधि के लिए दिया जाता है।

B. It is generally one-sixth of the total estimation. यह आम तौर पर कुल अनुमान का एक-छठा हिस्सा होता है।
Explanation :-
Government cannot withdraw money from the Consolidated Fund of India till the enactment of the appropriation bill. This takes time but the government needs money to carry on its normal activities after 31 March (the end of the financial year).
To overcome this functional difficulty, the Constitution has authorised the Lok Sabha to make any grant in advance in respect to the estimated expenditure for a part of the financial year, pending the completion of the voting of the demands for grants and the enactment of the appropriation bill.
This provision is known as the "vote on account". It is passed (or granted) after the general discussion on budget is over.
It is generally granted for two months for an amount equivalent to one-sixth of the total estimation.
सरकार विनियोग विधेयक के अधिनियमित होने तक भारत के समेकित कोष से धन नहीं निकाल सकती है। इसमें समय लगता है लेकिन सरकार को 31 मार्च (वित्तीय वर्ष की समाप्ति) के बाद अपनी सामान्य गतिविधियों को चलाने के लिए धन की आवश्यकता होती है।
इस कार्यात्मक कठिनाई को दूर करने के लिए, संविधान ने लोकसभा को वित्तीय वर्ष के एक हिस्से के लिए अनुमानित खर्च के संबंध में अग्रिम रूप से कोई अनुदान देने के लिए अधिकृत किया है, अनुदान की मांगों के मतदान को पूरा करने और विनियोग के अधिनियमित बिल।
इस प्रावधान को "वोट ऑन अकाउंट" के रूप में जाना जाता है। बजट पर सामान्य चर्चा समाप्त होने के बाद इसे पारित (या प्रदान) किया जाता है।
यह आम तौर पर कुल अनुमान के एक-छठे के बराबर राशि के लिए दो महीने के लिए दिया जाता है।

32.  Which of the following is/are true? निम्नलिखित में से कौन-सा सत्य है / हैं?
(1) West Pakistani Refugees are those that came to India in 1947 and not during other wars with Pakistan. पश्चिमी पाकिस्तानी शरणार्थी वे हैं जो 1947 में भारत आए थे और पाकिस्तान के साथ अन्य युद्धों के दौरान नहीं।
(2) Their children are allowed to be recruited in Central Armed Police Forces (CAPFs). उनके बच्चों को केंद्रीय सशस्त्र पुलिस बलों (CAPFs) में भर्ती होने की अनुमति है।

Code :-
A. Only 1
B. Only 2
C. Both 1 and 2
D. Neither 1 nor 2

(C) Both 1 and 2
Explanation :-
In the wake of Pakistani aggression in Jammu and Kashmir in 1947, about 5764 families who migrated from the then West Pakistan are residing mainly in Jammu, Kathua and Rajouri Districts of Jammu Division.
No separate proposal is under consideration of the Government to recruit the children of West Pakistani Refugees (WPRs) in paramilitary forces.
The children of these refugees are eligible to apply against the vacancies in paramilitary forces earmarked for the State of J&K.
However, these children were allowed to be recruited in Central Armed Police Forces (CAPFs) without the condition of having domicile certificate from the designated authority of the State Govt. of J&K.
Other proof such as names in the Parliamentary voter list, are to be taken as proof of their being refugees from West Pakistan and the certificate issued by the village Numberdar/ Sarpanch can be accepted and taken into account for the purpose of recruitment in CAPFs and Assam Rifles against the quota of the J&K State.
1947 में जम्मू और कश्मीर में पाकिस्तानी आक्रमण के मद्देनजर, तत्कालीन पश्चिमी पाकिस्तान से पलायन करने वाले लगभग 5764 परिवार मुख्य रूप से जम्मू, कठुआ और जम्मू डिवीजन के राजौरी जिलों में रह रहे हैं।
अर्धसैनिक बलों में वेस्ट पाकिस्तानी रिफ्यूजी (डब्ल्यूपीआर) के बच्चों की भर्ती के लिए कोई अलग प्रस्ताव सरकार के पास नहीं है।
इन शरणार्थियों के बच्चे जम्मू-कश्मीर राज्य के लिए निर्धारित अर्धसैनिक बलों में रिक्तियों के खिलाफ आवेदन करने के पात्र हैं।
हालांकि, इन बच्चों को राज्य सरकार के नामित प्राधिकारी से अधिवास प्रमाण पत्र लेने की शर्त के बिना केंद्रीय सशस्त्र पुलिस बलों (CAPFs) में भर्ती होने की अनुमति दी गई थी। जम्मू और कश्मीर के।
अन्य सबूत जैसे कि संसदीय मतदाता सूची में नाम, पश्चिम पाकिस्तान से उनके शरणार्थियों के प्रमाण के रूप में लिए जाने हैं और गाँव नंबरदार / सरपंच द्वारा जारी प्रमाण पत्र को CAPDF और असम में भर्ती के उद्देश्य से स्वीकार किया जा सकता है। जम्मू-कश्मीर राज्य के कोटे के खिलाफ राइफलें।

33. Which of the following is/are true? निम्नलिखित में से कौन-सा सत्य है / हैं?
(1) The West Pakistani Refugees settled in J&K are citizens of J&K. जम्मू-कश्मीर में बसे पश्चिमी पाकिस्तानी शरणार्थी जम्मू-कश्मीर के नागरिक हैं।
(2) However as of yet they do not have the right to vote in Parliamentary Elections. हालाँकि अभी तक उन्हें संसदीय चुनावों में मतदान करने का अधिकार नहीं है।

Code :-
A. Only 1
B. Only 2
C. Both 1 and 2
D. Neither 1 nor 2

(D) Neither 1 nor 2
Explanation :-
The West Pakistani Refugees settled in J&K are citizens of India and they have the right to vote in Parliamentary Elections.
However, they are not permanent residents of the State in terms of J&K Constitution.
They do not enjoy voting rights to the State Assembly and Local Bodies.
The conferment of Permanent Resident Status to the West Pakistani refugees settled in J&K falls within the purview of J&K Constitution, which will enable them for Jobs under State Government, for admission into the State Technical/ Professional institutions and the right to purchase/ acquire land/ immovable properties in the State of J&K.
जम्मू और कश्मीर में बसे पश्चिमी पाकिस्तानी शरणार्थी भारत के नागरिक हैं और उन्हें संसदीय चुनावों में वोट देने का अधिकार है।
हालांकि, वे जम्मू-कश्मीर संविधान के संदर्भ में राज्य के स्थायी निवासी नहीं हैं।
वे राज्य विधानसभा और स्थानीय निकायों को मतदान के अधिकार का आनंद नहीं देते हैं।
जम्मू और कश्मीर में बसे पश्चिमी पाकिस्तानी शरणार्थियों को स्थायी निवासी का दर्जा जम्मू-कश्मीर संविधान के दायरे में आता है, जो उन्हें राज्य सरकार के तहत नौकरियों के लिए सक्षम करेगा, राज्य तकनीकी / व्यावसायिक संस्थानों में प्रवेश के लिए और जमीन खरीदने / अधिग्रहित करने का अधिकार। जम्मू और कश्मीर राज्य में अचल संपत्तियां।

34. Which of the following states are among the high TFR states of India? निम्नलिखित में से कौन-सा राज्य भारत के उच्च TFR राज्यों में से है?
(1) Uttar Pradesh
(2) Chhattisgarh
(3) Assam

Code :-
A. 1
B. 2, 3
C. 1, 2
D. All of the above

(D) All of the above
Explanation :-
Total Fertility Rate (TFR) is the average number of children expected to be born per woman during her entire span of reproductive period.
High Total Fertility Rate (TFR) states of Uttar Pradesh, Bihar, Rajasthan, Madhya Pradesh, Chhattisgarh, Jharkhand and Assam that constitute 44% of the country’s population.
In India the Total Fertility Rate is 2.4.
Among the bigger States, Tamilnadu and West Bengal has the lowest TFR of 1.7
The highest TFR recorded is 3.6 for Bihar.
TFR is considered to be a useful indicator for analysing the prospects for population stabilization.
कुल प्रजनन दर (टीएफआर) प्रजनन अवधि के पूरे अवधि के दौरान प्रति महिला जन्म लेने वाले बच्चों की औसत संख्या है।
उच्च कुल प्रजनन दर (टीएफआर) उत्तर प्रदेश, बिहार, राजस्थान, मध्य प्रदेश, छत्तीसगढ़, झारखंड और असम राज्यों में है जो देश की आबादी का 44% है।
भारत में कुल प्रजनन दर 2.4 है।
बड़े राज्यों में, तमिलनाडु और पश्चिम बंगाल में सबसे कम TFR 1.7 है
बिहार के लिए सबसे अधिक TFR 3.6 दर्ज किया गया है।
TFR को जनसंख्या स्थिरीकरण की संभावनाओं के विश्लेषण के लिए एक उपयोगी संकेतक माना जाता है।

35. Which of the following is/are true regarding River Ganga (Rejuvenation, Protection and Management) Authorities Order, 2016? गंगा नदी (कायाकल्प, संरक्षण और प्रबंधन) प्राधिकरण आदेश, 2016 के बारे में निम्नलिखित में से कौन-सा सच है / हैं?
(1) NMCG earlier did not have powers to issue directions to the concerned authorities/polluters.  एनएमसीजी के पास पहले संबंधित अधिकारियों / प्रदूषकों को निर्देश जारी करने की शक्तियां नहीं थीं।
(2) NMCG will, however, take action only in the event when required action is not taken by CPCB. एनएमसीजी, तब ही घटना में कार्रवाई करेगा जब आवश्यक कार्रवाई सीपीसीबी द्वारा नहीं की जाएगी।

Code :-
A. Only 1
B. Only 2
C. Both 1 and 2
D. Neither 1 nor 2

(C) Both 1 and 2
Explanation :-
NMCG will, however, take action only in the event when required action is not taken by CPCB. CPCB shall also take action jointly with NMCG under the provisions of said Act.
Although, the NMCG has been functional as a registered Society since 2012 its role has been largely limited to fund the projects to implementing organizations.
It neither had the mandate to take cognizance of various threats to river Ganga nor the powers to issue directions to the concerned authorities/polluters.
While the organization has been made responsible as custodian of river Ganga in both public eye as well as various courts, the mission is grossly ill-equipped to handle such expectations.
It is expected that the move will ensure effective abatement of pollution and rejuvenation of the River Ganga; maintain ecological flows in the River; impose restrictions on polluting industries; and carry out inspections to ensure compliance.
In addition, it is proposed to maintain and disseminate data and carry out research on the condition of the river.
हालाँकि, NMCG केवल तभी कार्रवाई करेगा जब आवश्यक कार्रवाई CPCB द्वारा नहीं की जाएगी। उक्त अधिनियम के प्रावधानों के तहत CPCB NMCG के साथ संयुक्त रूप से कार्रवाई करेगा।
हालांकि, 2012 से NMCG एक पंजीकृत सोसायटी के रूप में कार्य कर रही है, इसकी भूमिका काफी हद तक संगठनों को लागू करने के लिए परियोजनाओं को निधि देने तक सीमित है।
इसमें न तो गंगा नदी को विभिन्न खतरों का संज्ञान लेने का अधिकार था और न ही संबंधित प्राधिकारियों / मतदाताओं को निर्देश जारी करने की शक्तियां।
जबकि इस संगठन को गंगा नदी के संरक्षक के रूप में सार्वजनिक आंखों और विभिन्न अदालतों के रूप में जिम्मेदार बनाया गया है, मिशन इस तरह की अपेक्षाओं को संभालने के लिए व्यापक रूप से बीमार है।
यह उम्मीद की जाती है कि इस कदम से गंगा नदी के प्रदूषण और कायाकल्प की प्रभावी व्यवस्था सुनिश्चित होगी; नदी में पारिस्थितिक प्रवाह बनाए रखना; प्रदूषणकारी उद्योगों पर प्रतिबंध; और अनुपालन सुनिश्चित करने के लिए निरीक्षण करना।
इसके अलावा, डेटा को बनाए रखने और प्रसारित करने और नदी की स्थिति पर शोध करने का प्रस्ताव है।

36. Which of the following is/are to carry out work as "Authority" under the River Ganga (Rejuvenation, Protection and Management) Authorities Order, 2016? गंगा नदी (कायाकल्प, संरक्षण और प्रबंधन) प्राधिकरण आदेश, 2016 के तहत निम्नलिखित में से कौन-सा कार्य "प्राधिकरण" के रूप में किया जाता है?
(1) NMCG
(2) State Ganga Committees
(3) District Ganga Committees

Code :-
A. 1, 3
B. 2, 3
C. 1, 2
D. All of the above

(D) All of the above
Explanation :-
The order declares National Mission for Clean Ganga (NMCG) as an Authority with powers to issue directions and also to exercise the powers under the Environment (Protection) Act, 1986 to enable it to carry out efficiently its mandate.
NMCG will comply with the decisions and directions of the National Ganga Council and implement the Ganga Basin Management Plan approved by it; co-ordinate and carry out all activities.
At the State level, it is proposed to create the State Ganga Committees in each of the defined States as Authority, to function as Authorities in respect of each State and perform the superintendence, direction and control over the District Ganga Protection Committees under their jurisdiction.
Similarly, the District Ganga Committees in each of the Ganga Bank Districts will carry out the assigned tasks as an Authority at the district level, to take cognizance of local threats and needs of river Ganga and conceptualize such measures as necessary to ensure overall quality of water in river Ganga and monitor various projects being implemented.
यह आदेश राष्ट्रीय स्वच्छ मिशन (एनएमसीजी) के लिए दिशा-निर्देश जारी करने के साथ-साथ पर्यावरण (संरक्षण) अधिनियम, 1986 के तहत शक्तियों का प्रयोग करने के लिए एक अधिकार के रूप में घोषित करता है ताकि इसे कुशलतापूर्वक अपने जनादेश को पूरा करने में सक्षम बनाया जा सके।
NMCG राष्ट्रीय गंगा परिषद के निर्णयों और निर्देशों का अनुपालन करेगा और इसके द्वारा अनुमोदित गंगा बेसिन प्रबंधन योजना को लागू करेगा; समन्वय और सभी गतिविधियों को अंजाम देना।
राज्य स्तर पर, प्राधिकरण के रूप में परिभाषित राज्यों में से प्रत्येक में राज्य गंगा समितियां बनाने, प्रत्येक राज्य के संबंध में प्राधिकरण के रूप में कार्य करने और अपने क्षेत्राधिकार के लिए जिला गंगा संरक्षण समितियों पर अधीक्षण, निर्देशन और नियंत्रण करने का प्रस्ताव है।
इसी प्रकार, गंगा बैंक जिलों में से प्रत्येक में जिला गंगा समितियों को जिला स्तर पर प्राधिकरण के रूप में सौंपे गए कार्यों को पूरा करना होगा, गंगा नदी की स्थानीय खतरों और आवश्यकताओं का संज्ञान लेना और पानी की समग्र गुणवत्ता सुनिश्चित करने के लिए आवश्यक उपायों जैसे संकल्पना। गंगा नदी में और कार्यान्वित की जा रही विभिन्न परियोजनाओं की निगरानी करें।

37. Which of the following is/are true regarding River Ganga (Rejuvenation, Protection and Management) Authorities Order, 2016? गंगा नदी (कायाकल्प, संरक्षण और प्रबंधन) प्राधिकरण आदेश, 2016 के बारे में निम्नलिखित में से कौन-सा सच है / हैं?
(1) NGRBA will come under National Council for River Ganga (Rejuvenation, Protection and Management). एनजीआरबीए गंगा नदी (कायाकल्प, संरक्षण और प्रबंधन) के लिए राष्ट्रीय परिषद के अंतर्गत आएगा।
(2) National Council for River Ganga (Rejuvenation, Protection and Management) will be headed by the Minister of Water Resources. गंगा नदी के लिए राष्ट्रीय परिषद (कायाकल्प, संरक्षण और प्रबंधन) का नेतृत्व जल संसाधन मंत्री करेंगे।

Code :-
A. Only 1
B. Only 2
C. Both 1 and 2
D. Neither 1 nor 2

(D) Neither 1 nor 2
Explanation :-
The order creates the National Council for River Ganga (Rejuvenation, Protection and Management), as an Authority under the Chairperson of Hon’ble Prime Minister, in place of the existing NGRBA for overall responsibility for superintendence of pollution prevention and rejuvenation of river Ganga Basin.
The government strengthened and elevated the National Mission on Clean Ganga (NMCG) to the level of an ‘Authority’ with powers to enforce pollution laws and penalize defaulters.
The NMCG, currently registered as a society, works under the Water Resources Ministry as the implementing agency of policies framed by the National Ganga River Basin Authority (NGRBA) that was created in 2010.
It will now come under National Council for River Ganga (Rejuvenation, Protection and Management).
यह आदेश राष्ट्रीय नदी परिषद गंगा (कायाकल्प, संरक्षण और प्रबंधन) के लिए एक अधिकार के रूप में माननीय प्रधान मंत्री की अध्यक्षता में, मौजूदा एनजीआरबीए के स्थान पर प्रदूषण की रोकथाम और नदी गंगा नदी के कायाकल्प के अधीक्षण के लिए समग्र जिम्मेदारी के लिए है। ।
सरकार ने प्रदूषण कानूनों को लागू करने और बकाएदारों को दंडित करने की शक्तियों के साथ (प्राधिकरण ’के स्तर पर स्वच्छ गंगा (NMCG) पर राष्ट्रीय मिशन को मजबूत और ऊंचा किया।
वर्तमान में एक सोसायटी के रूप में पंजीकृत NMCG, राष्ट्रीय गंगा नदी बेसिन प्राधिकरण (NGRBA) द्वारा बनाई गई नीतियों की कार्यान्वयन एजेंसी के रूप में जल संसाधन मंत्रालय के तहत काम करती है जो 2010 में बनाई गई थी।
यह अब गंगा नदी के लिए राष्ट्रीय परिषद (कायाकल्प, संरक्षण और प्रबंधन) के तहत आएगा।

38. The famous Berubari Union territory was transferred by which Constitutional Amendment Act? प्रसिद्ध बेरुबरी केंद्र शासित प्रदेश किस संवैधानिक संशोधन अधिनियम द्वारा स्थानांतरित किया गया था?
A. 9th
B. 10th
C. 12th
D. 14th

(A) 9th
Explanation :-
The Ninth Constitutional Amendment Act of 1960 facilitated the cession of Indian Territory of Berubari Union (located in West Bengal) to Pakistan (East Pakistan) as provided in the Indo-Pakistan Agreement (1958).
This Agreement was also known as Nehru-Noon Agreement of 1958.
The Supreme Court held that the parliament of India is not competent to make a law under article 3 for the implementation of the Nehru-Noon Agreement.
This was followed by an amendment of the Constitution by parliament using power of Article 368. The result was the Constitution (9th Amendment) Act 1960.
1960 के नौवें संवैधानिक संशोधन अधिनियम ने भारत-पाकिस्तान समझौते (1958) में प्रदत्त भारतीय बेरुबरी संघ (पश्चिम बंगाल में स्थित) के पाकिस्तान (पूर्वी पाकिस्तान) के कब्जे को सुविधाजनक बनाया।
इस समझौते को 1958 के नेहरू-नून समझौते के रूप में भी जाना जाता था।
सर्वोच्च न्यायालय ने माना कि भारत की संसद नेहरू-दोपहर समझौते के कार्यान्वयन के लिए अनुच्छेद 3 के तहत एक कानून बनाने के लिए सक्षम नहीं है।
इसके बाद अनुच्छेद 368 की शक्ति का उपयोग करके संसद द्वारा संविधान में संशोधन किया गया। परिणाम संविधान (9 वां संशोधन) अधिनियम 1960 था।

39. Which of the following acts are repealed by the Admiralty (Jurisdiction and Settlement of Maritime Claims) Bill 2016? निम्नलिखित में से कौन सा कार्य एडमिरल्टी (क्षेत्राधिकार और समुद्री दावों के निपटान) द्वारा निरस्त किया जाता है?
(1) The Admiralty Court Act, 1840
(2) Colonial Courts of Admiralty (India) Act, 1891
(3) Colonial Courts of Admiralty Act, 1890

Code :-
A. 1, 3
B. 1, 2
C. 2, 3
D. All of the above

(D) All of the above
Explanation :-
The Bill consolidates the existing laws relating to admiralty jurisdiction of courts, admiralty proceedings on maritime claims, arrest of vessels and related issues. विधेयक न्यायालयों की एडमिरलिटी क्षेत्राधिकार, समुद्री दावों पर एडमिरल्टी कार्यवाही, जहाजों की गिरफ्तारी और संबंधित मुद्दों से संबंधित मौजूदा कानूनों को समेकित करता है।
It also repeals five obsolete British statues on admiralty jurisdiction in civil matters, namely,
(A) The Admiralty Court Act, 1840
(B) The Admiralty Court Act, 1861,
(C) Colonial Courts of Admiralty Act, 1890,
(D) Colonial Courts of Admiralty (India) Act, 1891, and
(E) The provisions of the Letters Patent, 1865 applicable to the admiralty jurisdiction of the Bombay, Calcutta and Madras High Courts.

40.  Which of the following is/are true regarding Admiralty (Jurisdiction and Settlement of Maritime Claims) Bill 2016? निम्नलिखित में से कौन सा / से एडमिरल्टी (क्षेत्राधिकार और समुद्री दावों के निपटान) विधेयक 2016 के बारे में सच है?
(1) Even Inland vessels and vessels under construction are by default included in the admiralty jurisdiction. यहां तक ​​कि अंतर्देशीय जहाजों और निर्माणाधीन जहाजों को डिफ़ॉल्ट रूप से एडमिरलिटी क्षेत्राधिकार में शामिल किया गया है।
(2) It also applies to warships and naval auxiliary and vessels used for non-commercial purposes. यह युद्धपोतों और नौसैनिक सहायक और गैर-वाणिज्यिक प्रयोजनों के लिए उपयोग किए जाने वाले जहाजों पर भी लागू होता है।

Code :-
A. Only 1
B. Only 2
C. Both 1 and 2
D. Neither 1 nor 2

(D) Neither 1 nor 2
Explanation :-
It applies to every vessel irrespective of place of residence or domicile of owner.
Inland vessels and vessels under construction are excluded from its application but the Central Government is empowered to make it applicable to these vessels also by a notification if necessary.
It does not apply to warships and naval auxiliary and vessels used for non-commercial purposes.
In respect of aspects on which provisions are not laid down in the Bill, the Civil Procedure Code, 1908 is applicable.
यह निवास के स्थान या मालिक के अधिवास के बावजूद हर बर्तन पर लागू होता है।
निर्माणाधीन अंतर्देशीय जहाजों और जहाजों को इसके आवेदन से बाहर रखा गया है, लेकिन केंद्र सरकार को इन जहाजों पर लागू करने के लिए एक अधिसूचना द्वारा आवश्यक बनाने के लिए सशक्त बनाया गया है।
यह युद्धपोतों और नौसैनिक सहायक और गैर-वाणिज्यिक प्रयोजनों के लिए उपयोग किए जाने वाले जहाजों पर लागू नहीं होता है।
जिन पहलुओं पर विधेयक में प्रावधान नहीं हैं, उन पहलुओं के संबंध में, नागरिक प्रक्रिया संहिता, 1908 लागू है।

41. Which of the following is/are true regarding Admiralty (Jurisdiction and Settlement of Maritime Claims) Bill 2016? निम्नलिखित में से कौन सा / से एडमिरल्टी (क्षेत्राधिकार और समुद्री दावों के निपटान) विधेयक 2016 के बारे में सच है?
(1) The Bill confers admiralty jurisdiction on High Courts located in coastal states of India. विधेयक भारत के तटीय राज्यों में स्थित उच्च न्यायालयों पर अधिकार क्षेत्र को स्वीकार करता है।
(2) However, the jurisdiction is not extendable to the exclusive economic zone. हालाँकि, क्षेत्राधिकार अनन्य आर्थिक क्षेत्र के लिए विस्तार योग्य नहीं है।

Code :-
A. Only 1
B. Only 2
C. Both 1 and 2
D. Neither 1 nor 2

(A) Only 1
Explanation :-
The Bill confers admiralty jurisdiction on High Courts located in coastal states of India and this jurisdiction extends up to territorial waters.
The jurisdiction is extendable, by a Central Government notification, up to exclusive economic zone or any other maritime zone of India or islands constituting part of the territory of India.
The jurisdiction is for adjudicating on a set of maritime claims listed in the Bill.
In order to ensure security against a maritime claim a vessel can be arrested in certain circumstances.
The liability in respect of selected maritime claims on a vessel passes on to its new owners by way of maritime liens subject to a stipulated time limit.
यह विधेयक भारत के तटीय राज्यों में स्थित उच्च न्यायालयों पर प्रशंसा अधिकार क्षेत्र को स्वीकार करता है और यह अधिकार क्षेत्र क्षेत्रीय जल तक फैला हुआ है।
न्यायिक क्षेत्र विस्तार योग्य है, एक केंद्र सरकार की अधिसूचना द्वारा, विशेष आर्थिक क्षेत्र या भारत के किसी अन्य समुद्री क्षेत्र या भारत के क्षेत्र का हिस्सा बनने वाले द्वीपों तक।
अधिकार क्षेत्र विधेयक में सूचीबद्ध समुद्री दावों के एक सेट पर स्थगित करने के लिए है।
समुद्री दावे के खिलाफ सुरक्षा सुनिश्चित करने के लिए कुछ परिस्थितियों में एक पोत को गिरफ्तार किया जा सकता है।
एक जहाज पर चयनित समुद्री दावों के संबंध में देयता, अपने नए मालिकों को तय समय सीमा के अधीन समुद्री देयता के रास्ते से गुजरती है।

42. Admiralty jurisdiction relates to :- एडमिरलिटी क्षेत्राधिकार से संबंधित है: -
A. Powers of the Indian Navy in the EEZ
B. Powers of the Indian Navy in international waters
C. Powers of the High Courts in respect of claims associated with transport by sea
D. Powers of other countries in Indian waters.

(C) Powers of the High Courts in respect of claims associated with transport by sea समुद्र द्वारा परिवहन से जुड़े दावों के संबंध में उच्च न्यायालयों की शक्तियाँ
Explanation :-
Admiralty jurisdiction relates to powers of the High Courts in respect of claims associated with transport by sea and navigable waterways.
It deals with matters including marine commerce, marine navigation, marine salvaging, shipping, sailors, and the transportation of passengers and goods by sea. Admiralty law also covers many commercial activities, although land based or occurring wholly on land, that are maritime in character.
Admiralty law is distinguished from the Law of the Sea, which is a body of public international law dealing with navigational rights, mineral rights, jurisdiction over coastal waters and international law governing relationships between nations.
समुद्र और नौगम्य जलमार्गों द्वारा परिवहन से जुड़े दावों के संबंध में उच्च न्यायालयों की शक्तियों से संबंधित है।
यह समुद्री वाणिज्य, समुद्री नेविगेशन, समुद्री निस्तारण, नौवहन, नाविकों और समुद्र के द्वारा यात्रियों और वस्तुओं के परिवहन सहित मामलों से संबंधित है। एडमिरल्टी कानून में कई वाणिज्यिक गतिविधियों को भी शामिल किया गया है, हालांकि भूमि पर आधारित या पूरी तरह से होने वाली भूमि, जो चरित्र में समुद्री है।
एडमिरल्टी कानून सागर के कानून से अलग है, जो नौसैनिक अधिकारों, खनिज अधिकारों, तटीय जल पर अधिकार क्षेत्र और राष्ट्रों के बीच संबंधों को संचालित करने वाले अंतरराष्ट्रीय कानून से संबंधित एक सार्वजनिक अंतरराष्ट्रीय कानून है।

43. Which of the following is/are true regarding draft bill Major Port Authorities Act, 2016? ड्राफ्ट बिल मेजर पोर्ट अथॉरिटीज़ एक्ट, 2016 के बारे में निम्नलिखित में से कौन सा सही है / हैं?
(1) The status of Port Authority will be deemed as ‘local authority’. पोर्ट प्राधिकरण की स्थिति को 'स्थानीय प्राधिकारी' माना जाएगा।
(2) The need for Government approvals for raising loans has been dispensed with. ऋण बढ़ाने के लिए सरकार की मंजूरी की आवश्यकता के साथ भेज दिया गया है।

Code :-
A. Only 1
B. Only 2
C. Both 1 and 2
D. Neither 1 nor 2

(C) Both 1 and 2
Explanation :-
The status of Port Authority will be deemed as ‘local authority’ under the provisions of the General Clauses Act, 1887 & other applicable Statutes so that it could prepare appropriate regulations in respect of the area within the port limits to the exclusion of any Central, State of local laws.
The need for Government approvals for raising loans, appointment of consultants, execution of contracts and creation of service posts have been dispensed with.
The Board of Port Authority have been delegated power to raise loans and issue security for the purpose of capital expenditure and working capital requirement.
Concept of internal audit of the functions and activities of the Central Ports has been introduced on the lines of Companies Act, 2015.
पोर्ट प्राधिकरण की स्थिति को जनरल क्लॉज़ एक्ट, 1887 और अन्य लागू क़ानूनों के प्रावधानों के तहत 'स्थानीय प्राधिकरण' के रूप में समझा जाएगा, ताकि यह किसी केंद्रीय के बहिष्करण के लिए पोर्ट की सीमा के भीतर क्षेत्र के संबंध में उचित नियम तैयार कर सके, स्थानीय कानूनों का राज्य।
ऋण बढ़ाने, सलाहकारों की नियुक्ति, अनुबंधों को लागू करने और सेवा पदों के सृजन के लिए सरकार की मंजूरी की आवश्यकता को दूर कर दिया गया है।
पोर्ट अथॉरिटी के बोर्ड को पूंजी व्यय और कार्यशील पूंजी की आवश्यकता के उद्देश्य से ऋण जुटाने और सुरक्षा जारी करने के लिए शक्ति सौंपी गई है।
सेंट्रल पोर्ट्स के कार्यों और गतिविधियों की आंतरिक लेखापरीक्षा की संकल्पना कंपनी अधिनियम, 2015 की तर्ज पर शुरू की गई है।

44. Which of the following is/are true regarding draft bill Major Port Authorities Act, 2016? ड्राफ्ट बिल मेजर पोर्ट अथॉरिटीज़ एक्ट, 2016 के बारे में निम्नलिखित में से कौन सा सही है / हैं?
(1) Composition of board has been simplified consisting of 10 members including 5 independent members. बोर्ड की संरचना को 5 स्वतंत्र सदस्यों सहित 10 सदस्यों से मिलकर सरल बनाया गया है।
(2) Provisions of CSR & development of infrastructure by Port Authority have been introduced. पोर्ट प्राधिकरण द्वारा सीएसआर और बुनियादी ढांचे के विकास के प्रावधान पेश किए गए हैं।

Code :-
A. Only 1
B. Only 2
C. Both 1 and 2
D. Neither 1 nor 2

(B) Only 2
Explanation :-
The Ministry of Shipping has prepared a draft Bill “Major Port Authorities Act, 2016” to replace the Major Port Trusts Act, 1963 with a view to promote the port infrastructure and facilitate trade and commerce.
The proposed bill aims at giving more autonomy and flexibility to the major Ports and to bring in professional approach in their governance. This will help to impart faster and transparent decision making which will benefit the stakeholders.
Composition of board has been simplified. The board will consist of 10 members including 3 to 4 independent members instead of 17-19 under the present Port Trust Model.
Provisions has been made for inclusion of 3 functional heads of Major Ports as Members in the Board apart from a Government Nominee Member and a Labour Nominee Member.
Provisions of CSR & development of infrastructure by Port Authority have been introduced.
शिपिंग मंत्रालय ने पोर्ट इंफ्रास्ट्रक्चर को बढ़ावा देने और व्यापार और वाणिज्य को बढ़ावा देने के उद्देश्य से मेजर पोर्ट ट्रस्ट्स अधिनियम, 1963 को बदलने के लिए एक विधेयक "मेजर पोर्ट अथॉरिटीज एक्ट, 2016" तैयार किया है।
प्रस्तावित विधेयक का उद्देश्य प्रमुख बंदरगाहों को अधिक स्वायत्तता और लचीलापन देना और उनके शासन में पेशेवर दृष्टिकोण लाना है। यह तेजी से और पारदर्शी निर्णय लेने में मदद करेगा जिससे हितधारकों को लाभ होगा।
बोर्ड की संरचना को सरल बनाया गया है। बोर्ड में वर्तमान पोर्ट ट्रस्ट मॉडल के तहत 17-19 के बजाय 3 से 4 स्वतंत्र सदस्यों सहित 10 सदस्य शामिल होंगे।
बोर्ड में सदस्य के रूप में 3 कार्यात्मक प्रमुखों को शामिल करने के लिए प्रावधान किए गए हैं, एक सरकारी सदस्य और एक श्रमिक नामित सदस्य के अलावा बोर्ड में सदस्य।
पोर्ट प्राधिकरण द्वारा सीएसआर और बुनियादी ढांचे के विकास के प्रावधान पेश किए गए हैं।

45. Which of the following is the newly added clause regarding cancellation of OCI registration? ओसीआई पंजीकरण रद्द करने के संबंध में निम्नलिखित में से कौन सा नया जोड़ा गया खंड है?
A. Within five years of registration has been sentenced to imprisonment for two years or more. पंजीकरण के पांच साल के भीतर दो साल या उससे अधिक के कारावास की सजा सुनाई गई है।
B. It is in the interest of sovereignty and security of India. यह भारत की संप्रभुता और सुरक्षा के हित में है।
C. OCI has violated any law that is in force in the country. OCI ने देश में लागू होने वाले किसी भी कानून का उल्लंघन किया है।
D. OCI has registered through fraud. ओसीआई ने धोखाधड़ी के माध्यम से पंजीकरण किया है।

(C) OCI has violated any law that is in force in the country. OCI ने देश में लागू होने वाले किसी भी कानून का उल्लंघन किया है।

Explanation :-
The Citizenship Act, 1955 provides that the central government may cancel registration of OCIs on certain grounds.
These include:
(i) if the OCI has registered through fraud, or
(ii) within five years of registration has been sentenced to imprisonment for two years or more, or
(iii) it becomes necessary in the interest of sovereignty and security of India, etc.
The Bill adds one more ground for cancelling registration, that is, if the OCI has violated any law that is in force in the country.
नागरिकता अधिनियम, 1955 प्रदान करता है कि केंद्र सरकार कुछ आधारों पर ओसीआई के पंजीकरण को रद्द कर सकती है।
इसमें शामिल है:
(i) यदि ओसीआई ने धोखाधड़ी के माध्यम से पंजीकरण किया है, या
(ii) पंजीकरण के पांच साल के भीतर दो साल या उससे अधिक की कैद की सजा सुनाई गई है, या
(iii) यह भारत की संप्रभुता और सुरक्षा के हित में आवश्यक हो जाता है, आदि।
विधेयक पंजीकरण को रद्द करने के लिए एक और आधार जोड़ता है, अर्थात, यदि ओसीआई ने देश में लागू किसी कानून का उल्लंघन किया है।

46. Which of the following is/are true regarding The Citizenship (Amendment) Bill, 2016? नागरिकता (संशोधन) विधेयक, 2016 के बारे में निम्नलिखित में से कौन-सा सही है / हैं?
(1) Hindus, Sikhs, Buddhists, Jains, Parsis and Christians from Afghanistan, Bangladesh and Pakistan will not be treated as illegal immigrants.
(2) For Hindus, Sikhs, Buddhists, Jains, Parsis and Christians from Afghanistan, Bangladesh and Pakistan, , the 11 years’ requirement for naturalization will be increased to 15 years.

Code :-
A. Only 1
B. Only 2
C. Both 1 and 2
D. Neither 1 nor 2

(A) Only 1
Explanation :-
The Bill amends the Act to provide that that the following groups of persons will not be treated as illegal migrants: (i) Hindus, Sikhs, Buddhists, Jains, Parsis and Christians from Afghanistan, Bangladesh and Pakistan, (ii) who have been exempted from provisions of the Passport (Entry into India) Act, 1920, and the Foreigners Act, 1946 by the central government. The 1920 Act mandates foreigners to carry passport, while the 1946 Act regulates the entry and departure of foreigners in India.
Citizenship by naturalization: The Act allows a person to apply for citizenship by naturalization, if the person meets certain qualifications. One of the qualifications is that the person must have resided in India or been in service of the central government for at least 11 years before applying for citizenship.
The Bill creates an exception for Hindus, Sikhs, Buddhists, Jains, Parsis and Christians from Afghanistan, Bangladesh and Pakistan, with regard to this qualification. For these groups of persons, the 11 years’ requirement will be reduced to six years.
विधेयक इस अधिनियम में संशोधन करता है कि यह सुनिश्चित करने के लिए कि व्यक्तियों के निम्नलिखित समूहों को अवैध प्रवासी नहीं माना जाएगा: (i) हिंदू, सिख, बौद्ध, जैन, पारसी और ईसाई अफगानिस्तान, बांग्लादेश और पाकिस्तान से (ii) जिन्हें छूट दी गई है केंद्र सरकार द्वारा पासपोर्ट (भारत में प्रवेश) अधिनियम, 1920 और विदेशियों अधिनियम, 1946 के प्रावधानों से। 1920 का अधिनियम विदेशियों को पासपोर्ट ले जाने के लिए बाध्य करता है, जबकि 1946 अधिनियम भारत में विदेशियों के प्रवेश और प्रस्थान को नियंत्रित करता है।
प्राकृतिकिकरण द्वारा नागरिकता: अधिनियम किसी व्यक्ति को प्राकृतिक योग्यता द्वारा नागरिकता के लिए आवेदन करने की अनुमति देता है, यदि व्यक्ति कुछ योग्यताओं को पूरा करता है। योग्यताओं में से एक यह है कि नागरिकता के लिए आवेदन करने से पहले व्यक्ति कम से कम 11 साल तक भारत में रहा होगा या केंद्र सरकार की सेवा में रहा होगा।
इस योग्यता के संबंध में विधेयक अफगानिस्तान, बांग्लादेश और पाकिस्तान के हिंदुओं, सिखों, बौद्धों, जैनियों, पारसियों और ईसाइयों के लिए एक अपवाद बनाता है। व्यक्तियों के इन समूहों के लिए, 11 वर्ष की आवश्यकता को घटाकर छह वर्ष कर दिया जाएगा।

47. Which of the following is/are objectives of National Council for Tribal Welfare? निम्नलिखित में से कौन सा / से जनजातीय कल्याण परिषद के उद्देश्य हैं?
(1) Implement Tribal Plans जनजातीय योजनाओं को लागू करना
(2) Provide broad policy guidelines to bring about improvement in the lives of the Scheduled Tribes community अनुसूचित जनजाति समुदाय के जीवन में सुधार लाने के लिए व्यापक नीति दिशानिर्देश प्रदान करें
(3) Review the implementation of the Forest Rights Act वन अधिकार अधिनियम के कार्यान्वयन की समीक्षा करें

Code :-
A. 1, 3
B. 2, 3
C. 1, 2
D. All of the above

(B) 2, 3
Explanation :-
The National Council for Tribal Welfare set up objectives are :-
provide broad policy guidelines to bring about improvement in the lives of the Scheduled Tribes community in the country
review the implementation of the Forest Rights Act
review the implementation of Fifth Schedule and Sixth Schedule of the Constitution
monitor the implementation of Tribal Sub Plan
monitor programmes aimed at protecting particularly vulnerable Tribal Groups.
राष्ट्रीय जनजातीय कल्याण परिषद के उद्देश्य हैं: -
देश में अनुसूचित जनजाति समुदाय के जीवन में सुधार लाने के लिए व्यापक नीति दिशानिर्देश प्रदान करें
वन अधिकार अधिनियम के कार्यान्वयन की समीक्षा करें
पांचवीं अनुसूची और संविधान की छठी अनुसूची के कार्यान्वयन की समीक्षा करें
जनजातीय उप योजना के कार्यान्वयन की निगरानी करना
विशेष रूप से कमजोर जनजातीय समूहों की सुरक्षा के उद्देश्य से निगरानी कार्यक्रम।

48. National Council for Tribal Welfare is chaired by :-
A. Chairmanship is on rotational basis with each state’s Tribal Minister as head. प्रमुख के रूप में प्रत्येक राज्य के जनजातीय मंत्री के साथ अध्यक्षीय पद पर आसीन है।
B. Tribal Welfare Minister at the Centre
C. PM
D. President

(C) PM
Explanation :-
National Council for Tribal Welfare is chaired by the Prime Minister.
In addition to the Council, a Standing Committee has also been constituted. This will be chaired by the Minister of Tribal Affairs.
The States of Andhra Pradesh, Chhattisgarh, Gujarat, Himachal Pradesh, Jharkhand, Madhya Pradesh, Maharashtra, Odisha and Rajasthan are covered under the Fifth Schedule of the Constitution and have been entrusted with special responsibilities on matter pertaining to the welfare and advancement of the Scheduled Tribes.
राष्ट्रीय जनजाति कल्याण परिषद की अध्यक्षता प्रधान मंत्री द्वारा की जाती है।
परिषद के अलावा, एक स्थायी समिति का भी गठन किया गया है। इसकी अध्यक्षता जनजातीय मामलों के मंत्री करेंगे।
आंध्र प्रदेश, छत्तीसगढ़, गुजरात, हिमाचल प्रदेश, झारखंड, मध्य प्रदेश, महाराष्ट्र, ओडिशा और राजस्थान राज्य संविधान की पांचवीं अनुसूची के तहत आते हैं और अनुसूचित जाति के कल्याण और उन्नति से संबंधित मामले पर विशेष जिम्मेदारियां सौंपी गई हैं। जनजातियों।

49. Which constitutional amendment was struck down out of the following? निम्नलिखित में से किस संवैधानिक संशोधन पर प्रहार किया गया था?
A. 100
B. 101
C. 102
D. 99

(D) 99
Explanation :-
The amendment provides for the formation of a National Judicial Appointments Commission.
It was a proposed body which would have been responsible for the appointment and transfer of judges to the higher judiciary in India.
16 State assemblies out of 29 States ratified the Central Legislation, enabling the President of India to give assent to the bill.
The amendment was struck down by the Supreme Court on 16 October 2015.
The SC upheld the Collegium System.
संशोधन राष्ट्रीय न्यायिक नियुक्ति आयोग के गठन का प्रावधान करता है।
यह एक प्रस्तावित निकाय था जो भारत में उच्च न्यायपालिका के लिए न्यायाधीशों की नियुक्ति और हस्तांतरण के लिए जिम्मेदार था।
29 राज्यों में से 16 राज्यों की विधानसभाओं ने केंद्रीय विधान की पुष्टि की, जिससे भारत के राष्ट्रपति बिल को स्वीकृति प्रदान कर सके।
16 अक्टूबर 2015 को सुप्रीम कोर्ट द्वारा संशोधन को रद्द कर दिया गया था।
SC ने कॉलेजियम सिस्टम को बरकरार रखा।

50. Constitution (One Hundred and First Amendment) Act is for :- संविधान (एक सौ और पहला संशोधन) अधिनियम इस प्रकार है: -
A. NJAC
B. GST
C. Exchange of certain enclave territories with Bangladesh बांग्लादेश के साथ कुछ एन्क्लेव प्रदेशों का आदान-प्रदान
D. Added Part-IXB - The Co-operative Societies

(B) GST
Explanation :-
The Constitution (One Hundred and Twenty-second Amendment) Bill, 2016, for introduction of Goods and Services tax in the country was accorded assent by the President on 8th September, 2016, and the same has been notified as the Constitution (One Hundred and First Amendment) Act, 2016.
As per Article 279A (1) of the amended Constitution, the GST Council has to be constituted by the President within 60 days of the commencement of Article 279A.
8 सितंबर, 2016 को राष्ट्रपति द्वारा देश में वस्तु एवं सेवा कर लागू करने के लिए संविधान (एक सौ और दूसरा संशोधन) विधेयक, 2016 को मान्यता दी गई थी और इसे संविधान (एक सौ) के रूप में अधिसूचित किया गया था। पहला संशोधन) अधिनियम, 2016।
संशोधित संविधान के अनुच्छेद 279 ए (1) के अनुसार, जीएसटी परिषद को अनुच्छेद 279 ए के शुरू होने के 60 दिनों के भीतर राष्ट्रपति द्वारा गठित किया जाना है।

51. Ex-officio Secretary to the GST Council is :- जीएसटी परिषद के पदेन सचिव हैं: -
A. Secretary (Finance)
B. Head of NITI Aayog
C. Cabinet Secretary
D. Secretary (Revenue)

(D) Secretary (Revenue)
Explanation :-
The Cabinet inter alia took decisions for the following :-
Creation of the GST Council as per Article 279A of the amended Constitution;
Creation of the GST Council Secretariat, with its office at New Delhi;
Appointment of the Secretary (Revenue) as the Ex-officio Secretary to the GST Council;
Inclusion of the Chairperson, Central Board of Excise and Customs (CBEC), as a permanent invitee (non-voting) to all proceedings of the GST Council;
The Cabinet also decided to provide for adequate funds for meeting the recurring and non-recurring expenses of the GST Council Secretariat, the entire cost for which shall be borne by the Central Government. The GST Council Secretariat shall be manned by officers taken on deputation from both the Central and State Governments.
निम्नलिखित के लिए कैबिनेट अंतर निर्णय लिया गया: -
संशोधित संविधान के अनुच्छेद 279 ए के अनुसार जीएसटी परिषद का निर्माण;
नई दिल्ली में अपने कार्यालय के साथ जीएसटी परिषद सचिवालय का निर्माण;
जीएसटी परिषद के पदेन सचिव के रूप में सचिव (राजस्व) की नियुक्ति;
जीएसटी परिषद की सभी कार्यवाही के लिए स्थायी आमंत्रित (गैर-मतदान) के रूप में अध्यक्ष, केंद्रीय उत्पाद और सीमा शुल्क बोर्ड (सीबीईसी) को शामिल करना;
मंत्रिमंडल ने जीएसटी परिषद सचिवालय के आवर्ती और गैर-आवर्ती खर्चों को पूरा करने के लिए पर्याप्त धनराशि उपलब्ध कराने का भी निर्णय लिया, जिसका पूरा खर्च केंद्र सरकार द्वारा वहन किया जाएगा। जीएसटी परिषद सचिवालय को केंद्र और राज्य सरकारों दोनों से प्रतिनियुक्ति पर लिए गए अधिकारियों द्वारा नियंत्रित किया जाएगा।

52. Centre for Data Management and Analytics (CDMA) has been inaugurated at :- डेटा प्रबंधन और विश्लेषिकी केंद्र (सीडीएमए) का उद्घाटन यहां किया गया है: -
A. Nagpur
B. New Delhi
C. Bengaluru
D. Chennai

(B) New Delhi
Explanation :-
The CAG’s Centre for Data Management and Analytics (CDMA) is going to play a catalytic role to synthesize and integrate relevant data into auditing process.
The CDMA will help to enhance the efficiency and accuracy of evidence in the audit process.
CDMA aims to build up capacity in the Indian Audit and Accounts Department in Big Data Analytics to exploit the data rich environment in the Union and State Governments.
The Centre is to be equipped with latest analytic tools and infrastructure to assist and guide the Data Analytic Groups constituted in all audit offices of CAG across the country through setting up the data management protocols and steering the data analytic capacity of the department.
CAG of India is also a member of an international project on ‘Data Analytics’ under the Working Group on IT Audit of International Organization for Supreme Audit Institutions.
कैग का सेंटर फॉर डेटा मैनेजमेंट एंड एनालिटिक्स (सीडीएमए) संबंधित डेटा को ऑडिटिंग प्रक्रिया में संश्लेषित और एकीकृत करने के लिए एक उत्प्रेरक भूमिका निभाने जा रहा है।
सीडीएमए लेखापरीक्षा प्रक्रिया में साक्ष्य की दक्षता और सटीकता को बढ़ाने में मदद करेगा।
सीडीएमए का उद्देश्य केंद्र और राज्य सरकारों में डेटा समृद्ध वातावरण का दोहन करने के लिए बिग डेटा एनालिटिक्स में भारतीय लेखा परीक्षा और लेखा विभाग में क्षमता का निर्माण करना है।
केंद्र को डेटा प्रबंधन प्रोटोकॉल स्थापित करने और विभाग की डेटा विश्लेषणात्मक क्षमता को बढ़ाने के माध्यम से देश भर में CAG के सभी ऑडिट कार्यालयों में गठित डेटा एनालिटिक समूहों की सहायता और मार्गदर्शन करने के लिए नवीनतम विश्लेषणात्मक उपकरणों और बुनियादी ढांचे से लैस किया जाना है।
भारत का CAG सुप्रीम ऑडिट संस्थानों के लिए अंतर्राष्ट्रीय संगठन के आईटी ऑडिट पर कार्य समूह के तहत ‘डेटा एनालिटिक्स’ पर एक अंतर्राष्ट्रीय परियोजना का भी सदस्य है।

53. Which of the following are functions of NITI Aayog? निम्नलिखित में से कौन से NITI Aayog के कार्य हैं?
(1) Fostering cooperative federalism. सहकारी संघवाद को बढ़ावा देना।
(2) Platform for resolution of inter-sectoral and inter­ departmental issues. अंतर-क्षेत्रीय और अंतर विभागीय मुद्दों के समाधान के लिए मंच।
(3) Instrument to bring outside ideas into policy-making. बाहर के विचारों को नीति-निर्माण में लाने का साधन। 

Code :-
A. 1, 3
B. 1, 2
C. 2, 3
D. All of the above

(D) All of the above
Explanation :-
Apart from above NITI Aayog’s functions are :-
To maintain a state-of-the-art Resource Centre, be a repository of research on good governance and best practices in sustainable and equitable development as well as help their dissemination to stake-holders
To offer a platform for resolution of inter-sectoral and inter­ departmental issues in order to accelerate the implementation of the development agenda.
To create a knowledge, innovation and entrepreneurial support system through a collaborative community of national and international experts, practitioners and other partners.
To be the instrument through which ideas from outside are incorporated into policy-making.
उपरोक्त के अलावा NITI Aayog के कार्य हैं: -
अत्याधुनिक संसाधन केंद्र को बनाए रखने के लिए, सुशासन और सतत और समान विकास में सर्वोत्तम प्रथाओं के साथ-साथ हितधारकों के लिए उनके प्रसार में मदद करने के लिए अनुसंधान का एक भंडार हो
विकास के एजेंडा के कार्यान्वयन में तेजी लाने के लिए अंतर-क्षेत्रीय और अंतर विभागीय मुद्दों के समाधान के लिए एक मंच प्रदान करना।
राष्ट्रीय और अंतर्राष्ट्रीय विशेषज्ञों, चिकित्सकों और अन्य भागीदारों के एक सहयोगी समुदाय के माध्यम से एक ज्ञान, नवाचार और उद्यमशीलता सहायता प्रणाली बनाने के लिए।
वह साधन होना जिसके माध्यम से बाहर के विचारों को नीति-निर्माण में शामिल किया जाता है।

54. Which of the following are functions of NITI Aayog? निम्नलिखित में से कौन से NITI Aayog के कार्य हैं?
(1) To formulate credible plans at the village level.  ग्रामीण स्तर पर विश्वसनीय योजनाएँ बनाना।
(2) Partnerships with National and International Think Tanks. राष्ट्रीय और अंतर्राष्ट्रीय थिंक टैंक के साथ साझेदारी।
(3) To focus on technology upgradation and capacity building for for implementation of programmes and initiatives. कार्यक्रमों और पहलों के कार्यान्वयन के लिए प्रौद्योगिकी उन्नयन और क्षमता निर्माण पर ध्यान केंद्रित करना। 

Code :-
A. 1, 3
B. 1, 2
C. 2, 3
D. All of the above

(D) All of the above
Explanation :-
Apart from above NITI Aayog’s functions are :-
To develop mechanisms to formulate credible plans at the village level and aggregate these progressively at higher levels of government
To pay special attention to the sections of our society that may be at risk of not benefitting adequately from economic progress
To ensure, on areas that are specifically referred to it, that the interests of national security are incorporated in economic strategy and policy
Feedback for constant innovative improvements
To mainstream external ideas into Government policies, through collaboration with national and international experts;
To be the Government’s link to the outside world, outside experts and practitioners;
उपरोक्त के अलावा NITI Aayog के कार्य हैं: -
गाँव स्तर पर विश्वसनीय योजनाएँ बनाने के लिए तंत्र विकसित करना और सरकार के उच्च स्तरों पर इन्हें उत्तरोत्तर विकसित करना
हमारे समाज के उन वर्गों पर विशेष ध्यान देना जो आर्थिक प्रगति से पर्याप्त रूप से लाभान्वित नहीं होने के जोखिम में हो सकते हैं
यह सुनिश्चित करने के लिए, उन क्षेत्रों पर, जो विशेष रूप से इसके लिए संदर्भित हैं, कि आर्थिक रणनीति और नीति में राष्ट्रीय सुरक्षा के हितों को शामिल किया गया है
निरंतर नवीन सुधार के लिए प्रतिक्रिया
राष्ट्रीय और अंतर्राष्ट्रीय विशेषज्ञों के सहयोग से, सरकारी नीतियों में बाहरी विचारों को मुख्यधारा में लाना;
बाहर की दुनिया, बाहर के विशेषज्ञों और चिकित्सकों के लिए सरकार की कड़ी बनना;

55. Which of the following is/are true regarding Memorandum of Procedure? मेमोरेंडम ऑफ प्रोसीजर के संबंध में निम्नलिखित में से कौन-सा सही है / हैं?
(1) Evaluation of judgments delivered by a high court judge during the last three years and initiatives undertaken for improvement of judicial administration should be a yardstick of merit for promotion as chief justice of a high court.  पिछले तीन वर्षों के दौरान उच्च न्यायालय के न्यायाधीश द्वारा दिए गए निर्णयों का मूल्यांकन और न्यायिक प्रशासन में सुधार के लिए की गई पहल उच्च न्यायालय के मुख्य न्यायाधीश के रूप में पदोन्नति के लिए योग्यता का एक पैमाना होनी चाहिए।
(2) HC acting Chief Justice term should be limited to 3 months.  HC के कार्यवाहक मुख्य न्यायाधीश का कार्यकाल 3 महीने तक सीमित होना चाहिए।

Code :-
A. Only 1
B. Only 2
C. Both 1 and 2
D. Neither 1 nor 2

(B) Only 2
Explanation :-
Evaluation of judgements delivered by a high court judge during the last five years and initiatives undertaken for improvement of judicial administration should be a yardstick of merit for promotion as chief justice of a high court.
It seeks to introduce performance appraisal as a standard for appointing chief justices of high courts and Supreme Court judges.
The Union Law Minister should seek the recommendation of the incumbent CJI for appointment of his successor at least one month prior to his retirement.
A notice for vacancies of judges should be put up on the website of the high courts at the beginning of the year for appointments.
पिछले पांच वर्षों के दौरान एक उच्च न्यायालय के न्यायाधीश द्वारा वितरित निर्णयों का मूल्यांकन और न्यायिक प्रशासन के सुधार के लिए की गई पहल उच्च न्यायालय के मुख्य न्यायाधीश के रूप में पदोन्नति के लिए योग्यता का एक पैमाना होनी चाहिए।
यह उच्च न्यायालयों और सर्वोच्च न्यायालय के न्यायाधीशों के मुख्य न्यायाधीशों की नियुक्ति के लिए एक मानक के रूप में प्रदर्शन मूल्यांकन शुरू करना चाहता है।
केंद्रीय कानून मंत्री को अपनी सेवानिवृत्ति के कम से कम एक महीने पहले अपने उत्तराधिकारी की नियुक्ति के लिए सीजेआई की सिफारिश लेनी चाहिए।
नियुक्तियों के लिए उच्च न्यायालय की वेबसाइट पर न्यायाधीशों की रिक्तियों के लिए एक नोटिस लगाया जाना चाहिए।

56. Which of the following is/are true regarding Memorandum of Procedure? मेमोरेंडम ऑफ प्रोसीजर के संबंध में निम्नलिखित में से कौन-सा सही है / हैं?
(1) The MoP states that up to three judges in the Supreme Court need to be appointed from among the eminent members of the Bar and distinguished jurists. MoP कहता है कि सुप्रीम कोर्ट में तीन जजों को बार के प्रतिष्ठित सदस्यों और प्रतिष्ठित न्यायविदों के बीच से नियुक्त करने की आवश्यकता है।
(2) It proposes that for appointment of judges in the Supreme Court, the ‘prime criteria’ should be merit and not seniority. यह प्रस्ताव करता है कि सर्वोच्च न्यायालय में न्यायाधीशों की नियुक्ति के लिए, ’प्रधान मानदंड’ योग्यता होनी चाहिए न कि वरिष्ठता। 

Code :-
A. Only 1
B. Only 2
C. Both 1 and 2
D. Neither 1 nor 2

(A) Only 1
Explanation :-
It proposes that for appointment of judges in the Supreme Court, the “prime criteria” should be “seniority as chief justice/ judge of the high court”.
The MoP states that up to three judges in the Supreme Court need to be appointed from among the eminent members of the Bar and distinguished jurists with proven track record in their respective fields.
A permanent secretariat to be set up in the Supreme Court for maintaining records of high court judges, scheduling meetings of the SC Collegium, receiving recommendations as well as complaints in matters related to appointments.
यह प्रस्ताव करता है कि सर्वोच्च न्यायालय में न्यायाधीशों की नियुक्ति के लिए, "मुख्य मानदंड" "उच्च न्यायालय के मुख्य न्यायाधीश / न्यायाधीश के रूप में वरिष्ठता" होना चाहिए।
MoP में कहा गया है कि सुप्रीम कोर्ट में तीन न्यायाधीशों को बार के प्रतिष्ठित सदस्यों और अपने क्षेत्र में सिद्ध ट्रैक रिकॉर्ड के साथ प्रतिष्ठित न्यायविदों के बीच से नियुक्त करने की आवश्यकता है।
उच्च न्यायालय के न्यायाधीशों के रिकॉर्ड को बनाए रखने के लिए सुप्रीम कोर्ट में एक स्थायी सचिवालय स्थापित किया जाना है, अनुसूचित जाति कॉलेजियम की बैठकों का समय निर्धारण, सिफारिशों के साथ-साथ नियुक्तियों से संबंधित मामलों में शिकायतें प्राप्त करना।

57. Memorandum of Procedure has been in news in relation to? मेमोरेंडम ऑफ प्रोसीजर के संबंध में खबरों में रहा है?
A. Appointment of Judges
B. Revised Standards for International agreements
C. Amendments to procedure followed while conducting business of the Parliament
D. Simplification of process to obtain loan under Start-up India

(A) Appointment of Judges
Explanation :- 
A Group of Ministers (GoM) headed by External Affairs Minister have finalized the new Memorandum of Procedure (MoP) for appointment of Judges.
After quashing the proposed National Judicial Appointments Commission (NJAC), the Supreme Court’s Constitution Bench had asked the Centre to consult the CJI for drafting the new memorandum for appointments of judges to the higher judiciary.
For the first time, it has been asked to include “merit and integrity” as “prime criteria” for appointment of judges to the higher judiciary.
विदेश मंत्री की अगुवाई में मंत्रियों के एक समूह (जीओएम) ने न्यायाधीशों की नियुक्ति के लिए नए मेमोरंडम ऑफ प्रोसीजर (एमओपी) को अंतिम रूप दिया है।
प्रस्तावित राष्ट्रीय न्यायिक नियुक्ति आयोग (एनजेएसी) को रद्द करने के बाद, सर्वोच्च न्यायालय की संविधान पीठ ने केंद्र से उच्च न्यायपालिका के लिए न्यायाधीशों की नियुक्तियों के लिए नए ज्ञापन का मसौदा तैयार करने के लिए CJI से परामर्श करने के लिए कहा था।
पहली बार, उच्चतर न्यायपालिका में न्यायाधीशों की नियुक्ति के लिए "योग्यता और अखंडता" को "मुख्य मानदंड" के रूप में शामिल करने के लिए कहा गया है।

58. Which of the following is/are true regarding Jamia Millia Islamia? जामिया मिलिया इस्लामिया के बारे में निम्नलिखित में से कौन-सा सच है / हैं?
(1) Jamia Millia Islamia is another name for Aligarh Muslim University and is considered a part of AMU. जामिया मिलिया इस्लामिया अलीगढ़ मुस्लिम विश्वविद्यालय का दूसरा नाम है और इसे एएमयू का हिस्सा माना जाता है।
(2) It is easier to establish minority universities than minority educational institutions.अल्पसंख्यक शिक्षण संस्थानों की तुलना में अल्पसंख्यक विश्वविद्यालयों की स्थापना करना अधिक आसान है।

Code :- 
A. Only 1
B. Only 2
C. Both 1 and 2
D. Neither 1 nor 2

(D) Neither 1 nor 2
Explanation :- 
While a number of minority educational institutions exist, there are issues with regard to minority universities.
For incorporating any university, a statute is needed and thus it is done by the state.
Those against minority status for universities say that since, university is established by law and not minorities, it is not a minority institution.
But, those in support argue that establishment is different from incorporation and for incorporation of a university, law is needed irrespective of the fact that it was established by minorities.
Jamia Millia Islamia was founded in 1920 during Khilafat and Non-cooperation Movement.
It became a Central University through the Jamia Millia Islamia Act, 1988.
It was declared a minority in 2011.
जबकि कई अल्पसंख्यक शैक्षणिक संस्थान मौजूद हैं, अल्पसंख्यक विश्वविद्यालयों के संबंध में मुद्दे हैं।
किसी भी विश्वविद्यालय को शामिल करने के लिए एक क़ानून की आवश्यकता होती है और इस प्रकार यह राज्य द्वारा किया जाता है।
विश्वविद्यालयों के लिए अल्पसंख्यक दर्जे के खिलाफ लोगों का कहना है कि चूंकि, विश्वविद्यालय कानून द्वारा स्थापित किया गया है और अल्पसंख्यकों द्वारा नहीं, यह अल्पसंख्यक संस्थान नहीं है।
लेकिन, समर्थन करने वालों का तर्क है कि स्थापना निगमन से अलग है और एक विश्वविद्यालय के निगमन के लिए, इस तथ्य के बावजूद कानून की आवश्यकता है कि यह अल्पसंख्यकों द्वारा स्थापित किया गया था।
जामिया मिलिया इस्लामिया की स्थापना 1920 में खिलाफत और असहयोग आंदोलन के दौरान हुई थी।
यह जामिया मिलिया इस्लामिया अधिनियम, 1988 के माध्यम से एक केंद्रीय विश्वविद्यालय बन गया।
इसे 2011 में अल्पसंख्यक घोषित किया गया था।

59. Which article of the Indian Constitution deals with the Borrowings by State? भारतीय संविधान का कौन-सा अनुच्छेद राज्य द्वारा उधारों से संबंधित है?
A. 213
B. 296
C. 293
D. 282

(C) 293
Explanation :- 
Article 293 (3) - A State may not without the consent of the Government of India raise any loan.
In the spirit of Co-operative federalism and in order to bring-in the transparency and predictability in the Open Market Borrowings (OMBs) by the States, the Central Government has decided to simplify the consent mechanism for OMBs under Article 293 (3) of the Constitution.
Till now, the States were required to obtain quarterly consent from the Central Government for raising OMBs within the Net Borrowing Ceiling (NBC) fixed for each of the States as per the formula prescribed by the Fourteenth Finance Commission (FFC).
The simplified procedure will ensure that consent under Article 293(3) is issued only on three occasions during the year, one in the month of April for first nine months after fixation of borrowing ceilings, second in the month of December for the first two months of the fourth quarter and last in the month of March after the assessment of actual borrowings by the States.
अनुच्छेद 293 (3) - भारत सरकार की सहमति के बिना कोई राज्य कोई ऋण नहीं उठा सकता है।
सहकारी संघवाद की भावना में और राज्यों द्वारा खुले बाजार उधार (ओएमबी) में पारदर्शिता और भविष्यवाणी लाने के लिए, केंद्र सरकार ने अनुच्छेद 293 (3) के तहत ओएमबी के लिए सहमति तंत्र को आसान बनाने का फैसला किया है। संविधान।
अब तक, राज्यों को चौदहवें वित्त आयोग (FFC) द्वारा निर्धारित फार्मूले के अनुसार प्रत्येक राज्य के लिए निर्धारित नेट बोरिंग सीलिंग (NBC) के भीतर OMBs बढ़ाने के लिए केंद्र सरकार से त्रैमासिक सहमति प्राप्त करने की आवश्यकता थी।
सरलीकृत प्रक्रिया यह सुनिश्चित करेगी कि अनुच्छेद 293 (3) के तहत सहमति वर्ष के दौरान केवल तीन अवसरों पर जारी की जाती है, अप्रैल के महीने में एक, उधार लेने की छत के निर्धारण के बाद पहले नौ महीनों के लिए, दूसरे में दिसंबर के पहले दो महीनों के लिए। चौथी तिमाही और राज्यों द्वारा वास्तविक उधार के आकलन के बाद मार्च के महीने में अंतिम।

60. Which of the following is/are true? निम्नलिखित में से कौन-सा सत्य है / हैं?
(1) National Medical Commission (NMC) will replace MCI and UGC. राष्ट्रीय चिकित्सा आयोग (NMC) MCI और UGC का स्थान लेगा।
(2) NMC will have two boards under it. NMC के अंतर्गत दो बोर्ड होंगे।

Code :- 
A. Only 1
B. Only 2
C. Both 1 and 2
D. Neither 1 nor 2

(D) Neither 1 nor 2
Explanation :- 
A committee is set to recommend the scrapping of the Medical Council of India (MCI) and replacing it with the National Medical Commission (NMC).
Four Boards —Under Graduate Medical Board, Post Graduate Medical Board, Accreditation and Assessment Board and a board for registration of medical colleges as well monitoring of the ethics in the profession — will be set up under the Commission.
The committee is also debating if a universal exit exam should be introduced for medical students to make the medical education system merit-based.
The NMC will become the main regulatory body and will take over all roles and responsibilities of the MCI.
The main objective of the reform is to end the Inspector Raj in the medical education sector.
For allowing private medical colleges, the reformed system will strive to be more assessment-based, focussing on outcomes rather than relying on the inputs-based eligibility criteria in the present system.
मेडिकल काउंसिल ऑफ इंडिया (MCI) के स्क्रैपिंग की सिफारिश करने और इसे नेशनल मेडिकल कमीशन (NMC) से बदलने के लिए एक कमेटी गठित की गई है।
चार बोर्ड-ग्रेजुएट ग्रेजुएट मेडिकल बोर्ड, पोस्ट ग्रेजुएट मेडिकल बोर्ड, प्रत्यायन और मूल्यांकन बोर्ड और मेडिकल कॉलेजों के पंजीकरण के लिए एक बोर्ड के साथ-साथ पेशे में नैतिकता की निगरानी - आयोग के तहत स्थापित किया जाएगा।
समिति यह भी बहस कर रही है कि क्या चिकित्सा शिक्षा प्रणाली को मेरिट-आधारित बनाने के लिए मेडिकल छात्रों के लिए एक सार्वभौमिक निकास परीक्षा शुरू की जानी चाहिए।
NMC मुख्य नियामक संस्था बन जाएगी और MCI की सभी भूमिकाओं और जिम्मेदारियों को संभाल लेगी।
सुधार का मुख्य उद्देश्य चिकित्सा शिक्षा क्षेत्र में इंस्पेक्टर राज को समाप्त करना है।
निजी मेडिकल कॉलेजों की अनुमति देने के लिए, सुधार प्रणाली वर्तमान प्रणाली में इनपुट-आधारित पात्रता मानदंडों पर निर्भर होने के बजाय परिणामों पर अधिक ध्यान केंद्रित करने, अधिक मूल्यांकन-आधारित होने का प्रयास करेगी।

61. Which of the following are mentioned specifically in articles of the constitution? निम्नलिखित में से कौन से संविधान के लेखों में विशेष रूप से उल्लिखित हैं?
(1) Taxes levied and collected by the Union but assigned to the states. संघ द्वारा लगाया और वसूला जाने वाला कर लेकिन राज्यों को सौंपा जाता है।
(2) Taxes Levied and Collected by the Centre but Distributed between the Centre and the States. कर और केंद्र द्वारा एकत्र लेकिन केंद्र और राज्यों के बीच वितरित।
3) Taxes Levied and Collected and Retained by the States. करों और राज्यों द्वारा एकत्र और सेवानिवृत्त।  

Code :- 
A. 1, 3
B. 2, 3
C. 1, 2
D. All of the above

(C) 1, 2
Explanation :- 
268. Duties levied by the Union but collected and appropriated by the states.
268A. Service tax levied by Union and collected and appropriated by the Union and the states (will be omitted due to GST amendment)
269. Taxes levied and collected by the Union but assigned to the states
270. Taxes Levied and Collected by the Centre but Distributed between the Centre and the States
271. Surcharge on certain duties and taxes for purposes of the Union
272. Taxes which are levied and collected by the Union and may be distributed between the Union and the states (Repealed).
Taxes Levied and Collected and Retained by the States – it is not directly mentioned as a separate article. It is inferred from the list of 20 entries in the State List of Seventh Schedule.
268. संघ द्वारा लगाए गए कर्तव्य, लेकिन राज्यों द्वारा एकत्र और विनियोजित।
268A। संघ और राज्यों द्वारा एकत्रित और विनियोजित सेवा कर (जीएसटी संशोधन के कारण छोड़े जाएंगे)
269. संघ द्वारा लगाया और वसूला जाने वाला कर लेकिन राज्यों को सौंपा गया
270. केंद्र द्वारा लेवीड एंड कलेक्टेड लेकिन केंद्र और राज्यों के बीच वितरित
271. संघ के प्रयोजनों के लिए कुछ कर्तव्यों और करों पर अधिभार
272. कर जो संघ द्वारा वसूल किए जाते हैं और एकत्र किए जाते हैं और संघ और राज्यों (निरस्त) के बीच वितरित किए जा सकते हैं।
टैक्स लेवीड और स्टेट्स द्वारा एकत्रित और रिटायर्ड - यह सीधे एक अलग लेख के रूप में उल्लेख नहीं किया गया है। यह सातवीं अनुसूची की राज्य सूची में 20 प्रविष्टियों की सूची से अनुमानित है।

62. Which of the following subjects will be omitted from the Seventh Schedule? निम्नलिखित में से कौन सा विषय सातवीं अनुसूची से छोड़ा जाएगा?
(1) Entry 54 from State List
(2) Entry 55 from State List
(3) Entry 92 and 92C from Union List 

Code :- 
A. 1, 2
B. 2, 3
C. 1, 3
D. All of the above

(B) 2, 3
Explanation :- 
GST Constitutional Amendment Bill omits Entry 92 and 92C from the Union List and Entry 52 and 55 of the State List of the Seventh Schedule.
92 of Union list – Taxes on the sale or purchase of newspapers and on advertisements published therein.
92C of Union List – Taxes on services
52 of State List – Taxes on the entry of goods into a local area
55 of State List – Taxes on advertisements other than advertisements published in the newspapers and advertisements broadcast by radio or television
54 of state list is reworded but nor omitted.
जीएसटी संवैधानिक संशोधन विधेयक केंद्रीय सूची से प्रविष्टि 92 और 92 सी और सातवीं अनुसूची की राज्य सूची में प्रवेश 52 और 55 से बाहर है।
संघ सूची के 92 - समाचार पत्रों की बिक्री या खरीद पर और उसमें प्रकाशित विज्ञापनों पर कर।
संघ सूची का 92C - सेवाओं पर कर
52 राज्य सूची - स्थानीय क्षेत्र में माल के प्रवेश पर कर
55 राज्य सूची - समाचार पत्रों में प्रकाशित विज्ञापनों और रेडियो या टेलीविजन द्वारा प्रसारित विज्ञापनों के अलावा अन्य विज्ञापनों पर कर
54 राज्य सूची का पुनर्परीक्षण किया गया है लेकिन उसे छोड़ा नहीं गया है।

63. Which of the following article will be omitted due to the GST Constitutional Amendment Bill? GST संवैधानिक संशोधन विधेयक के कारण निम्नलिखित में से कौन सा लेख छोड़ा जाएगा?
A. 270A
B. 250A
C. 269A
D. 268A

(D) 268A
Explanation :- 
Article 268A of the Constitution, as inserted by the Constitution (Eighty-eighth Amendment) Act, 2003 shall be omitted.
The 88th Amendment has added a new Article 268-A dealing with service tax. It also added a new subject in the Union List – entry 92-C (taxes on services).
Service tax is levied by the centre but collected and appropriated by both the centre and the states.
संविधान का अनुच्छेद 268A, जैसा कि संविधान (अस्सी-आठवां संशोधन) अधिनियम, 2003 द्वारा डाला गया है।
88 वें संशोधन ने सेवा कर के साथ एक नया अनुच्छेद 268-ए जोड़ा है। इसने संघ सूची में एक नया विषय भी जोड़ा - प्रवेश 92-सी (सेवाओं पर कर)।
सेवा कर केंद्र द्वारा लगाया जाता है, लेकिन केंद्र और राज्यों दोनों द्वारा एकत्र और विनियोजित किया जाता है।

64. What shall constitute quorum of the GS Council meeting? जीएस काउंसिल की बैठक का कोरम क्या होगा?
A. One half of the total number of its members इसके सदस्यों की कुल संख्या का आधा
B. Two-thirds of the total number of its members इसके सदस्यों की कुल संख्या का दो-तिहाई
C. One Fourth of the total number of its members अपने सदस्यों की कुल संख्या का एक चौथाई 
D. One Third of the total number of its members अपने सदस्यों की कुल संख्या का एक तिहाई

(A) One half of the total number of its members इसके सदस्यों की कुल संख्या का आधा
Explanation :- 
One half of the total number of Members of the Goods and Services Tax Council shall constitute the quorum at its meetings.
Every decision of the Goods and Services Tax Council shall be taken at a meeting, by a majority of not less than three-fourths of the weighted votes of the Members present and voting, in accordance with the following principles, namely :-
(A) the vote of the Central Government shall have a weightage of one-third of the total votes cast, and
(B) the votes of all the State Governments taken together shall have a weightage of two-thirds of the total votes cast in that meeting.
माल और सेवा कर परिषद के कुल सदस्यों की संख्या का आधा हिस्सा अपनी बैठकों में कोरम का गठन करेगा।
गुड्स एंड सर्विसेज टैक्स काउंसिल का प्रत्येक निर्णय निम्नलिखित सिद्धांतों के अनुसार उपस्थित और मतदान करने वाले सदस्यों के भारित मतों के तीन-चौथाई से कम नहीं के बहुमत से एक बैठक में लिया जाएगा: -
(ए) केंद्र सरकार के वोट में डाले गए कुल वोटों का एक तिहाई और, का वेटेज होगा
(बी) सभी राज्य सरकारों के वोटों को एक साथ लेने पर उस बैठक में डाले गए कुल वोटों का दो-तिहाई का वेटेज होगा।

65. Which article spells formation of GST Council? किस लेख में GST परिषद के गठन का मंत्र है?
A. 270
B. 246A (2)
C. 269A (1)
D. 279A

(D) 279A
Explanation :- 
279A (1) – The president shall formulate GST Council within 60 days from the date of commencement of the Constitution Amendment Act.
279A (2) – Members of council.
279A (3) - The Members of the Goods and Services Tax Council shall, as soon as may be, choose one amongst themselves to be the Vice-Chairperson of the Council for such period as they may decide.
279A (1) - राष्ट्रपति संविधान संशोधन अधिनियम के प्रारंभ होने की तिथि से 60 दिनों के भीतर जीएसटी परिषद का गठन करेगा।
279A (2) - परिषद के सदस्य।
279A (3) - माल और सेवा कर परिषद के सदस्य, जितनी जल्दी हो सके, उतनी अवधि के लिए परिषद के उपाध्यक्ष बनने के लिए आपस में एक का चयन कर सकते हैं।

66. Which article mentions distribution of IGST among Centre and State? किस अनुच्छेद में केंद्र और राज्य के बीच IGST के वितरण का उल्लेख है?
A. 246A
B. 269A
C. 279A
D. 271

(B) 269A
Explanation :- 
Goods and services tax on supplies in the course of inter-State trade or commerce shall be levied and collected by the Government of India and such tax shall be apportioned between the Union and the States in the manner as may be provided by Parliament by law on the recommendations of the Goods and Services Tax Council.
For the purposes of this clause, supply of goods, or of services, or both in the course of import into the territory of India shall be deemed to be supply of goods, or of services, or both in the course of inter-State trade or commerce.
अंतरराज्यीय व्यापार या वाणिज्य के दौरान आपूर्ति पर माल और सेवा कर भारत सरकार द्वारा लगाया और एकत्र किया जाएगा और इस तरह का कर संघ और राज्यों के बीच उस तरीके से लागू किया जाएगा जिस तरह संसद द्वारा कानून द्वारा प्रदान किया जा सकता है माल और सेवा कर परिषद की सिफारिशें।
इस खंड के प्रयोजनों के लिए, माल की आपूर्ति, या सेवाओं, या भारत के क्षेत्र में आयात के दौरान दोनों को माल की आपूर्ति, या सेवाओं की आपूर्ति के रूप में माना जाएगा, या दोनों अंतर-राज्यीय व्यापार के दौरान या वाणिज्य।

67. Which of the following article grants powers to state legislatures for making laws with respect to goods and services tax imposed? निम्नलिखित में से किस अनुच्छेद में माल और सेवा कर के संबंध में कानून बनाने के लिए राज्य विधानसभाओं को शक्तियां प्रदान की गई हैं?
A. 246A
B. 269A
C. 270
D. 279A

(A) 246A
Explanation :- 
246A (1) - Parliament, and, subject to clause (2), the Legislature of every State, have power to make laws with respect to goods and services tax imposed by the Union or by such State.
246A (2) - Parliament has exclusive power to make laws with respect to goods and services tax where the supply of goods, or of services, or both takes place in the course of inter-State trade or commerce.
246A (1) - संसद, और, खंड (2) के अधीन, हर राज्य के विधानमंडल, संघ या ऐसे राज्य द्वारा लगाए गए माल और सेवा कर के संबंध में कानून बनाने की शक्ति रखते हैं।
246A (2) - संसद के पास माल और सेवा कर के संबंध में कानून बनाने की विशेष शक्ति है जहां वस्तुओं की आपूर्ति, या सेवाओं की, या दोनों अंतर-राज्यीय व्यापार या वाणिज्य के दौरान होती है।

68. Which of the following is/are true? निम्नलिखित में से कौन-सा सत्य है / हैं?
(1) No tax can be imposed on the consumption or sale of electricity by the states, which is consumed in the construction, maintenance or operation of any railway by the Centre or by the concerned railway company. राज्यों द्वारा बिजली की खपत या बिक्री पर कोई कर नहीं लगाया जा सकता है, जो कि केंद्र द्वारा या संबंधित रेलवे कंपनी द्वारा किसी भी रेलवे के निर्माण, रखरखाव या संचालन में उपयोग किया जाता है।
(2) A state legislature can impose taxes on the sale or purchase of goods (other than newspapers). एक राज्य विधायिका माल की बिक्री या खरीद (समाचार पत्रों के अलावा) पर कर लगा सकती है।

Code :- 
A. Only 1
B. Only 2
C. Both 1 and 2
D. Neither 1 nor 2

(C) Both 1 and 2
Explanation :- 
A state legislature can impose tax on the consumption or sale of electricity.
But, no tax can be imposed on the consumption or sale of electricity which is
(A) consumed by the Centre or sold to the Centre; or
(B) consumed in the construction, maintenance or operation of any railway by the Centre or by the concerned railway company or sold to the Centre or the railway company for the same purpose.
A state legislature can impose a tax in respect of any water or electricity stored, generated, consumed, distributed or sold by any authority established by Parliament for regulating or developing any inter-state river or river valley.
But, such a law, to be effective, should be reserved for the president’s consideration and receive his assent.
एक राज्य विधायिका बिजली की खपत या बिक्री पर कर लगा सकती है।
लेकिन, बिजली की खपत या बिक्री पर कोई कर नहीं लगाया जा सकता है
(ए) केंद्र द्वारा खपत या केंद्र को बेची गई; या
(बी) केंद्र द्वारा या संबंधित रेलवे कंपनी द्वारा किसी भी रेलवे के निर्माण, रखरखाव या संचालन में खपत या उसी उद्देश्य के लिए केंद्र या रेलवे कंपनी को बेच दिया।
एक राज्य विधायिका किसी भी अंतर-राज्यीय नदी या नदी घाटी को विनियमित करने या विकसित करने के लिए संसद द्वारा स्थापित किसी भी प्राधिकरण द्वारा संग्रहीत, उत्पादित, खपत, वितरित या बेची गई किसी भी पानी या बिजली के संबंध में एक कर लगा सकती है।
लेकिन, ऐसा कानून, प्रभावी होने के लिए, राष्ट्रपति के विचार के लिए आरक्षित होना चाहिए और उसकी स्वीकृति प्राप्त करना चाहिए।

69. The total amount of taxes levied by state like taxes on professions, trades, etc. should not exceed :- राज्य द्वारा लगाए गए करों की कुल राशि जैसे व्यवसायों, व्यवसायों, आदि पर करों से अधिक नहीं होनी चाहिए: -
A. Rs. 2500 annually
B. Rs. 5000 annually
C. Rs. 250 annually
D. Rs. 250 monthly

(A) Rs. 2500 annually रु। 2500 प्रतिवर्ष
Explanation :- 
A state legislature can impose taxes on professions, trades, callings and employments.
But, the total amount of such taxes payable by any person should not exceed Rs. 2,500 per annum.
Originally, this limit was only Rs 250 per annum. The 60th Amendment Act of 1988 raised it to Rs. 2,500 per annum.
एक राज्य विधायिका व्यवसायों, ट्रेडों, कॉलिंग और रोजगार पर कर लगा सकती है।
लेकिन, किसी भी व्यक्ति द्वारा देय ऐसे करों की कुल राशि रुपये से अधिक नहीं होनी चाहिए। 2,500 प्रति वर्ष।
मूल रूप से, यह सीमा केवल 250 रुपये प्रति वर्ष थी। 1988 के 60 वें संशोधन अधिनियम ने इसे बढ़ाकर रु। 2,500 प्रति वर्ष।

70. Which of the following is/are true? निम्नलिखित में से कौन-सा सत्य है / हैं?
(1) Both the State and Centre will have power to make laws on taxation of goods and services. राज्य और केंद्र दोनों के पास माल और सेवाओं के कराधान पर कानून बनाने की शक्ति होगी।
(2) Parliament’s law will not override a state law on GST. संसद का कानून GST पर राज्य के कानून को अधिरोहित नहीं करेगा।

Code :- 
A. Only 1
B. Only 2
C. Both 1 and 2
D. Neither 1 nor 2

(A) Only 1
Explanation :- 
Both Parliament, state Houses will have the power to make laws on the taxation of goods and services.
Parliament’s law will not override a state law on GST
Exclusive power to Centre to levy, collect GST in the course of interstate trade or commerce, or imports. This will be known as Integrated GST (IGST)
Central law will prescribe manner of sharing of IGST between Centre and states, based on GST Council’s views.
Alcoholic liquor for human consumption and Petroleum crude, high speed diesel, motor spirit (petrol), natural gas and aviation turbine fuel - excluded from GST purview until GST Council decides.
दोनों संसद, राज्य सदनों को माल और सेवाओं के कराधान पर कानून बनाने की शक्ति होगी।
संसद का कानून GST पर राज्य के कानून को अधिरोहित नहीं करेगा
केंद्र को लेवी, अंतरराज्यीय व्यापार या वाणिज्य, या आयात के दौरान जीएसटी एकत्र करने की विशेष शक्ति। इसे एकीकृत जीएसटी (IGST) के रूप में जाना जाएगा
केंद्रीय कानून GST परिषद के विचारों के आधार पर केंद्र और राज्यों के बीच IGST के बंटवारे के तरीके को निर्धारित करेगा।
मानव उपभोग और पेट्रोलियम क्रूड, उच्च गति डीजल, मोटर स्पिरिट (पेट्रोल), प्राकृतिक गैस और विमानन टरबाइन ईंधन के लिए मादक शराब - जीएसटी काउंसिल द्वारा जीएसटी के दायरे से बाहर किए जाने तक।

71. Who of the following will be the members of the GST Council? निम्नलिखित में से कौन जीएसटी परिषद का सदस्य होगा?
(1) Union Finance Minister  केंद्रीय वित्त मंत्री
(2) Union Minister of State in charge of Revenue or Finance केंद्रीय राज्य मंत्री राजस्व या वित्त के प्रभारी
(3) Chief Ministers of States  राज्यों के मुख्यमंत्री


Code :- 
A. 1, 3
B. 1, 2
C. 2, 3
D. All of the above

(B) 1, 2
Explanation :- 
The composition of the GST Council includes :-
The Union Finance Minister (as Chairman),
The Union Minister of State in charge of Revenue or Finance, and
The Minister in charge of Finance or Taxation or any other Minister, nominated by each state government.
The decisions of the GST Council will be made by three-fourth majority of the votes cast. The centre shall have one-third of the votes cast, and the states together shall have two-third of the votes cast. Mechanism for resolving disputes arising out of its recommendations may be decided by the Council itself.
केंद्रीय वित्त मंत्री (अध्यक्ष के रूप में),
केंद्रीय राज्य मंत्री राजस्व या वित्त के प्रभारी, और
वित्त या कराधान या किसी अन्य मंत्री के प्रभारी मंत्री, प्रत्येक राज्य सरकार द्वारा नामित।
जीएसटी परिषद के निर्णय वोटों के तीन-चौथाई बहुमत से किए जाएंगे। केंद्र के पास डाले गए मतों का एक तिहाई हिस्सा होगा, और राज्यों के पास एक तिहाई वोट होंगे। इसकी सिफारिशों से उत्पन्न विवादों को हल करने का तंत्र परिषद द्वारा ही तय किया जा सकता है।

72.  Which of the following is/are true regarding recent GST Bill? निम्नलिखित में से कौन-सा हाल के जीएसटी बिल के बारे में सही है / हैं?
(1) Recent Goods and Services Tax (GST) Bill is a money bill.  हालिया गुड्स एंड सर्विस टैक्स (GST) बिल एक मनी बिल है।
(2) Being a money bill, Rajya Sabha cannot move amendments to the recently passed GST Bill. धन विधेयक होने के कारण, राज्यसभा हाल ही में पारित जीएसटी विधेयक में संशोधन नहीं कर सकती है।
Code :- 
A. Only 1
B. Only 2
C. Both 1 and 2
D. Neither 1 nor 2

(D) Neither 1 nor 2
Explanation :- 
GST Bill is a constitutional Amendment Bill. Rajya Sabha can make amendments to the bill.
It is not a money bill as it makes changes to the constitution for altering the way taxes are levied, collected and divided among the centre and the state.
The Bill amends the Constitution to give concurrent powers to Parliament and state legislatures to levy a Goods and Services tax (GST).
This implies that the centre will levy a central GST (CGST), while states will be permitted to levy a state GST (SGST).
For goods and services that pass through several states, or imports, the centre will levy another tax, the Integrated GST (IGST).
जीएसटी विधेयक एक संवैधानिक संशोधन विधेयक है। राज्यसभा बिल में संशोधन कर सकती है।
यह एक धन विधेयक नहीं है क्योंकि यह केंद्र और राज्य के बीच कर, एकत्र और विभाजित किए गए तरीके को बदलने के लिए संविधान में बदलाव करता है।
माल और सेवा कर (जीएसटी) लगाने के लिए विधेयक संसद और राज्य विधानसभाओं को समवर्ती अधिकार देने के लिए संविधान में संशोधन करता है।
इसका मतलब है कि केंद्र केंद्रीय जीएसटी (सीजीएसटी) लगाएगा, जबकि राज्यों को राज्य जीएसटी (एसजीएसटी) लगाने की अनुमति होगी।
कई राज्यों या आयातों से गुजरने वाली वस्तुओं और सेवाओं के लिए, केंद्र एक और कर, एकीकृत जीएसटी (IGST) लगाएगा।

73. Which of the following is the most important for amendments to a bill? किसी विधेयक में संशोधन के लिए निम्नलिखित में से कौन सबसे महत्वपूर्ण है?
A. Approval by Union Cabinet
B. Approval by President
C. Approval of the Council of Ministers
D. Approval of the PM and the Council of Ministers

(A) Approval by Union Cabinet केंद्रीय मंत्रिमंडल द्वारा अनुमोदन
Explanation :- 
Amendments to Bills can be moved under three conditions -

one, when cleared by the Union Cabinet;
second, the Prime Minister invokes Rule 12 of the transaction of business and takes a suo motu decision, and a post facto approval is then sought from the Union Cabinet;
Third, an empowered group of ministers meet and clear it, again with post facto approval of the Cabinet.
In all these cases, approval by union cabinet is needed be it before or after.
बिल में संशोधन को तीन शर्तों के तहत स्थानांतरित किया जा सकता है -

एक, जब केंद्रीय मंत्रिमंडल द्वारा मंजूरी दे दी गई थी;
दूसरा, प्रधान मंत्री व्यापार के लेन-देन के नियम 12 को आमंत्रित करता है और एक स्व-प्रेरक निर्णय लेता है, और उसके बाद केंद्रीय मंत्रिमंडल से एक वास्तविक स्वीकृति प्राप्त की जाती है;
तीसरा, मंत्रियों का एक अधिकार प्राप्त समूह इसे पूरा करता है और इसे मंजूरी देता है, फिर से मंत्रिमंडल की वास्तविक स्वीकृति के साथ।
इन सभी मामलों में, केंद्रीय मंत्रिमंडल द्वारा अनुमोदन की आवश्यकता है, यह पहले या बाद में होना चाहिए।

74. Which of the following is/are true? निम्नलिखित में से कौन-सा सत्य है / हैं?
(1) Industry is a Concurrent List subject. उद्योग एक समवर्ती सूची विषय है।
(2) However, Central Government can make laws only for Defence industries and Industries, the control of which by the Union is in the public interest. हालाँकि, केंद्र सरकार केवल रक्षा उद्योगों और उद्योगों के लिए कानून बना सकती है, जिसका नियंत्रण संघ द्वारा जनता के हित में है।

Code :- 
A. Only 1
B. Only 2
C. Both 1 and 2
D. Neither 1 nor 2

(B) Only 2
Explanation :- 
Industry is a State subject.
The role of Department of Heavy Industry (DHI) is confined to the administration of Central Public Sector Enterprises (CPSEs) and Sectors under its administrative control.
Union List includes Defence Industries and Industries, the control of which by the Union is in the public interest.
उद्योग एक राज्य का विषय है।
भारी उद्योग विभाग (DHI) की भूमिका केंद्रीय सार्वजनिक क्षेत्र के उद्यमों (CPSE) और क्षेत्रों के प्रशासन के प्रशासनिक नियंत्रण में सीमित है।
संघ सूची में रक्षा उद्योग और उद्योग शामिल हैं, जिनका नियंत्रण संघ द्वारा जनता के हित में है।

75. Which of the following made suggestion that the centre should restrict itself to only areas of defence, foreign affairs, communications, and currency/economics? निम्नलिखित में से किसने सुझाव दिया कि केंद्र को केवल रक्षा, विदेशी मामलों, संचार और मुद्रा / अर्थशास्त्र के क्षेत्रों तक ही सीमित रखना चाहिए?
(1) Rajamannar Committee
(2) Anandpur Sahib Resolution
(3) West Bengal Memorandum 

Code :- 
A. 1, 3
B. 1, 2
C. 2, 3
D. All of the above

(C) 2, 3
Explanation :- 
Anandpur Sahib Resolution and West Bengal Memorandum demanded that the Centre’s jurisdiction should be restricted only to defence, foreign affairs, communications, and currency and the entire residuary powers should be vested in the states.
West Bengal Memorandum was in 1977.
Anandpur Sahib Resolution was in 1973.
Rajamannar Committiee appointed by Tamil Nadu Government in 1969, proposed in 1971 that the provision that the state ministry holds office during the pleasure of the governor should be omitted; Certain subjects of the Union List and the Concurrent List should be transferred to the State List; the residuary powers should be allocated to the states; and All-India services (IAS, IPS and IFS) should be abolished.
आनंदपुर साहिब रिज़ॉल्यूशन और पश्चिम बंगाल मेमोरेंडम ने मांग की कि केंद्र के अधिकार क्षेत्र को केवल रक्षा, विदेशी मामलों, संचार और मुद्रा तक सीमित रखा जाना चाहिए और राज्यों में संपूर्ण अवशिष्ट शक्तियों को निहित किया जाना चाहिए।
1977 में पश्चिम बंगाल मेमोरेंडम था।
आनंदपुर साहिब संकल्प 1973 में था।
1969 में तमिलनाडु सरकार द्वारा नियुक्त राजमन्नार कमिटि ने 1971 में प्रस्ताव दिया कि राज्यपाल के आनंद के दौरान राज्य मंत्रालय के पास जो प्रावधान है, उसे छोड़ दिया जाना चाहिए; संघ सूची और समवर्ती सूची के कुछ विषयों को राज्य सूची में स्थानांतरित किया जाना चाहिए; अवशिष्ट शक्तियों को राज्यों को आवंटित किया जाना चाहिए; और अखिल भारतीय सेवाओं (IAS, IPS और IFS) को समाप्त कर दिया जाना चाहिए।

76.  Which of the following are issues which created tensions and conflicts between the Centre and states? निम्नलिखित में से कौन से मुद्दे हैं जिन्होंने केंद्र और राज्यों के बीच तनाव और टकराव पैदा किया है?
(1) Encroachment by the Centre on the State List राज्य सूची में केंद्र द्वारा अतिक्रमण
(2) Use of electronic media for political purposes राजनीतिक उद्देश्यों के लिए इलेक्ट्रॉनिक मीडिया का उपयोग
(3) Mode of appointment and dismissal of governor राज्यपाल की नियुक्ति और बर्खास्तगी की विधि

Code :- 
A. 2, 3
B. 1, 2
C. 1, 3
D. All of the above

(D) All of the above
Explanation :-
The issues which created tensions and conflicts between the Centre and states are :- 
(1) Mode of appointment and dismissal of governor;
(2) Discriminatory and partisan role of governors;
(3) Imposition of President’s Rule for partisan interests;
(4) Deployment of Central forces in the states to maintain law and order;
(5) Reservation of state bills for the consideration of the President;
(6) Discrimination in financial allocations to the states;
(7) Role of Planning Commission in approving state projects;
(8) Management of All-India Services (IAS, IPS, and IFS);
(9) Use of electronic media for political purposes;
(10) Appointment of enquiry commissions against the chief ministers;
(11) Sharing of finances (between Centre and states); and
(12) Encroachment by the Centre on the State List.
केंद्र और राज्यों के बीच तनाव और टकराव पैदा करने वाले मुद्दे हैं: -
(1) राज्यपाल की नियुक्ति और बर्खास्तगी की विधि;
(2) राज्यपालों की भेदभावपूर्ण और पक्षपातपूर्ण भूमिका;
(3) पक्षपातपूर्ण हितों के लिए राष्ट्रपति शासन का प्रभाव;
(4) कानून और व्यवस्था बनाए रखने के लिए राज्यों में केंद्रीय बलों की तैनाती;
(5) राष्ट्रपति के विचार के लिए राज्य बिलों का आरक्षण;
(6) राज्यों को वित्तीय आवंटन में भेदभाव;
(7) राज्य परियोजनाओं को मंजूरी देने में योजना आयोग की भूमिका;
(8) अखिल भारतीय सेवाओं (IAS, IPS, और IFS) का प्रबंधन;
(9) राजनीतिक उद्देश्यों के लिए इलेक्ट्रॉनिक मीडिया का उपयोग;
(10) मुख्यमंत्रियों के खिलाफ जाँच आयोगों की नियुक्ति;
(11) वित्त का बंटवारा (केंद्र और राज्यों के बीच); तथा
(12) राज्य सूची में केंद्र द्वारा अतिक्रमण।

77. Which of the following is/are true regarding recommendations of the Punchhi commission? पंची आयोग की सिफारिशों के बारे में निम्नलिखित में से कौन-सा सच है / हैं?
(1) Finance Commission division in the Ministry of Finance should be converted into a full-fledged department, serving as the permanent secretariat for the Finance Commissions.  वित्त मंत्रालय में वित्त आयोग के विभाजन को एक पूर्ण विभाग में परिवर्तित किया जाना चाहिए, जो वित्त आयोगों के लिए स्थायी सचिवालय के रूप में कार्य करेगा।
(2) The Governor should have the right to sanction for prosecution of a state minister against the advice of the Council of Ministers in any case to keep the government corruption free. राज्यपाल को किसी भी मामले में सरकार को भ्रष्टाचार मुक्त रखने के लिए मंत्रिपरिषद की सलाह के विरुद्ध राज्य मंत्री के खिलाफ मुकदमा चलाने की मंजूरी का अधिकार होना चाहिए।

Code :-
A. Only 1
B. Only 2
C. Both 1 and 2
D. Neither 1 nor 2

(A) Only 1
Explanation :-
Some important recommendations are  :-

There should be much better coordination between the Finance Commission and the Planning Commission. The synchronisation of the periods covered by the Finance Commission and the Five-Year Plan will considerably improve such coordination.
Finance Commission division in the Ministry of Finance should be converted into a full-fledged department, serving as the permanent secretariat for the Finance Commissions.
The Governor should have the right to sanction for prosecution of a state minister against the advice of the Council of Ministers, if the Cabinet decision appears to the Governor to be motivated by bias in the face of overwhelming material.
कुछ महत्वपूर्ण सिफारिशें हैं: -

वित्त आयोग और योजना आयोग के बीच बेहतर समन्वय होना चाहिए। वित्त आयोग और पंचवर्षीय योजना द्वारा कवर की गई समयावधि का समन्वय इस तरह के समन्वय में काफी सुधार करेगा।
वित्त मंत्रालय में वित्त आयोग के विभाजन को एक पूर्ण विभाग में परिवर्तित किया जाना चाहिए, जो वित्त आयोगों के लिए स्थायी सचिवालय के रूप में कार्य करेगा।
राज्यपाल को मंत्रिपरिषद की सलाह के खिलाफ राज्य मंत्री के खिलाफ मुकदमा चलाने की मंजूरी देने का अधिकार होना चाहिए, अगर मंत्रिमंडल का निर्णय राज्यपाल को भारी सामग्री के पक्ष में पूर्वाग्रह से प्रेरित प्रतीत होता है।

78. Which of the following is/are true regarding recommendations of the Punchhi commission? पंची आयोग की सिफारिशों के बारे में निम्नलिखित में से कौन-सा सच है / हैं?
(1) Setting up of an Inter-State Trade and Commerce Commission under Article 307. अनुच्छेद ३० of के तहत एक अंतर-राज्यीय व्यापार और वाणिज्य आयोग की स्थापना।
(2) It rejects the idea of localised emergency. यह स्थानीयकृत आपातकाल के विचार को खारिज करता है।

Code :-
A. Only 1
B. Only 2
C. Both 1 and 2
D. Neither 1 nor 2

(A) Only 1
Explanation :-
Steps should be taken for the setting up of an Inter-State Trade and Commerce Commission under Article 307 read with Entry 42 of List-I. This Commission should be vested with both advisory and executive roles with decision making powers.
As a Constitutional body, the decisions of the Commission should be final and binding on all states as well as the Union of India.
Any party aggrieved with the decision of the Commission may prefer an appeal to the Supreme Court.
It provides the framework for “localised emergency” that would ensure that the state government can continue to function and the Assembly would not have to be dissolved while providing a mechanism to let the Central Government respond to the issue specifically and locally.
The imposition of local emergency is fully justified under the mandate of Article 355 read with Entry 2A of List I and Entry 1 of List II of the Seventh Schedule.
अनुच्छेद -30 की सूची 42 के साथ पढ़े गए अनुच्छेद 307 के तहत अंतर-राज्य व्यापार और वाणिज्य आयोग की स्थापना के लिए कदम उठाए जाने चाहिए। इस आयोग को निर्णय लेने की शक्तियों के साथ सलाहकार और कार्यकारी दोनों भूमिकाओं के साथ निहित होना चाहिए।
संवैधानिक निकाय के रूप में, आयोग के निर्णय अंतिम और सभी राज्यों के साथ-साथ भारत संघ पर भी बाध्यकारी होने चाहिए।
आयोग के निर्णय से नाराज कोई भी दल सर्वोच्च न्यायालय में अपील करना पसंद कर सकता है।
यह "स्थानीय आपातकाल" के लिए रूपरेखा प्रदान करता है जो यह सुनिश्चित करेगा कि राज्य सरकार कार्य करना जारी रख सकती है और केंद्र सरकार को विशेष रूप से और स्थानीय रूप से इस मुद्दे पर प्रतिक्रिया देने के लिए एक तंत्र प्रदान करते समय विधानसभा को भंग नहीं करना होगा।
सातवीं अनुसूची की सूची II की प्रविष्टि 2A और प्रविष्टि 1 की प्रविष्टि 1 के साथ पढ़े गए अनुच्छेद 355 के जनादेश के तहत स्थानीय आपातकाल लगाने को पूरी तरह से उचित ठहराया गया है।

79.  Which of the following is/are true regarding recommendations of the Punchhi commission? पंची आयोग की सिफारिशों के बारे में निम्नलिखित में से कौन-सा सच है / हैं?
(1) The scope of devolution of powers to local bodies to act as institutions of self-government should be constitutionally defined through appropriate amendments. स्वशासन की संस्थाओं के रूप में कार्य करने के लिए स्थानीय निकायों को शक्तियों के विचलन का दायरा संवैधानिक रूप से उपयुक्त संशोधनों के माध्यम से परिभाषित किया जाना चाहिए।
(2) Equality of seats to states in the Rajya Sabha. राज्य सभा में राज्यों को सीटों की समानता।

Code :-
A. Only 1
B. Only 2
C. Both 1 and 2
D. Neither 1 nor 2

(C) Both 1 and 2
Explanation :-
Some important recommendations are :-
A balance of power between states inter se is desirable and this is possible by equality of representation in the Rajya Sabha. This requires amendment of the relevant provisions to give equality of seats to states in the Rajya Sabha, irrespective of their population size.
The scope of devolution of powers to local bodies to act as institutions of self-government should be constitutionally defined through appropriate amendments.
Factors inhibiting the composition and functioning of the Second Chamber as a representative forum of states should be removed or modified even if it requires amendment of the Constitutional provisions. In fact, Rajya Sabha offers immense potential to negotiate acceptable solutions to the friction points which emerge between Centre and states in fiscal, legislative and administrative relations.
कुछ महत्वपूर्ण सिफारिशें हैं: -
अंतर राज्य के बीच शक्ति का संतुलन वांछनीय है और राज्य सभा में प्रतिनिधित्व की समानता से यह संभव है। इसके लिए राज्यसभा में राज्यों को समानता देने के लिए संबंधित प्रावधानों में संशोधन की आवश्यकता है, चाहे उनकी जनसंख्या का आकार कुछ भी हो।
स्वशासन की संस्थाओं के रूप में कार्य करने के लिए स्थानीय निकायों को शक्तियों के विचलन का दायरा संवैधानिक रूप से उपयुक्त संशोधनों के माध्यम से परिभाषित किया जाना चाहिए।
राज्यों के प्रतिनिधि फोरम के रूप में दूसरे चैंबर की संरचना और कामकाज को बाधित करने वाले कारकों को हटा दिया जाना चाहिए या संशोधित किया जाना चाहिए, भले ही इसके लिए संवैधानिक प्रावधानों में संशोधन की आवश्यकता हो। वास्तव में, राज्य सभा घर्षण बिंदुओं के लिए स्वीकार्य समाधान पर बातचीत करने की अपार क्षमता प्रदान करती है जो केंद्र और राज्यों के बीच राजकोषीय, विधायी और प्रशासनिक संबंधों में उभरती है।

80. Which of the following is/are true regarding recommendations of the Punchhi commission? पंची आयोग की सिफारिशों के बारे में निम्नलिखित में से कौन सा सच है / हैं?
(1) On the question of dismissal of a Chief Minister, the Governor should invariably insist on the Chief Minister proving his majority on the floor of the House for which he should prescribe a time limit. एक मुख्यमंत्री की बर्खास्तगी के सवाल पर, राज्यपाल को हमेशा मुख्यमंत्री पर जोर देना चाहिए कि वे सदन के पटल पर अपना बहुमत साबित करें, जिसके लिए उन्हें एक समय सीमा निर्धारित करनी चाहिए।
(2) Role of Governors should not be confined to the Constitutional provisions only. राज्यपालों की भूमिका केवल संवैधानिक प्रावधानों तक ही सीमित नहीं होनी चाहिए।

Code :-
A. Only 1
B. Only 2
C. Both 1 and 2
D. Neither 1 nor 2

(A) Only 1
Explanation :-
On the question of dismissal of a Chief Minister, the Governor should invariably insist on the Chief Minister proving his majority on the floor of the House for which he should prescribe a time limit.
The convention of Governors acting as Chancellors of Universities and holding other statutory positions should be done away with. His role should be confined to the Constitutional provisions only.
The Governor should have the right to sanction for prosecution of a state minister against the advice of the Council of Ministers, if the Cabinet decision appears to the Governor to be motivated by bias in the face of overwhelming material.
एक मुख्यमंत्री की बर्खास्तगी के सवाल पर, राज्यपाल को हमेशा मुख्यमंत्री पर जोर देना चाहिए कि वे सदन के पटल पर अपना बहुमत साबित करें, जिसके लिए उन्हें एक समय सीमा निर्धारित करनी चाहिए।
विश्वविद्यालयों के कुलाधिपति के रूप में कार्य करने वाले राज्यपालों के सम्मेलन और अन्य वैधानिक पदों को रखने के साथ दूर किया जाना चाहिए। उसकी भूमिका केवल संवैधानिक प्रावधानों तक ही सीमित होनी चाहिए।
राज्यपाल को मंत्रिपरिषद की सलाह के खिलाफ राज्य मंत्री के खिलाफ मुकदमा चलाने की मंजूरी देने का अधिकार होना चाहिए, अगर मंत्रिमंडल का निर्णय राज्यपाल को भारी सामग्री के पक्ष में पूर्वाग्रह से प्रेरित प्रतीत होता है।

81. Which of the following is/are true regarding recommendations of the Punchhi commission? पंची आयोग की सिफारिशों के बारे में निम्नलिखित में से कौन-सा सच है / हैं?
(1) A time period can be made applicable for the President to decide on assenting or withholding assent to a state bill reserved for consideration of the President. राष्ट्रपति द्वारा राष्ट्रपति के विचार के लिए आरक्षित किसी राज्य के बिल पर सहमति या रोक लगाने का निर्णय लेने के लिए एक समय अवधि लागू की जा सकती है।
(2) While selecting Governors, the Central Government should adopt the following strict guidelines as recommended in the Sarkaria Commission report. राज्यपालों का चयन करते समय, केंद्र सरकार को निम्नलिखित सख्त दिशानिर्देशों को अपनाना चाहिए जैसा कि सरकारिया आयोग की रिपोर्ट में सुझाया गया है।

Code :-
A. Only 1
B. Only 2
C. Both 1 and 2
D. Neither 1 nor 2

(C) Both 1 and 2
Explanation :-
Some important recommendations are :-
The period of six months prescribed in Article 201 for State Legislature to act when the bill is returned by the President can be made applicable for the President also to decide on assenting or withholding assent to a state bill reserved for consideration of the President.
While selecting Governors, the Central Government should adopt the following strict guidelines as recommended in the Sarkaria Commission report and follow its mandate in letter and spirit :
(i) He should be eminent in some walk of life
(ii) He should be a person from outside the state
(iii) He should be a detached figure and not too intimately connected with the local politics of the states
(iv) He should be a person who has not taken too great a part in politics generally and particularly in the recent past
कुछ महत्वपूर्ण सिफारिशें हैं: -
राष्ट्रपति द्वारा विधेयक वापस किए जाने पर राज्य विधानमंडल के लिए अनुच्छेद 201 में निर्धारित छह महीने की अवधि राष्ट्रपति के विचार के लिए आरक्षित राज्य के बिल पर सहमति या रोक लगाने के निर्णय के लिए राष्ट्रपति के लिए भी लागू की जा सकती है।
राज्यपालों का चयन करते समय, केंद्र सरकार को निम्नलिखित सख्त दिशा निर्देशों को अपनाना चाहिए जैसा कि सरकारिया आयोग की रिपोर्ट में सिफारिश की गई है और पत्र और ज्ञान में इसके जनादेश का पालन करें:
(i) उसे जीवन के कुछ पड़ावों में प्रख्यात होना चाहिए
(ii) उसे राज्य के बाहर का व्यक्ति होना चाहिए
(iii) उसे एक अलग व्यक्ति होना चाहिए और राज्यों की स्थानीय राजनीति से बहुत आत्मीयता से नहीं जुड़ा होना चाहिए
(iv) वह ऐसा व्यक्ति होना चाहिए जिसने आम तौर पर और विशेष रूप से हाल के दिनों में राजनीति में बहुत बड़ा हिस्सा नहीं लिया हो

82. Which of the following is/are true? निम्नलिखित में से कौन-सा सत्य है / हैं?
(1) There have been only 10 Inter-State Council meetings. केवल 10 अंतर-राज्य परिषद की बैठकें हुई हैं।
(2) The 10th meeting focussed on Disaster Management (Preparedness of States to Cope With Disaster). 10 वीं बैठक आपदा प्रबंधन (राज्यों की आपदा से निपटने की तैयारी) पर केंद्रित है।

Code :-
A. Only 1
B. Only 2
C. Both 1 and 2
D. Neither 1 nor 2

(A) Only 1
Explanation :-
The Tenth Meeting of the Inter-State Council held on 9.12.2006 discussed Atrocities on Scheduled Castes and Scheduled Tribes and status of implementation of the Scheduled Castes/Scheduled Tribes (Prevention of Atrocities) Act, 1989. This year 11th meeting took place. It focussed on following points :-

Consideration of the recommendations of the Punchhi Commission on Centre-State relations,
Use of Aadhaar as an identifier and use of Direct Benefit Transfer (DBT) for providing subsidies, benefits and Public services,
Improving quality of school education with focus on improving learning outcomes, incentivising better performance, and Internal security
9.12.2006 को आयोजित अंतर-राज्य परिषद की दसवीं बैठक में अनुसूचित जाति और अनुसूचित जनजाति पर अत्याचार और अनुसूचित जाति / अनुसूचित जनजाति (अत्याचार निवारण) अधिनियम, 1989 के कार्यान्वयन की स्थिति पर चर्चा हुई। इस वर्ष 11 वीं बैठक हुई। यह निम्नलिखित बिंदुओं पर केंद्रित है: -

केंद्र-राज्य संबंधों पर पंची आयोग की सिफारिशों पर विचार,
एक पहचानकर्ता के रूप में आधार का उपयोग और सब्सिडी, लाभ और सार्वजनिक सेवाएं प्रदान करने के लिए प्रत्यक्ष लाभ हस्तांतरण (डीबीटी) का उपयोग,
सीखने के परिणामों में सुधार, बेहतर प्रदर्शन को प्रोत्साहित करने और आंतरिक सुरक्षा पर ध्यान देने के साथ स्कूली शिक्षा की गुणवत्ता में सुधार करना

83. Which of the following are indicators of Clean Village? निम्न में से कौन स्वच्छ ग्राम के संकेतक हैं?
(1) Open Defecation Free खुले में शौच मुक्त
(2) No visual pollution कोई दृश्य प्रदूषण नहीं
(3) Visual Cleanliness दृश्य स्वच्छता

Code :-
A. 1, 2
B. 1
C. 1, 3
D. All of the above

(C) 1, 3
Explanation :-
Village Swachhta Index (VSI) which will be the national metric to measure cleanliness of Indian villages in objective terms. Clean village is broadly defined as a village which :-

is Open Defecation Free.
has effective Solid and Liquid Waste Management.
has Visual Cleanliness.
ग्राम स्वच्छ सूचकांक (VSI) जो उद्देश्यपूर्ण दृष्टि से भारतीय गांवों की स्वच्छता को मापने के लिए राष्ट्रीय मीट्रिक होगा। स्वच्छ गाँव को मोटे तौर पर एक गाँव के रूप में परिभाषित किया गया है: -

खुले में शौच मुक्त है।
प्रभावी ठोस और तरल अपशिष्ट प्रबंधन है।
दृश्य सफाई है।

84. Which state is poised to become the second Open Defecation free state? दूसरा राज्य खुले में शौच मुक्त राज्य बनने के लिए किस राज्य की ओर अग्रसर है?
A. Sikkim
B. Kerala
C. Uttarakhand
D. Himachal Pradesh

(B) Kerala
Explanation :-
Kerala has taken up the Open Defecation Free (ODF) challenge and is aiming to become India’s second ODF State (after Sikkim) by November 1, 2016.
Kerala has adopted the slogan “My Waste is My Responsibility” as an important means to create public awareness.
Kerala has the one of the highest level of rural consumption, which means that it is one of the largest generators of rural waste.
केरल ने खुले में शौच मुक्त (ओडीएफ) चुनौती ली है और 1 नवंबर, 2016 तक भारत का दूसरा ओडीएफ राज्य (सिक्किम के बाद) बनने का लक्ष्य है।
केरल ने "जागरुकता मेरी ज़िम्मेदारी है" नारे को जन जागरूकता पैदा करने के लिए एक महत्वपूर्ण साधन के रूप में अपनाया है।
केरल में ग्रामीण खपत का उच्चतम स्तर है, जिसका अर्थ है कि यह ग्रामीण कचरे के सबसे बड़े जनरेटर में से एक है।

85. Who of the following can declare application of AFSPA in a disturbed area? निम्नलिखित में से कौन अशांत क्षेत्र में AFSPA के आवेदन की घोषणा कर सकता है?
(1) Judiciary (Supreme Court and High Courts only). न्यायपालिका (सर्वोच्च न्यायालय और उच्च न्यायालय)।
(2) State Governments राज्य सरकारें
(3) Central Government केंद्र सरकार

Code :-
A. 3
B. 2, 3
C. 1, 3
D. All of the above

(B) 2, 3
Explanation :-
Section (3) of the AFSPA Act empowers the governor of the state or Union territory to issue an official notification on The Gazette of India, following which the centre has the authority to send in armed forces for civilian aid. It is still unclear whether the governor has to prompt the centre to send in the army or whether the centre on its own sends in troops.
Once declared ‘disturbed’, the region has to maintain status quo for a minimum of three months, according to The Disturbed Areas (Special Courts) Act, 1976.
The state governments, as in Tripura’s case, can suggest whether the Act is required to be enforced or not. But under Section (3) of the Act, their opinion can still be overruled by the governor or the centre.
Judiciary cannot declare AFSPA
AFSPA अधिनियम की धारा (3) राज्य या केंद्र शासित प्रदेश के राज्यपाल को भारत के राजपत्र पर एक आधिकारिक अधिसूचना जारी करने का अधिकार देती है, जिसके बाद केंद्र को नागरिक सहायता के लिए सशस्त्र बलों में भेजने का अधिकार है। यह अभी भी स्पष्ट नहीं है कि क्या राज्यपाल को सेना में भेजने के लिए केंद्र को या सेना को सैनिकों में भेजने के लिए संकेत देना होगा।
एक बार maintain परेशान ’घोषित होने के बाद, द डिस्टर्ब्ड एरियाज (विशेष न्यायालय) अधिनियम, 1976 के अनुसार, इस क्षेत्र को न्यूनतम तीन महीने तक यथास्थिति बनाए रखनी होगी।
राज्य सरकारें, जैसा कि त्रिपुरा के मामले में है, यह सुझाव दे सकती है कि अधिनियम को लागू करने की आवश्यकता है या नहीं। लेकिन अधिनियम की धारा (3) के तहत, उनकी राय अभी भी राज्यपाल या केंद्र द्वारा खारिज की जा सकती है।
न्यायपालिका AFSPA की घोषणा नहीं कर सकती

86. Which of the following states/UTs were under Armed Forces (Special Powers) Act (AFSPA) at some or the other point since independence? निम्नलिखित में से कौन सा राज्य / केंद्रशासित प्रदेश सशस्त्र बल (विशेष शक्तियां) अधिनियम (AFSPA) के तहत कुछ या किसी अन्य बिंदु पर थे?
(1) Tripura
(2) Chandigarh
(3) Sikkim

Code :-
A. 2, 3
B. 1, 2
C. 1, 3
D. All of the above

(B) 1, 2
Explanation :-
The Armed Forces (Special Powers) Act was enacted in 1958 to bring under control what the government of India considered ‘disturbed’ areas.
Assam, Nagaland, Manipur (except the Imphal municipal area), Arunachal Pradesh (only the Tirap, Changlang and Longding districts plus a 20-km belt bordering Assam), Meghalaya (confined to a 20-km belt bordering Assam) and Jammu and Kashmir are under AFSPA.
The Armed Forces (Punjab and Chandigarh) Special Powers Act, 1983 passed in 1983 and applicable to Punjab and Chandigarh was withdrawn in 1997, roughly 14 years after it came to force.
While the Punjab government withdrew its Disturbed Areas Act (DAA) in 2008, it continued in Chandigarh till September 2012 when the Punjab and Haryana high court struck it down.
1958 में सशस्त्र बल (विशेष शक्तियां) अधिनियम को नियंत्रण में लाया गया था, जिसे भारत सरकार ने 'अशांत' क्षेत्रों में माना था।
असम, नागालैंड, मणिपुर (इंफाल म्युनिसिपल एरिया को छोड़कर), अरुणाचल प्रदेश (केवल तिरप, चांगलांग और लॉन्गडिंग जिले और असम की सीमा 20 किलोमीटर है), मेघालय (असम और जम्मू और कश्मीर सीमा पर 20 किलोमीटर की सीमा तक सीमित) AFSPA के तहत हैं।
सशस्त्र बल (पंजाब और चंडीगढ़) विशेष शक्तियां अधिनियम, 1983 1983 में पारित और पंजाब और चंडीगढ़ में लागू 1997 में वापस ले लिया गया था, लगभग 14 साल बाद यह लागू हुआ।
जबकि पंजाब सरकार ने 2008 में अपने डिस्टर्बड एरियाज एक्ट (DAA) को वापस ले लिया, लेकिन सितंबर 2012 तक यह चंडीगढ़ में जारी रहा जब पंजाब और हरियाणा उच्च न्यायालय ने इसे रद्द कर दिया।

87. Which of the following is/are true regarding the recent verdict on immunity enjoyed by armed forces? निम्नलिखित में से कौन-सा सशस्त्र बलों द्वारा प्राप्त प्रतिरक्षा पर हाल के फैसले के बारे में सच है / हैं?
(1) Every death is to be thoroughly enquired by the CID at the instance of NHRC.  एनएचआरसी के उदाहरण पर सीआईडी ​​द्वारा हर मौत की गहन पूछताछ की जानी है।
(2) Absolute immunity is only available if the victim is found to be enemy. पूर्ण प्रतिरक्षा तभी उपलब्ध होती है जब पीड़ित शत्रु पाया जाता है।

Code :-
A. Only 1
B. Only 2
C. Both 1 and 2
D. Neither 1 nor 2

(A) Only 1
Explanation :-
Every death caused by the armed forces in a disturbed area, whether the victim is a dreaded criminal or a militant or a terrorist or an insurgent, should be thoroughly enquired into, the Supreme Court held.
This is to address any allegation of use of excessive or retaliatory force beyond the call of duty.
Dealing a blow to the immunity enjoyed by security personnel under the Armed Forces (Special Powers) Act of 1958 (AFSPA) against criminal action for acts committed in disturbed areas, the apex court held that “there is no concept of absolute immunity from trial by a criminal court” if an Army man has committed an offence.
Even if the enquiry finds the victim an enemy, a probe should look into whether excessive or retaliatory force was used.
अशांत क्षेत्र में सशस्त्र बलों के कारण होने वाली हर मौत, चाहे पीड़ित एक खूंखार अपराधी हो या आतंकवादी या आतंकवादी या विद्रोही हो, सर्वोच्च न्यायालय में पूरी तरह से पूछताछ की जानी चाहिए।
यह ड्यूटी के आह्वान से परे अत्यधिक या प्रतिशोधी बल के उपयोग के किसी भी आरोप को संबोधित करने के लिए है।
सशस्त्र बल (विशेष शक्तियां) अधिनियम 1958 (AFSPA) के तहत सुरक्षाकर्मियों द्वारा हासिल की गई प्रतिरक्षा के लिए एक झटका से परेशान क्षेत्रों में किए गए कृत्यों के लिए आपराधिक कार्रवाई के खिलाफ, शीर्ष अदालत ने कहा कि “मुकदमे से पूर्ण प्रतिरक्षा की कोई अवधारणा नहीं है एक आपराधिक अदालत ”यदि सेना के एक व्यक्ति ने अपराध किया है।
यहां तक ​​कि अगर जांच में पीड़ित को दुश्मन पाया जाता है, तो जांच में यह देखना चाहिए कि अत्यधिक या प्रतिशोधी बल का उपयोग किया गया था या नहीं।

88. Which article deals with the Presidential reference to the Supreme Court of India? भारत के सर्वोच्च न्यायालय में राष्ट्रपति के संदर्भ में कौन-सा अनुच्छेद है?
A. 142
B. 130
C. 143
D. 132

(C) 143
Explanation :-
President can seek advice from the Supreme Court on any question of law or fact. However, the advice tendered by the Supreme Court is not binding on the President.
The Constitutional cases or references made by the President under Article 143 are decided by a Bench consisting of at least five judges.
राष्ट्रपति कानून या तथ्य के किसी भी सवाल पर उच्चतम न्यायालय से सलाह ले सकते हैं। हालाँकि, सर्वोच्च न्यायालय द्वारा दी गई सलाह राष्ट्रपति के लिए बाध्यकारी नहीं है।
अनुच्छेद 143 के तहत राष्ट्रपति द्वारा किए गए संवैधानिक मामले या संदर्भ एक बेंच द्वारा कम से कम पांच न्यायाधीशों द्वारा तय किए जाते हैं।

89. Which of the following is/are true? निम्नलिखित में से कौन-सा सत्य है / हैं?
(1) The SC cannot itself restore and can only advice the President to restore the dismissed state government if the Presidential Proclamation in the state is held unconstitutional and invalid. SC खुद को बहाल नहीं कर सकता है और केवल राष्ट्रपति को बर्खास्त राज्य सरकार को बहाल करने की सलाह दे सकता है यदि राज्य में राष्ट्रपति पद का प्रस्ताव असंवैधानिक और अमान्य है।
(2) The state legislative assembly should be dissolved only after the Parliament has approved the presidential proclamation. राज्य विधान सभा को संसद द्वारा राष्ट्रपति घोषणा को मंजूरी देने के बाद ही भंग किया जाना चाहिए।

Code :-
A. Only 1
B. Only 2
C. Both 1 and 2
D. Neither 1 nor 2

(B) Only 2
Explanation :-
If the court holds the presidential proclamation to be unconstitutional and invalid, it has power to restore the dismissed state government and revive the state legislative assembly if it was suspended or dissolved.
The state legislative assembly should be dissolved only after the Parliament has approved the presidential proclamation. Until such approval is given, the president can only suspend the assembly. In case the Parliament fails to approve the proclamation, the assembly would get reactivated.
The question of the state government losing the confidence of the legislative assembly should be decided on the floor of the House and until that is done the ministry should not be unseated.
Where a new political party assumes power at the Centre, it will not have the authority to dismiss ministries formed by other parties in the states.
यदि अदालत राष्ट्रपति की घोषणा को असंवैधानिक और अमान्य मानती है, तो उसके पास बर्खास्त राज्य सरकार को बहाल करने और राज्य विधान सभा को पुनर्जीवित करने या उसे भंग करने की शक्ति है।
संसद के राष्ट्रपति पद की घोषणा को मंजूरी देने के बाद ही राज्य विधान सभा को भंग कर दिया जाना चाहिए। जब तक ऐसी मंजूरी नहीं दी जाती है, तब तक अध्यक्ष केवल विधानसभा को निलंबित कर सकता है। यदि संसद उद्घोषणा को मंजूरी देने में विफल रहती है, तो विधानसभा पुनः सक्रिय हो जाएगी।
राज्य सरकार के विधान सभा का विश्वास खोने के सवाल को सदन के पटल पर रखा जाना चाहिए और जब तक ऐसा नहीं किया जाता तब तक मंत्रालय को एकजुट नहीं होना चाहिए।
जहां एक नया राजनीतिक दल केंद्र में सत्ता ग्रहण करता है, उसे राज्यों में अन्य दलों द्वारा गठित मंत्रालयों को खारिज करने का अधिकार नहीं होगा।

90. Which of the following is/are true regarding Article 370? अनुच्छेद 370 के संबंध में निम्नलिखित में से कौन-सा सच है / हैं?
(1) The Centre can revoke Article 370 which provides special status to Jammu and Kashmir, unilaterally. केंद्र अनुच्छेद 370 को निरस्त कर सकता है जो जम्मू-कश्मीर को एकतरफा रूप से विशेष दर्जा प्रदान करता है।
(2) It does not ratification by half of the states. यह आधे राज्यों द्वारा अनुसमर्थन नहीं करता है।

Code :-
A. Only 1
B. Only 2
C. Both 1 and 2
D. Neither 1 nor 2

(D) Neither 1 nor 2
Explanation :-
According to the clause 3 of Article 370, The President may, by public notification, declare that this article shall cease to be operative, provided that he receives the recommendation of the Constituent Assembly of the State (J&K).
Thus, Article 370 can be revoked only if a new Constituent Assembly of Kashmir recommends revocation.
Since the last Constituent Assembly was dissolved in January 1957 after it completed the task of framing the state’s Constitution, so if the parliament agrees to scrap Article 370, a fresh constituent Assembly will have to be formed.
The constituent Assembly will consist of the same MLAs elected to the State Assembly. Thus the Centre cannot repeal Article 370 without the permission of J&K State.
अनुच्छेद 370 के खंड 3 के अनुसार, राष्ट्रपति, सार्वजनिक अधिसूचना के द्वारा यह घोषणा कर सकते हैं कि यह लेख ऑपरेटिव नहीं होगा, बशर्ते कि उन्हें राज्य की संविधान सभा (J & K) की अनुशंसा प्राप्त हो।
इस प्रकार, अनुच्छेद 370 को केवल तभी रद्द किया जा सकता है जब कश्मीर की एक नई संविधान सभा निरसन की सिफारिश करती है।
चूंकि पिछली संविधान सभा को जनवरी 1957 में भंग कर दिया गया था क्योंकि इसने राज्य के संविधान को तैयार करने का काम पूरा कर लिया था, इसलिए यदि संसद अनुच्छेद 370 को भंग करने के लिए सहमत हो जाती है, तो एक नए घटक विधानसभा का गठन करना होगा।
घटक विधानसभा में राज्य विधानसभा के लिए चुने गए समान विधायक शामिल होंगे। इस प्रकार केंद्र जम्मू और कश्मीर राज्य की अनुमति के बिना धारा 370 को निरस्त नहीं कर सकता है।

91. Which of the following is/are true regarding Cooperative Societies? सहकारी समितियों के संबंध में निम्नलिखित में से कौन सा सच है / हैं?
(1) In case of cooperative banks, other than multi-state cooperative banks board shall be superseded or kept under suspension for a period exceeding six months. सहकारी बैंकों के मामले में, बहु-राज्य सहकारी बैंकों के अलावा अन्य को छह महीने से अधिक की अवधि के लिए निलंबित या निलंबित रखा जाएगा।
(2) The state legislature may provide for access to every member of a co-operative society to the information, accounts, etc. of the co-operative society. राज्य विधायिका सहकारी समिति के प्रत्येक सदस्य को सहकारी समिति की जानकारी, खातों आदि की पहुँच प्रदान कर सकती है।

Code :-
A. Only 1
B. Only 2
C. Both 1 and 2
D. Neither 1 nor 2

(B) Only 2
Explanation :-
In case of cooperative banks, other than multi-state cooperative banks, this period cannot exceed one year.
The state legislature may make provisions to ensure the participation of members in the management of the cooperative society.
Further, it may provide for co-operative education and training for its members.
सहकारी बैंकों के मामले में, बहु-राज्य सहकारी बैंकों के अलावा, यह अवधि एक वर्ष से अधिक नहीं हो सकती है।
राज्य विधायिका सहकारी समिति के प्रबंधन में सदस्यों की भागीदारी सुनिश्चित करने के लिए प्रावधान कर सकती है।
इसके अलावा, यह अपने सदस्यों के लिए सहकारी शिक्षा और प्रशिक्षण प्रदान कर सकता है।

92. Which of the following is/are true regarding Cooperative Societies? सहकारी समितियों के संबंध में निम्नलिखित में से कौन-सा सच है / हैं?
(1) The superintendence, direction and conduct of elections to a co-operative society shall be done by the State Election Commission only. सहकारी समिति के लिए चुनावों का अधीक्षण, निर्देशन और संचालन केवल राज्य निर्वाचन आयोग द्वारा किया जाएगा।
2) The audit report of the accounts of an apex co-operative society shall be laid before the state legislature. एक शीर्ष सहकारी समिति के खातों की लेखा परीक्षा रिपोर्ट राज्य विधायिका के समक्ष रखी जाएगी।

Code :-
A. Only 1
B. Only 2
C. Both 1 and 2
D. Neither 1 nor 2

(B) Only 2
Explanation :-
The superintendence, direction and control of the preparation of electoral rolls and the conduct of elections to a co-operative society shall vest in such body, as may be provided by the state legislature.
Every co-operative society shall be audited by an auditor or auditing firm, appointed by the general body of the co-operative society. But, such an auditor or auditing firm shall be appointed from a panel approved by the State Government or a body authorised by the State Government on this behalf.
The accounts of every co-operative society shall be audited within six months of the close of the financial year.
मतदाता सूची तैयार करने और सहकारी समिति के चुनावों का अधीक्षण, निर्देशन और नियंत्रण ऐसे निकाय में निहित होगा, जो राज्य विधायिका द्वारा प्रदान किया जा सकता है।
प्रत्येक सहकारी समिति का लेखा-जोखा ऑडिटर या ऑडिटिंग फर्म द्वारा किया जाएगा, जिसे सहकारी समिति के सामान्य निकाय द्वारा नियुक्त किया जाएगा। लेकिन, इस तरह के ऑडिटर या ऑडिटिंग फर्म को राज्य सरकार द्वारा अनुमोदित पैनल या राज्य सरकार द्वारा अधिकृत निकाय द्वारा नियुक्त किया जाएगा।
प्रत्येक सहकारी समिति के खातों का वित्त वर्ष के बंद होने के छह महीने के भीतर ऑडिट किया जाएगा।

93. Which of the following is/are true regarding Cooperative Societies? सहकारी समितियों के संबंध में निम्नलिखित में से कौन-सा सच है / हैं?
(1) The number of such co-opted members in board of the cooperative shall not exceed 4. सहकारी बोर्ड में ऐसे सह-चयनित सदस्यों की संख्या 4 से अधिक नहीं होगी।
(2) Co-opted members have the right to vote in any election of the co-operative society. सह-निर्वाचित सदस्यों को सहकारी समिति के किसी भी चुनाव में मतदान करने का अधिकार है।

Code :-
A. Only 1
B. Only 2
C. Both 1 and 2
D. Neither 1 nor 2

(D) Neither 1 nor 2
Explanation :-
The state legislature shall make provisions for co-option of persons having experience in the field of banking, management, finance or specialisation in any other related field, as members of the board.
But, the number of such co-opted members shall not exceed two (in addition to twenty-one directors).
Further, the co-opted members shall not have the right to vote in any election of the co-operative society or be eligible to be elected as office bearers of the board.
The functional directors of a co-operative society shall also be the members of the board and such members shall be excluded for the purpose of counting the total number of directors (that is, twenty one).
राज्य विधायिका बोर्ड के सदस्यों के रूप में किसी अन्य संबंधित क्षेत्र में बैंकिंग, प्रबंधन, वित्त या विशेषज्ञता के क्षेत्र में अनुभव रखने वाले व्यक्तियों के सह-विकल्प के लिए प्रावधान करेगी।
लेकिन, ऐसे सह-चयनित सदस्यों की संख्या दो (इक्कीस निदेशकों के अलावा) से अधिक नहीं होगी।
इसके अलावा, सह-चयनित सदस्यों को सहकारी समिति के किसी भी चुनाव में वोट देने का अधिकार नहीं होगा या वे बोर्ड के पदाधिकारी के रूप में चुने जाने के योग्य नहीं होंगे।
सहकारी समिति के कार्यात्मक निदेशक भी बोर्ड के सदस्य होंगे और ऐसे सदस्यों को कुल निदेशकों की संख्या (यानी इक्कीस) की गणना के उद्देश्य से बाहर रखा जाएगा।

94. The maximum number of directors of a co-operative society shall not exceed :- सहकारी समिति के निदेशकों की अधिकतम संख्या अधिक नहीं होगी: -
A. 25
B. 30
C. 21
D. 15

(C) 21
Explanation :-
The board shall consist of such number of directors as may be provided by the state legislature.
But, the maximum number of directors of a co-operative society shall not exceed twenty-one.
The state legislature shall provide for the reservation of one seat for the Scheduled Castes or the Scheduled Tribes and two seats for women on the board of every co-operative society having members from such a category of persons.
The term of office of elected members of the board and its office bearers shall be five years from the date of election.
बोर्ड में ऐसे कई निदेशक शामिल होंगे जो राज्य विधायिका द्वारा प्रदान किए जा सकते हैं।
लेकिन, सहकारी समिति के निदेशकों की अधिकतम संख्या इक्कीस से अधिक नहीं होगी।
राज्य विधायिका अनुसूचित जाति या अनुसूचित जनजाति के लिए एक सीट और महिलाओं के लिए दो सीटें, प्रत्येक सहकारी समिति के बोर्ड पर महिलाओं के लिए ऐसी श्रेणी के सदस्यों के लिए प्रदान करेगी।
बोर्ड और उसके पदाधिकारियों के निर्वाचित सदस्यों के पद की अवधि चुनाव की तारीख से पांच वर्ष होगी।

95. Which of the following is/are true regarding Cooperative Societies? सहकारी समितियों के संबंध में निम्नलिखित में से कौन-सा सच है / हैं?
(1) Right to form co-operative societies is only a legal right.  सहकारी समितियों के गठन का अधिकार केवल कानूनी अधिकार है।
(2) Co-operatives have been in the directive principles since adoption of the constitution. सहकारिता संविधान को अपनाने के बाद से निर्देशात्मक सिद्धांतों में रही है।
(3) Right to form co-operative societies is a fundamental right. सहकारी समितियों के गठन का अधिकार एक मौलिक अधिकार है।

Code :-
A. 1, 3
B. 1, 2
C. 2, 3
D. All of the above

(C) 2, 3
Explanation :-
The 97th Constitutional Amendment Act of 2011 gave a constitutional status and protection to co-operative societies. In this context, it made the following three changes in the constitution :-
It made the right to form co-operative societies a fundamental right (Article 191).
It included a new Directive Principle of State Policy on promotion of co-operative societies (Article 43-B).
It added a new Part IX-B in the Constitution which is entitled “The Co-operative Societies” (Articles 243-ZH to 243-ZT).
2011 के 97 वें संवैधानिक संशोधन अधिनियम ने सहकारी समितियों को संवैधानिक दर्जा और संरक्षण दिया। इस संदर्भ में, इसने संविधान में निम्नलिखित तीन बदलाव किए: -
इसने सहकारी समितियों को मौलिक अधिकार बनाने का अधिकार बनाया (अनुच्छेद 191)।
इसमें सहकारी समितियों के संवर्धन पर राज्य नीति का एक नया निर्देश सिद्धांत शामिल था (अनुच्छेद 43-बी)।
इसने संविधान में एक नया भाग IX-B जोड़ा है जो "सहकारी समितियों" (अनुच्छेद 243-ZH से 243-ZT) का हकदार है।

96. 97th Constitutional Amendment is related to :- 97 वां संवैधानिक संशोधन संबंधित है: -
A. Name change of Orissa उड़ीसा का नाम परिवर्तन
B. Cooperatives सहकारिता
C. National Judicial Appointments Commission राष्ट्रीय न्यायिक नियुक्ति आयोग
D. Exchange of enclaves with Bangladesh सहकारी

(B) Cooperatives सहकारिता
Explanation :-
The 97th Constitutional Amendment Act of 2011 gave a constitutional status and protection to co-operative societies.
It added a new Part IX-B in the Constitution which is entitled “The Co-operative Societies” (Articles 243-ZH to 243-ZT).
February 15, 2012, is the date of commencement of the Constitution (Ninety-seventh Amendment) Act, 2011. The Centre has asked state governments to amend their respective State Cooperative Society Act in tune with the Constitution (97th Amendment) Act, 2011 before February 14, 2013.
2011 के 97 वें संवैधानिक संशोधन अधिनियम ने सहकारी समितियों को संवैधानिक दर्जा और संरक्षण दिया।
इसने संविधान में एक नया भाग IX-B जोड़ा है जो "सहकारी समितियों" (अनुच्छेद 243-ZH से 243-ZT) का हकदार है।
15 फरवरी, 2012, संविधान (निन्यानबेवें संशोधन) अधिनियम, 2011 के प्रारंभ होने की तारीख है। केंद्र ने राज्य सरकारों से कहा है कि वे संविधान (97 वां संशोधन) अधिनियम, 2011 के अनुसार राज्य संबंधित अपने संबंधित सहकारी समिति अधिनियम में संशोधन करें। 14 फरवरी, 2013।

97. Constitution of India mentions "Cooperative Societies" in :- भारत के संविधान में "सहकारी समितियों" का उल्लेख है :-
(1) Directive Principles of State Policy राज्य नीति के निर्देशक सिद्धांत
(2) 7th Schedule
(3) Local Governments (Panchayats and Municipalities) स्थानीय सरकारें (पंचायत और नगर पालिकाएँ)

Code :-
A. 1, 2
B. 1, 3
C. 2, 3
D. All of the above

(A) 1, 2
Explanation :-
Cooperatives find explicit mention in two places in our Constitution.
First, as part of Article 43 as a Directive Principle which enjoins the State to promote cottage industry through individual or cooperative basis in rural areas and second, in schedule 7 as entries 43 and 44 in the Union list and as entry 32 in the State list.
Cooperative Societies have been recognized as Constitutional Entities.
सहकारिता हमारे संविधान में दो स्थानों पर स्पष्ट उल्लेख करती है।
पहला, एक निर्देशक सिद्धांत के रूप में अनुच्छेद 43 के भाग के रूप में, जो ग्रामीण क्षेत्रों में व्यक्तिगत या सहकारी आधार के माध्यम से कुटीर उद्योग को बढ़ावा देने के लिए राज्य में शामिल है और दूसरा, अनुसूची 7 में संघ सूची में 43 और 44 के रूप में और राज्य सूची में 32 के रूप में प्रवेश करता है। ।
सहकारी समितियों को संवैधानिक संस्थाओं के रूप में मान्यता दी गई है।

98. Which of the following is/are true regarding Judiciary of India? भारत की न्यायपालिका के बारे में निम्नलिखित में से कौन-सा सच है / हैं?
(1) The names of High Courts can be changed by the respective State/s with the permission of the Parliament of India.
(2) Recently names of Bombay, Madras and Calcutta High Courts have been changed.

Code :-
A. Only 1
B. Only 2
C. Both 1 and 2
D. Neither 1 nor 2

(B) Only 2
Explanation :-
The High Courts (Alteration of Names) Bill, 2016 will facilitate the changing of the names of ‘Bombay High Court as Mumbai High Court’ and ‘Madras High Court as Chennai High Court’ respectively. Efforts to change the name of Calcutta High Court to Kolkata High Court are too going on.
The High Courts of Bombay and Madras were named after the cities in which they were located. Consequence to the change in the names of these cities, there has been demands for change in the names of these High Courts.
At present, there is no Central Law under which the proposal for change of names of these High Courts can be addressed. This Legislation is to address this requirement.
Only Parliament can change the names of High Courts. State can make a request but have no power to make the changes.
उच्च न्यायालयों (नामों का परिवर्तन) विधेयक, 2016 मुंबई बॉम्बे उच्च न्यायालय के नाम को 'मुंबई उच्च न्यायालय' और 'मद्रास उच्च न्यायालय को क्रमशः चेन्नई उच्च न्यायालय' के नाम से बदलने की सुविधा प्रदान करेगा। कलकत्ता उच्च न्यायालय का नाम बदलकर कोलकाता उच्च न्यायालय करने के प्रयास भी चल रहे हैं।
बंबई और मद्रास के उच्च न्यायालयों का नाम उन शहरों के नाम पर रखा गया था जिनमें वे स्थित थे। इन शहरों के नाम में परिवर्तन के परिणामस्वरूप, इन उच्च न्यायालयों के नामों में परिवर्तन की मांग की गई है।
वर्तमान में, कोई केंद्रीय कानून नहीं है जिसके तहत इन उच्च न्यायालयों के नाम बदलने के प्रस्ताव को संबोधित किया जा सकता है। यह विधान इस आवश्यकता को संबोधित करने के लिए है।
केवल संसद ही उच्च न्यायालयों के नाम बदल सकती है। राज्य अनुरोध कर सकते हैं लेकिन परिवर्तन करने की शक्ति नहीं है।

99. The President of India inaugurated a smart model village pilot project at Rashtrapati Bhavan. Under it 5 villages are selected in the pilot run for development into smart Villages. They are from which state? भारत के राष्ट्रपति ने राष्ट्रपति भवन में एक स्मार्ट मॉडल विलेज पायलट प्रोजेक्ट का उद्घाटन किया। इसके तहत स्मार्ट गांवों में विकास के लिए पायलट रन में 5 गांवों का चयन किया जाता है। वे किस राज्य से हैं?
A. Haryana
B. Odisha
C. Maharashtra
D. Uttar Pradesh

(A) Haryana
Explanation :-
The President of India inaugurated a smart model village pilot project at Rashtrapati Bhavan.
The five villages which will be developed into the smart villages under this pilot project are Dhaula, Alipur, Harichandpur and Taj Nagar from Gurgaon district and Rojka Meo from Mewat district of Haryana. भारत के राष्ट्रपति ने राष्ट्रपति भवन में एक स्मार्ट मॉडल विलेज पायलट प्रोजेक्ट का उद्घाटन किया।
इस पायलट प्रोजेक्ट के तहत जिन पाँच गाँवों को स्मार्ट गाँवों में विकसित किया जाएगा, वे हैं धौला, अलीपुर, हरिचंदपुर और ताज नगर गुड़गांव जिले से और हरियाणा के मेवात ज़िले से रोज्का मेओ।

100. Which of the following is/are true regarding National Mineral Exploration Policy? राष्ट्रीय खनिज अन्वेषण नीति के बारे में निम्नलिखित में से कौन-सा सच है / हैं?
(1) Private entities engaged in carrying out regional and detailed exploration would get a certain share in revenue, when the region is mined. क्षेत्रीय और विस्तृत अन्वेषण करने में लगे निजी संस्थानों को राजस्व में एक निश्चित हिस्सा मिलेगा, जब इस क्षेत्र में खनन होता है।
(2) Of India’s entire Obvious Geological Potential (OGP) area, identified by GSI, only 10 per cent has been explored. जीएसआई द्वारा पहचाने गए भारत के संपूर्ण स्पष्ट भूवैज्ञानिक संभावित (ओजीपी) क्षेत्र में केवल 10 प्रतिशत की खोज हुई है।

Code :-
A. Only 1
B. Only 2
C. Both 1 and 2
D. Neither 1 nor 2

(C) Both 1 and 2
Explanation :-
One of the important features of NMEP is attractive provisions for private investment in the exploration sector.
NMEP has proposed that private entities engaged in carrying out regional and detailed exploration would get a certain share in revenue (by way of royalty or premium accruing to the State government) in mining operation from the successful bidder after the e-auction of the mineral block.
Selection of private explorer is proposed to be done through a transparent process of competitive bidding through e-auction. For this, reasonable areas or blocks for regional exploration will be earmarked or identified by the government for auctioning.
Of India’s entire Obvious Geological Potential (OGP) area, identified by GSI, only 10 per cent has been explored and mining is undertaken in 1.5-2 per cent of this area.
NMEP की एक महत्वपूर्ण विशेषता अन्वेषण क्षेत्र में निजी निवेश के लिए आकर्षक प्रावधान हैं।
NMEP ने प्रस्तावित किया है कि क्षेत्रीय और विस्तृत अन्वेषण करने में लगे निजी संस्थानों को खनिज ब्लॉक की ई-नीलामी के बाद सफल बोली लगाने वाले से खनन कार्य में राजस्व (राज्य सरकार को रॉयल्टी या प्रीमियम अर्जित करने के माध्यम से) में एक निश्चित हिस्सा मिलेगा। ।
निजी खोजकर्ता का चयन ई-नीलामी के माध्यम से प्रतिस्पर्धी बोली की पारदर्शी प्रक्रिया के माध्यम से किया जाना प्रस्तावित है। इसके लिए, क्षेत्रीय अन्वेषण के लिए उचित क्षेत्रों या ब्लॉकों को नीलामी के लिए सरकार द्वारा निर्धारित या चिह्नित किया जाएगा।
जीएसआई द्वारा पहचाने जाने वाले भारत के संपूर्ण स्पष्ट भूवैज्ञानिक संभावित (ओजीपी) क्षेत्र में, केवल 10 प्रतिशत की खोज की गई है और इस क्षेत्र में 1.5-2 प्रतिशत खनन किया जाता है।

101. LIMBS – an Information Management System is developed for :- LIMBS - के लिए एक सूचना प्रबंधन प्रणाली विकसित की गई है: -
A. Legal System
B. Police Force
C. Legislative purposes
D. None of the above

(A) Legal System कानूनी प्रणाली
Explanation :-
LIMBS is one of the progressive steps taken by the Ministry of Law and Justice in the direction of Digital India.
LIMBS helps in monitoring court cases in a transparent manner which has never been done before. It can be implemented in all Ministries.41 Ministries are currently using it.
LIMBS provides a low cost web technology access to all the stakeholders involved in a court case in a coordinated way whereby it provides inputs which are available seamlessly on 24x7 basis as per the defined access rules. This way, it is possible to know the various stages of a case and monitor its progress on a continued basis through an elaborate set of user friendly reports.
It is in line with Prime Minister's vision of Digital India to digitalise the details of court cases and brings various stakeholders on a single platform.
This will not only help to introduce transparency but also a sense of ownership among various stakeholders during the life cycle of a court case.
Among others, it will help authorities to take 'data driven decision making' and to evaluate performance of various stake holders and to conduct legal audit. This is a simple, user friendly and innovative web based application with robust architect which digitalises all the processes of a court case.
LIMBS डिजिटल इंडिया की दिशा में कानून और न्याय मंत्रालय द्वारा उठाए गए प्रगतिशील कदमों में से एक है।
LIMBS पारदर्शी तरीके से अदालती मामलों की निगरानी करने में मदद करता है जो पहले कभी नहीं किया गया। इसे सभी मंत्रालयों में लागू किया जा सकता है ।.41 मंत्रालय वर्तमान में इसका उपयोग कर रहे हैं।
LIMBS एक समन्वित तरीके से कोर्ट केस में शामिल सभी हितधारकों को एक कम लागत वाली वेब प्रौद्योगिकी पहुंच प्रदान करता है, जिसमें यह इनपुट प्रदान करता है जो परिभाषित एक्सेस नियमों के अनुसार 24x7 आधार पर मूल रूप से उपलब्ध हैं। इस तरह, किसी मामले के विभिन्न चरणों को जानना और उपयोगकर्ता के अनुकूल रिपोर्ट के विस्तृत सेट के माध्यम से निरंतर आधार पर इसकी प्रगति की निगरानी करना संभव है।
यह अदालत के मामलों के विवरण को डिजिटल बनाने के लिए प्रधान मंत्री के डिजिटल इंडिया के दृष्टिकोण के अनुरूप है और विभिन्न हितधारकों को एक मंच पर लाता है।
यह न केवल पारदर्शिता लाने में मदद करेगा, बल्कि एक अदालत के मामले के जीवन चक्र के दौरान विभिन्न हितधारकों के बीच स्वामित्व की भावना भी पैदा करेगा।
दूसरों के बीच, यह अधिकारियों को 'डेटा संचालित निर्णय लेने' और विभिन्न हितधारकों के प्रदर्शन का मूल्यांकन करने और कानूनी ऑडिट करने में मदद करेगा। यह मजबूत वास्तुकार के साथ एक सरल, उपयोगकर्ता के अनुकूल और अभिनव वेब आधारित अनुप्रयोग है जो एक कोर्ट केस की सभी प्रक्रियाओं को डिजिटल बनाता है।

102. Which of the following is/are true regarding National Mineral Exploration Policy? राष्ट्रीय खनिज अन्वेषण नीति के बारे में निम्नलिखित में से कौन-सा सच है / हैं?
(1) About 5 percent of royalty will be set aside for National Mineral Exploration Trust. लगभग 5 प्रतिशत रॉयल्टी राष्ट्रीय खनिज अन्वेषण ट्रस्ट के लिए अलग रखी जाएगी।
(2) All participants will now have to bid for mining blocks through an auction.  सभी प्रतिभागियों को अब नीलामी के माध्यम से खनन ब्लॉकों के लिए बोली लगानी होगी।

Code :-
A. Only 1
B. Only 2
C. Both 1 and 2
D. Neither 1 nor 2

(B) Only 2
Explanation :-
NMEP will pave way for auction of 100 prospective mineral blocks. It boosts India’s mining potential.
It allows private mining companies to carry out regional and detailed exploration of mining blocks.
Earlier, private explorers used to apply for mining leases which would then be given out on a first-cum-first-serve basis. The new policy does away with that and all participants will now have to bid for mining blocks through an auction.
About 2 percent of royalty will be set aside for National Mineral Exploration Trust.
The policy allows host states a share in revenue from mineral exploration. Revenue sharing model for exploration to be lump sum or an annuity basis payment.
NMEP 100 संभावित खनिज ब्लॉकों की नीलामी का मार्ग प्रशस्त करेगा। यह भारत की खनन क्षमता को बढ़ाता है।
यह निजी खनन कंपनियों को खनन ब्लॉकों के क्षेत्रीय और विस्तृत अन्वेषण करने की अनुमति देता है।
पहले, निजी खोजकर्ता खनन पट्टों के लिए आवेदन करते थे जो बाद में पहले-सह-पहले-सेवा के आधार पर दिए जाते थे। नई नीति इससे दूर हो जाती है और सभी प्रतिभागियों को अब नीलामी के माध्यम से खनन ब्लॉकों के लिए बोली लगानी होगी।
लगभग 2 प्रतिशत रॉयल्टी राष्ट्रीय खनिज अन्वेषण ट्रस्ट के लिए अलग रखी जाएगी।
नीति मेजबान राज्यों को खनिज अन्वेषण से राजस्व में हिस्सेदारी की अनुमति देती है। एकमुश्त या वार्षिकी आधार भुगतान के लिए अन्वेषण के लिए राजस्व साझाकरण मॉडल।

103.  Which of the following is/are true regarding CAG? CAG के संबंध में निम्नलिखित में से कौन-सा सत्य है / हैं?
(1) CAG submits three reports to the Parliament of India where the Public Accounts Committee examines them. CAG भारत की संसद को तीन रिपोर्ट सौंपता है जहाँ लोक लेखा समिति उनकी जाँच करती है।
(2) The appropriation accounts compare the actual expenditure with the expenditure sanctioned by the Parliament through the Appropriation Act. विनियोग खाते संसद द्वारा विनियोग अधिनियम के माध्यम से स्वीकृत व्यय से वास्तविक व्यय की तुलना करते हैं।

Code :-
A. Only 1
B. Only 2
C. Both 1 and 2
D. Neither 1 nor 2

(B) Only 2
Explanation :-
The CAG submits three audit reports to the President - audit report on appropriation accounts, audit report on finance accounts, and audit report on public undertakings.
The President lays these reports before both the Houses of Parliament.
After this, the Public Accounts Committee examines them and reports its findings to the Parliament.
The appropriation accounts compare the actual expenditure with the expenditure sanctioned by the Parliament through the Appropriation Act.
CAG राष्ट्रपति को तीन ऑडिट रिपोर्ट प्रस्तुत करता है - विनियोग खातों पर ऑडिट रिपोर्ट, वित्त खातों पर ऑडिट रिपोर्ट और सार्वजनिक उपक्रमों पर ऑडिट रिपोर्ट।
राष्ट्रपति इन रिपोर्टों को संसद के दोनों सदनों के समक्ष रखता है।
इसके बाद, लोक लेखा समिति उनकी जांच करती है और संसद को उसके निष्कर्षों की रिपोर्ट देती है।
विनियोग खाते संसद द्वारा विनियोग अधिनियम के माध्यम से स्वीकृत व्यय के साथ वास्तविक व्यय की तुलना करते हैं।

104. Which constitutional body recently formulated a Big Data Management Policy? हाल ही में किस संवैधानिक संस्था ने बिग डेटा मैनेजमेंट पॉलिसी बनाई?
A. Central Information Commission
B. Finance Commission
C. Election Commission
D. CAG

(D) CAG
Explanation :-
CAG Institution in India has taken several initiatives towards use of latest data analytical tools for audit planning & analysis and is also fine tuning its methodology for audit of environmental issues for better impact.
CAG has formulated a Big Data Management Policy and is in the process of establishing a Data Analytics Centre.
भारत में सीएजी संस्थान ने ऑडिट योजना और विश्लेषण के लिए नवीनतम डेटा विश्लेषणात्मक उपकरणों के उपयोग की दिशा में कई पहल की हैं और बेहतर प्रभाव के लिए पर्यावरणीय मुद्दों के ऑडिट के लिए इसकी कार्यप्रणाली को ठीक कर रहा है।
कैग ने एक बड़ी डेटा प्रबंधन नीति तैयार की है और एक डेटा एनालिटिक्स केंद्र की स्थापना की प्रक्रिया में है।

105. URJA app was recently launched. URJA stands for :- URJA ऐप को हाल ही में लॉन्च किया गया था। URJA का मतलब है: -
A. Urban Jyoti Abhiyaan
B. Urban Rural Jyoti Abhiyaan
C. Jawahar Urban Renewal Abhiyaan
D. None of the above

(A) Urban Jyoti Abhiyaan
Explanation :- 
The app URJA - Urban Jyoti Abhiyaan Mobile app, is developed by Power Finance Corporation on behalf of Ministry of Power for Urban Power Distribution Sector to enhance consumer connect with the Urban Power Distribution sector by providing information of IT enabled towns on important parameters which concern the consumers like outage information, timely release of connections, addressing complaints, power reliability etc.
The app will work as manifestation of Prime Minister’s principle of good governance i.e. People focus, co-operative federalism.
एप्लिकेशन URJA - शहरी ज्योति अभियान मोबाइल ऐप, शहरी विकास वितरण क्षेत्र के लिए बिजली मंत्रालय की ओर से पावर फाइनेंस कॉरपोरेशन द्वारा विकसित किया गया है जो महत्वपूर्ण मापदंडों पर आईटी सक्षम शहरों की जानकारी प्रदान करके शहरी बिजली वितरण क्षेत्र के साथ उपभोक्ता संपर्क को बढ़ाता है, जो चिंता का विषय है उपभोक्ताओं को आउटेज जानकारी, कनेक्शन का समय पर रिलीज, शिकायतों को संबोधित करना, बिजली की विश्वसनीयता आदि।
ऐप प्रधानमंत्री के सुशासन के सिद्धांत यानी लोगों के फोकस, को-ऑपरेटिव फेडरलिज्म के रूप में काम करेगा।

106. Capital of Namibia is :- 
A. Maseru
B. Gaborone
C. Windhoek
D. Accra

(C) Windhoek
Explanation :- 
Recently President of India visited Namibia.
The visit to Namibia was the second by an Indian President and after a gap of 21 years.
Two MOUs were signed, between the Namibia Institute of Public Administration and Management (NIPAM) and Indian Institute of Management, Ahmedabad (IIMA) and on the establishment of a Centre of Excellence in Information Technology in Namibia.
Namibia is located in Southern Africa.
It got independence from South Africa in 1990.
The Government of India accorded full diplomatic recognition to the SWAPO office in New Delhi even before Namibian independence.
South West Africa People's Organization, officially known as SWAPO Party of Namibia, is a political party and former national liberation movement in Namibia.
हाल ही में भारत के राष्ट्रपति ने नामीबिया का दौरा किया।
नामीबिया की यात्रा एक भारतीय राष्ट्रपति द्वारा दूसरी और 21 साल के अंतराल के बाद की गई।
नामीबिया इंस्टीट्यूट ऑफ पब्लिक एडमिनिस्ट्रेशन एंड मैनेजमेंट (NIPAM) और भारतीय प्रबंधन संस्थान, अहमदाबाद (IIMA) के बीच और नामीबिया में सूचना प्रौद्योगिकी के उत्कृष्टता केंद्र की स्थापना के बीच दो समझौता ज्ञापनों पर हस्ताक्षर किए गए।
नामीबिया दक्षिणी अफ्रीका में स्थित है।
इसे 1990 में दक्षिण अफ्रीका से स्वतंत्रता मिली।
भारत सरकार ने नामीबिया की स्वतंत्रता से पहले नई दिल्ली में SWAPO कार्यालय को पूर्ण राजनयिक मान्यता प्रदान की।
दक्षिण पश्चिम अफ्रीका पीपुल्स ऑर्गनाइजेशन, जिसे आधिकारिक तौर पर नामीबिया की SWAPO पार्टी के रूप में जाना जाता है, एक राजनीतिक पार्टी और नामीबिया में पूर्व राष्ट्रीय मुक्ति आंदोलन है।

107. Which of the following are used to decide if an office is "office of profit"?
(1) Whether the government makes the appointment क्या सरकार नियुक्ति करती है
(2) What the functions of the holder are धारक के कार्य क्या हैं
(3) Whether the government pays remuneration क्या सरकार पारिश्रमिक का भुगतान करती है


Code :- 
A. 1, 3
B. 1, 3
C. 2, 3
D. All of the above

(D) All of the above
Explanation :- 
"Office of profit" is not defined in the Constitution. However, in past judgments, the Election Commission has noted “what constitutes an office of profit under the Government is now well established by a catena of judgments of the Supreme Court.”
Five tests have been laid down: (i) whether the government makes the appointment; (ii) whether the government has the right to remove or dismiss the holder; (iii) whether the government pays remuneration; (iv) what the functions of the holder are; and (v) does the government exercise any control over the performance of these functions.
Other broad principles have evolved for determining whether an office attracts the constitutional disqualification - Whether the body in which the office is held has government powers (releasing money, allotment of land, granting licences etc.); whether the office enables the holder to influence by way of patronage.
EC noted that the Supreme Court, in various cases has held that all the five tests “need not co-exist conjointly for determining whether an office is an office of profit under the government.”
"ऑफिस ऑफ प्रॉफिट" संविधान में परिभाषित नहीं है। हालांकि, पिछले निर्णयों में, चुनाव आयोग ने उल्लेख किया है कि "सरकार के अधीन लाभ का कार्यालय गठित करने का कार्य अब सर्वोच्च न्यायालय के निर्णयों के एक समूह द्वारा अच्छी तरह से स्थापित किया गया है।"
पांच परीक्षण निर्धारित किए गए हैं: (i) क्या सरकार नियुक्ति करती है; (ii) क्या सरकार को धारक को हटाने या खारिज करने का अधिकार है; (iii) क्या सरकार पारिश्रमिक का भुगतान करती है; (iv) धारक के कार्य क्या हैं; और (v) क्या सरकार इन कार्यों के प्रदर्शन पर कोई नियंत्रण रखती है।
अन्य व्यापक सिद्धांत यह निर्धारित करने के लिए विकसित हुए हैं कि क्या कोई कार्यालय संवैधानिक अयोग्यता को आकर्षित करता है - चाहे वह निकाय जिसमें कार्यालय में सरकारी शक्तियां हों (धन जारी करना, भूमि का आवंटन, लाइसेंस देना आदि); क्या कार्यालय धारक को संरक्षण के माध्यम से प्रभावित करने में सक्षम बनाता है।
चुनाव आयोग ने कहा कि उच्चतम न्यायालय ने विभिन्न मामलों में माना है कि सभी पांच परीक्षणों में "यह निर्धारित करने के लिए सह-अस्तित्व की आवश्यकता नहीं है कि क्या एक कार्यालय सरकार के अधीन लाभ का कार्यालय है।"

108. Which article deals with the cap on total number of ministers in the Union Government? केंद्र सरकार में मंत्रियों की कुल संख्या पर टोपी के साथ कौन-सा लेख है?
A. 75(1)
B. 75(1A)
C. 74 (2)
D. 75(1B)

(B) 75(1A)
Explanation :- 
Article 75 (1A) states that the total number of Ministers, including the Prime Minister, in the Council of Ministers shall not exceed fifteen per cent. of the total number of members of the House of the People.
It was inserted by the 91st Constitutional Amendment Act, 2003.
A similar cap was placed on states too. Here the minimum number of ministers should not be less than 12. For smaller states the minimum number is 7.
अनुच्छेद 75 (1 ए) कहता है कि मंत्रिपरिषद में प्रधान मंत्री सहित कुल मंत्रियों की संख्या पंद्रह प्रतिशत से अधिक नहीं होगी। लोक सभा के सदस्यों की कुल संख्या।
इसे 91 वें संवैधानिक संशोधन अधिनियम, 2003 द्वारा डाला गया था।
राज्यों पर भी इसी तरह की टोपी लगाई गई थी। यहां मंत्रियों की न्यूनतम संख्या 12. से कम नहीं होनी चाहिए। छोटे राज्यों के लिए न्यूनतम संख्या 7 है।

109. Number of ministers including the Chief Minister has to be within how much percentage of the total number of members of the Delhi Assembly? मुख्यमंत्री सहित मंत्रियों की संख्या दिल्ली विधानसभा के कुल सदस्यों के कितने प्रतिशत के भीतर है?
A. 15
B. 7
C. 10
D. 5

(C) 10
Explanation :- 
Article 164(1A) specifies that the number of ministers including the Chief Minister has to be within 15% of the total number of members of the Assembly.
It is 10% in the case of Delhi, which is not a ‘full’ state. This has been mentioned in Article 239AA of the Indian Constitution.
अनुच्छेद 164 (1 ए) निर्दिष्ट करता है कि मुख्यमंत्री सहित मंत्रियों की संख्या विधानसभा के सदस्यों की कुल संख्या के 15% के भीतर होनी चाहिए।
यह दिल्ली के मामले में 10% है, जो 'पूर्ण' राज्य नहीं है। भारतीय संविधान के अनुच्छेद 239AA में इसका उल्लेख किया गया है।

110. Which of the following is/are true about Parliamentary secretaries? संसदीय सचिवों के बारे में निम्नलिखित में से कौन-सा सच है / हैं?
(1) Parliamentary Secretary helps other ministers in their Parliamentary work. संसदीय सचिव अन्य मंत्रियों को उनके संसदीय कार्य में मदद करता है।
(2) Appointing Parliamentary Secretaries is being seen as an attempt to bypass the cap on number of Council of Ministers. संसदीय सचिवों की नियुक्ति को मंत्रिपरिषद की संख्या के आधार पर टोपी को दरकिनार करने के प्रयास के रूप में देखा जा रहा है।

Code :- 
A. Only 1
B. Only 2
C. Both 1 and 2
D. Neither 1 nor 2

(C) Both 1 and 2
Explanation :- 
A Parliamentary Secretary is a minister however he/she is not included within the limit of 15% put on the number of Council of Ministers.
A Parliamentary Secretary, who is not a member of the House, is not entitled to attend its sitting.
Office of Profit applies to elected persons who derive other financial benefits, over their elected rights.
Constitutional experts affirm that government salaries or facilities not related to rights, after taking office, would come under the purview of office of profit.
एक संसदीय सचिव एक मंत्री होता है, हालांकि उसे मंत्रिपरिषद की संख्या के 15% की सीमा के भीतर शामिल नहीं किया जाता है।
एक संसदीय सचिव, जो सदन का सदस्य नहीं है, उसके बैठने का अधिकार नहीं है।
ऑफिस ऑफ प्रॉफिट उन चुने हुए व्यक्तियों पर लागू होता है जो अपने चुने हुए अधिकारों पर अन्य वित्तीय लाभ प्राप्त करते हैं।
संवैधानिक विशेषज्ञ इस बात की पुष्टि करते हैं कि सरकारी वेतन या सुविधाएं, अधिकारों से संबंधित नहीं हैं, पद ग्रहण करने के बाद, लाभ के पद के दायरे में आएंगी।

111. Which of the following is/are true about Parliamentary Secretary? संसदीय सचिव के बारे में निम्नलिखित में से कौन-सा सच है / हैं?
(1) Parliamentary Secretary is a type of Council of Ministers. संसदीय सचिव मंत्रिपरिषद का एक प्रकार है।
(2) Parliamentary Secretary post owes its origin to the Constitution of India. संसदीय सचिव पद भारत के संविधान के मूल में है।

Code :- 
A. Only 1
B. Only 2
C. Both 1 and 2
D. Neither 1 nor 2

(A) Only 1
Explanation :- 
Many states in the Indian Union have instituted the post of Parliamentary Secretary.
A Parliament Secretary often holds the rank of Minister of State and has the same entitlements and is assigned to a government department.
Manipur, HP, Mizoram, Assam, Rajasthan, Punjab, Goa are some of the states where MLAs have been appointed Parliament Secretaries by the Government.
Various High Courts have deemed the appointment of Parliamentary Secretaries unconstitutional and have ruled against such appointments often in the past.
The post is also considered as among the ones which are called offices of profit.
भारतीय संघ के कई राज्यों ने संसदीय सचिव के पद की स्थापना की है।
एक संसद सचिव अक्सर राज्य मंत्री का पद रखता है और उसके समान अधिकार प्राप्त होते हैं और उसे एक सरकारी विभाग को सौंपा जाता है।
मणिपुर, एचपी, मिजोरम, असम, राजस्थान, पंजाब, गोवा कुछ ऐसे राज्य हैं जहाँ विधायकों को सरकार द्वारा संसद सचिव नियुक्त किया गया है।
विभिन्न उच्च न्यायालयों ने संसदीय सचिवों की नियुक्ति को असंवैधानिक माना है और अतीत में अक्सर ऐसी नियुक्तियों के खिलाफ फैसला सुनाया है।
पद को उन लोगों के बीच भी माना जाता है जिन्हें लाभ के कार्यालय कहा जाता है।

112. Recently there have been debates on office of profit. Which of the following posts are at the centre of those discussions? हाल ही में ऑफिस ऑफ प्रॉफिट पर बहस हुई है। निम्नलिखित में से कौन सा पद उन चर्चाओं के केंद्र में है?
(1) Parliamentary Secretaries
(2) MPs
(3) MLAs 

Code :- 
A. 1, 2
B. 2, 3
C. 1, 3
D. All of the above

(D) All of the above
Explanation :- 
Many states have appointed MLAs as Parliamentary Secretaries.
Sometimes MPs too are appointed as Parliamentary Secretaries.
There have been PILs filed citing Article 102 of the Constitution of India, which deals with disqualification of members of parliament for holding office of profit.
The post of Parliamentary secretary has been considered a drain of money.
Delhi legislative Assembly sought to exclude this post from the list of office of profit, however its proposal has been refused assent by the president.
कई राज्यों ने विधायकों को संसदीय सचिव नियुक्त किया है।
कभी-कभी सांसदों को संसदीय सचिव के रूप में भी नियुक्त किया जाता है।
भारत के संविधान के अनुच्छेद 102 का हवाला देते हुए जनहित याचिका दायर की गई है, जो लाभ के पद धारण करने के लिए संसद के सदस्यों की अयोग्यता से संबंधित है।
संसदीय सचिव के पद को धन की नाली माना गया है।
दिल्ली विधान सभा ने इस पद को लाभ के पद की सूची से बाहर करने की मांग की, हालाँकि इसके प्रस्ताव को राष्ट्रपति द्वारा स्वीकार करने से इनकार कर दिया गया है।

113. A Joint Committee on office of profit has been constituted to regularly review various offices for deciding if they are office of profit? What is the composition of this committee? लाभ के कार्यालय पर एक संयुक्त समिति का गठन विभिन्न कार्यालयों की नियमित समीक्षा करने के लिए किया गया है कि क्या वे लाभ के पद हैं? इस समिति की संरचना क्या है?
A. 15 members from the Lok Sabha and 10 members from the Rajya Sabha. लोकसभा से 15 सदस्य और राज्यसभा से 10 सदस्य।
B. 15 members from the Lok Sabha.
C. 10 members from the Lok Sabha and 5 from the Rajya Sabha.  लोकसभा से 10 सदस्य और राज्य सभा से 5 सदस्य।
D. 14 members from the Lok Sabha and 7 members from the Rajya Sabha. लोकसभा से 14 सदस्य और राज्य सभा से 7 सदस्य।

(C) 10 members from the Lok Sabha and 5 from the Rajya Sabha.  लोकसभा से 10 सदस्य और राज्य सभा से 5 सदस्य।
Explanation :- 
As new bodies are created regularly, the question as to the membership of which of these bodies would be a disqualification for membership of Parliament is a matter demanding constant review.
To meet this need, a Joint Committee on office of profit has been constituted consisting of 10 members from the Lok Sabha and five members from the Rajya Sabha.
The function of the Committee, inter alia, is to undertake a continuous scrutiny of the composition and character of various government appointed bodies and report to both houses as to the membership of which of these ought to disqualify a person for membership of Parliament.
The Committee generally applies two tests in deciding whether a member of a body ought to be exempted from disqualification: (1) the emoluments and allowances attached to the members; and (2) the nature and function of the body.
चूंकि नए निकाय नियमित रूप से बनाए जाते हैं, इसलिए सवाल यह है कि इनमें से किस निकाय की सदस्यता के लिए संसद की सदस्यता के लिए अयोग्यता होगी? यह लगातार समीक्षा की मांग का विषय है।
इस आवश्यकता को पूरा करने के लिए, लाभ के पद पर एक संयुक्त समिति का गठन किया गया है जिसमें लोकसभा के 10 सदस्य और राज्यसभा के पांच सदस्य शामिल हैं।
समिति का कार्य, अन्य बातों के साथ, संसद की सदस्यता के लिए किसी व्यक्ति को अयोग्य घोषित करने के लिए विभिन्न सरकारी निकायों की संरचना और चरित्र की निरंतर जांच करना और दोनों सदनों को रिपोर्ट करना है।
समिति आम तौर पर यह तय करने में दो परीक्षण लागू करती है कि क्या किसी निकाय के सदस्य को अयोग्यता से छूट दी जानी चाहिए: (1) सदस्यों को संलग्न और भत्ते; और (2) शरीर की प्रकृति और कार्य।

114. Which of the following is/are true regarding office of profit? निम्नलिखित में से कौन-सा कार्यालय लाभ के संबंध में सही है / हैं?
(1) Office of profit is defined in the Representation of the People Act, 1951. ऑफिस ऑफ प्रॉफिट को जन प्रतिनिधित्व अधिनियम, 1951 में परिभाषित किया गया है।
(2) Parliament can change rules and laws to allow its members to hold certain offices of profit. संसद अपने सदस्यों को लाभ के कुछ कार्यालय रखने की अनुमति देने के लिए नियम और कानून बदल सकती है।

Code :-
A. Only 1
B. Only 2
C. Both 1 and 2
D. Neither 1 nor 2

(B) Only 2
Explanation :-
A minister in the Union or state government is not considered as holding the office of profit.
Also, the Parliament can declare that a particular office of profit will not disqualify its holder from parliamentary membership.
There is neither a Constitutional nor statutory definition of “office of profit”.
The exemption from the definition of ‘office of profit’, however, has been done by Parliament on a case by case and ad hoc basis and not on the basis of any universal definition.
One source of clarity on the expression are the rulings of courts. Another source is the Election Commission rulings.
संघ या राज्य सरकार में एक मंत्री को लाभ का पद धारण करने वाला नहीं माना जाता है।
साथ ही, संसद यह घोषणा कर सकती है कि लाभ का एक विशेष कार्यालय अपने धारक को संसदीय सदस्यता से अयोग्य नहीं ठहराएगा।
"लाभ के कार्यालय" की न तो संवैधानिक है और न ही वैधानिक परिभाषा है।
हालाँकि, 'ऑफिस ऑफ़ प्रॉफ़िट' की परिभाषा से छूट 'संसद' द्वारा केस और तदर्थ आधार पर संसद द्वारा की गई है, न कि किसी सार्वभौमिक परिभाषा के आधार पर।
अभिव्यक्ति पर स्पष्टता का एक स्रोत अदालतों के फैसले हैं। एक अन्य स्रोत चुनाव आयोग के फैसले हैं।

115. Which article deals with the disqualification of members of Parliament? कौन-सा अनुच्छेद संसद के सदस्यों की अयोग्यता से संबंधित है?
A. 101
B. 191
C. 103
D. 102

(D) 102
Explanation :-
Article 102 deals with the disqualification of members of parliament.
Article 191 deals with disqualification of members of state legislature.
Article 101 deals with vacation of seats
Article 103 deals with decisions on questions of disqualifications of members of Parliament.
अनुच्छेद 102 संसद के सदस्यों की अयोग्यता से संबंधित है।
अनुच्छेद 191 राज्य विधायिका के सदस्यों की अयोग्यता से संबंधित है।
अनुच्छेद 101 सीटों की छुट्टी से संबंधित है
अनुच्छेद 103 संसद के सदस्यों की अयोग्यता के प्रश्नों पर निर्णय लेता है।

116. Which of the following became first state to give social welfare benefits to transgender community? निम्नलिखित में से कौन ट्रांसजेंडर समुदाय को सामाजिक कल्याण लाभ देने वाला पहला राज्य बन गया?
A. Odisha
B. West Bengal
C. Kerala
D. Tamil Nadu

(A) Odisha
Explanation :-
Odisha is first in the country to give transgender people social welfare benefits - such as a pension, housing and food grains - usually allocated for only the most impoverished.
The move to give the transgender community the same benefits as those living below the poverty line was aimed at improving their overall social and economic status.
Members of the transgender community will be given Below Poverty Line (BPL) cards that will allow them to access benefits under various government welfare programmes.
This includes schemes that provide free housing, 100 days of paid work annually, pensions and loans to start up their own businesses. They would also be entitled 5 kg of food grains every month under India's National Food Security Act.
ट्रांसजेंडर लोगों को सामाजिक कल्याण लाभ देने के लिए ओडिशा देश में पहले स्थान पर है - जैसे कि पेंशन, आवास और खाद्यान्न - आमतौर पर केवल सबसे कमजोर लोगों के लिए आवंटित किया जाता है।
ट्रांसजेंडर समुदाय को गरीबी रेखा से नीचे जीवन यापन करने वाले लोगों को उनकी समग्र सामाजिक और आर्थिक स्थिति में सुधार लाने के उद्देश्य से यह लाभ दिया गया।
ट्रांसजेंडर समुदाय के सदस्यों को गरीबी रेखा से नीचे (बीपीएल) कार्ड दिए जाएंगे जो उन्हें विभिन्न सरकारी कल्याण कार्यक्रमों के तहत लाभ प्राप्त करने की अनुमति देगा।
इसमें ऐसी योजनाएं शामिल हैं जो मुफ्त आवास प्रदान करती हैं, सालाना 100 दिन का भुगतान किया गया काम, पेंशन और अपना खुद का व्यवसाय शुरू करने के लिए ऋण। वे भारत के राष्ट्रीय खाद्य सुरक्षा अधिनियम के तहत हर महीने 5 किलो अनाज का हकदार भी होंगे।

117.  Which of the following is/are true regarding Financial Bill (I)? वित्तीय विधेयक (I) के बारे में निम्नलिखित में से कौन-सा सच है / हैं?
(1) Financial bill (I) contains provisions involving expenditure from the Consolidated Fund of India, but does not include any of the matters mentioned in Article 110. वित्तीय बिल (I) में भारत के समेकित कोष से व्यय से संबंधित प्रावधान हैं, लेकिन इसमें अनुच्छेद 110 में उल्लिखित कोई भी मामला शामिल नहीं है।
(2) Financial bill (I) can only be introduced in the Lok Sabha on the recommendation of the President but can be rejected by the Rajya Sabha. वित्तीय बिल (I) केवल राष्ट्रपति की सिफारिश पर लोकसभा में पेश किया जा सकता है, लेकिन राज्यसभा द्वारा खारिज किया जा सकता है।

Code :-
A. Only 1
B. Only 2
C. Both 1 and 2
D. Neither 1 nor 2

(B) Only 2
Explanation :-
A financial bill (I) is a bill that contains not only any or all the matters mentioned in Article 110, but also other matters of general legislation. For instance, a bill that contains a borrowing clause, but does not exclusively deal with borrowing.
In two respects, a financial bill (I) is similar to a money bill—(a) both of them can be introduced only in the Lok Sabha and not in the Rajya Sabha, and (b) both of them can be introduced only on the recommendation of the president.
In all other respects, a financial bill (I) is governed by the same legislative procedure applicable to an ordinary bill. Hence, it can be either rejected or amended by the Rajya Sabha (except that an amendment other than for reduction or abolition of a tax cannot be moved in either House without the recommendation of the president).
A financial bill (II) contains provisions involving expenditure from the Consolidated Fund of India, but does not include any of the matters mentioned in Article 110.
एक वित्तीय बिल (I) एक ऐसा बिल है जिसमें अनुच्छेद 110 में उल्लिखित कोई भी या सभी मामले नहीं हैं, बल्कि सामान्य कानून के अन्य मामले भी हैं। उदाहरण के लिए, एक बिल जिसमें एक उधार क्लॉज़ होता है, लेकिन यह विशेष रूप से उधार लेने से नहीं निपटता है।
दो अर्थों में, एक वित्तीय बिल (I) एक मनी बिल के समान है- (a) इन दोनों को केवल लोकसभा में ही पेश किया जा सकता है और राज्यसभा में नहीं, और (b) इन दोनों को ही पेश किया जा सकता है राष्ट्रपति की सिफारिश।
अन्य सभी मामलों में, एक वित्तीय बिल (I) एक साधारण बिल पर लागू एक ही विधायी प्रक्रिया द्वारा शासित होता है। इसलिए, इसे या तो राज्यसभा द्वारा अस्वीकार या संशोधित किया जा सकता है (सिवाय इसके कि टैक्स में कटौती या उन्मूलन के अलावा कोई संशोधन राष्ट्रपति की सिफारिश के बिना किसी भी सदन में स्थानांतरित नहीं किया जा सकता)।
एक वित्तीय विधेयक (II) में भारत के समेकित निधि से व्यय शामिल प्रावधान हैं, लेकिन अनुच्छेद 110 में उल्लिखित मामलों में से कोई भी शामिल नहीं है।

118. How many types of financial bills are there? वित्तीय बिल कितने प्रकार के होते हैं?
A. 3
B. 2
C. 4
D. 1

(A) 3
Explanation :-
Financial bills are those bills that deal with fiscal matters, that is, revenue or expenditure. However, the Constitution uses the term ‘financial bill’ in a technical sense.
Financial bills are of three kinds :-
1. Money bills—Article 110
2. Financial bills (I)—Article 117 (1)
3. Financial bills (II)—Article 117 (3)
This classification implies that money bills are simply a species of financial bills. Hence, all money bills are financial bills but all financial bills are not money bills.
Only those financial bills are money bills which contain exclusively those matters which are mentioned in Article 110 of the Constitution.
These are also certified by the Speaker of Lok Sabha as money bills. The financial bills (I) and (II), on the other hand, have been dealt with in Article 117 of the Constitution.
वित्तीय बिल वे बिल होते हैं जो राजकोषीय मामलों से संबंधित होते हैं, अर्थात् राजस्व या व्यय। हालाँकि, संविधान तकनीकी अर्थ में 'वित्तीय बिल' शब्द का उपयोग करता है।
वित्तीय बिल तीन प्रकार के होते हैं: -
1. मनी बिल- अनुच्छेद 110
2. वित्तीय बिल (I) -आर्टिकल 117 (1)
3. वित्तीय बिल (II) -आर्टिकल 117 (3)
इस वर्गीकरण का तात्पर्य है कि धन बिल केवल वित्तीय बिलों की एक प्रजाति है। इसलिए, सभी मनी बिल वित्तीय बिल हैं, लेकिन सभी वित्तीय बिल मनी बिल नहीं हैं।
केवल वे वित्तीय बिल ही मनी बिल होते हैं जिनमें विशेष रूप से वे मामले होते हैं जिनका उल्लेख संविधान के अनुच्छेद 110 में किया गया है।
इन्हें लोकसभा अध्यक्ष द्वारा मनी बिल के रूप में भी प्रमाणित किया जाता है। दूसरी ओर, वित्तीय बिल (I) और (II) को संविधान के अनुच्छेद 117 में निपटा दिया गया है।

119. Which of the following is/are true? निम्नलिखित में से कौन सा सत्य है / हैं?
(1) If a bill is a combination of financial matters and some other provisions not incidental to those matters, it can be called a money bill. यदि कोई विधेयक वित्तीय मामलों का संयोजन है और कुछ अन्य प्रावधान उन मामलों के लिए आकस्मिक नहीं हैं, तो इसे धन विधेयक कहा जा सकता है।
(2) There is no remedy lying with Rajya Sabha for wrong decision taken by speaker regarding certification of money bill. धन विधेयक के प्रमाणीकरण के संबंध में स्पीकर द्वारा लिए गए गलत निर्णय के लिए राज्यसभा के पास कोई उपाय नहीं है।

Code :-
A. Only 1
B. Only 2
C. Both 1 and 2
D. Neither 1 nor 2

(B) Only 2
Explanation :-
Article 110 defines what constitutes money bill and the Constitution has vested power to certify money bill in Lok Sabha Speaker, whose decision is final.
If any question arises whether a bill is a money bill or not, the decision of the Speaker of the Lok Sabha is final. His decision in this regard cannot be questioned in any court of law or in the either House of Parliament or even the president.
Only those financial bills are money bills which contain exclusively those matters which are mentioned in Article 110 of the Constitution.
अनुच्छेद 110 यह बताता है कि धन विधेयक का गठन क्या है और संविधान में लोकसभा अध्यक्ष में धन विधेयक को प्रमाणित करने की शक्ति निहित है, जिसका निर्णय अंतिम है।
यदि कोई प्रश्न उठता है कि कोई विधेयक धन विधेयक है या नहीं, लोकसभा के अध्यक्ष का निर्णय अंतिम है। इस संबंध में उनके निर्णय को कानून के किसी भी न्यायालय या संसद के किसी भी सदन या राष्ट्रपति में पूछताछ नहीं की जा सकती है।
केवल वे वित्तीय बिल ही मनी बिल होते हैं जिनमें विशेष रूप से वे मामले होते हैं जिनका उल्लेख संविधान के अनुच्छेद 110 में किया गया है।

120. Which of the following states were also Union territories at some point after Independence of India till now? निम्नलिखित में से कौन से राज्य अब तक भारत की स्वतंत्रता के बाद किसी समय केंद्र शासित प्रदेश थे?
(1) Arunachal Pradesh
(2) Tripura
(3) Meghalaya
(4) Goa
(5) Mizoram

Code :-
A. 1, 2, 4, 5
B. 1, 3, 4
C. 2, 3, 5
D. All of the above

(A) 1, 2, 4, 5
Explanation :-
Before attaining full statehood, Meghalaya was given semi-autonomous status in 1970. It was a sub-state.
In 1971, the Parliament passed the North-Eastern Areas (Reorganization) Act, 1971, which conferred full statehood on the autonomous state of Meghalaya. Meghalaya attained statehood on 21 January 1972, with a Legislative Assembly of its own.
In 1972, the political map of Northeast India underwent a major change. Thus, the two Union Territories of Manipur and Tripura and the Sub-State of Meghalaya got statehood and the two union territories of Mizoram and Arunachal Pradesh (originally known as North-East Frontier Agency—NEFA) came into being.
पूर्ण राज्य का दर्जा पाने से पहले, मेघालय को 1970 में अर्ध-स्वायत्त दर्जा दिया गया था। यह एक उप-राज्य था।
1971 में, संसद ने पूर्वोत्तर क्षेत्र (पुनर्गठन) अधिनियम, 1971 पारित किया, जिसने मेघालय के स्वायत्त राज्य पर पूर्ण राज्य का दर्जा दिया। मेघालय ने 21 जनवरी 1972 को अपनी खुद की एक विधान सभा के साथ राज्य का दर्जा प्राप्त किया।
1972 में, पूर्वोत्तर भारत के राजनीतिक मानचित्र में एक बड़ा बदलाव आया। इस प्रकार, मणिपुर और त्रिपुरा के दो केंद्र शासित प्रदेश और मेघालय के उप-राज्य को राज्य का दर्जा मिला और मिज़ोरम और अरुणाचल प्रदेश के दो केंद्र शासित प्रदेश (मूल रूप से नॉर्थ-ईस्ट फ्रंटियर एजेंसी- NEFA) के रूप में जाना जाता है।

121. Which of the following is/are true? निम्नलिखित में से कौन सा सत्य है / हैं?
(1) Inter-State Council and Zonal Councils earlier had a common secretariat but now they have separate ones. इंटर-स्टेट काउंसिल और जोनल काउंसिल में पहले एक सामान्य सचिवालय था, लेकिन अब उनके पास अलग-अलग हैं।
(2) The Constitution contains the “Full Faith and Credit” clause, as per which "public acts" includes only executive acts of the government and not legislative acts. संविधान में "पूर्ण विश्वास और ऋण" खंड शामिल है, जिसके अनुसार "सार्वजनिक कृत्यों" में सरकार के केवल कार्यकारी कार्य शामिल हैं न कि विधायी कार्य।

Code :-
A. Only 1
B. Only 2
C. Both 1 and 2
D. Neither 1 nor 2

(D) Neither 1 nor 2
Explanation :-
The Council is assisted by a secretariat called the Inter-State Council Secretariat. This secretariat was set-up in 1991 and is headed by a secretary to the Government of India. Since 2011, it is also functioning as the secretariat of the Zonal Councils.
Under the Constitution, the jurisdiction of each state is confined to its own territory. Hence, it is possible that the acts and records of one state may not be recognised in another state. To remove any such difficulty, the Constitution contains the “Full Faith and Credit” clause which lays down the following:
(i) Full faith and credit is to be given throughout the territory of India to public acts, records and judicial proceedings of the Centre and every state.
(ii) The expression "public acts" includes both legislative and executive acts of the government.
(iii) The expression "public record" includes any official book, register or record made by a public servant in the discharge of his official duties.
परिषद को एक सचिवालय द्वारा सहायता प्राप्त है जिसे अंतर-राज्य परिषद सचिवालय कहा जाता है। यह सचिवालय 1991 में स्थापित किया गया था और भारत सरकार के एक सचिव के नेतृत्व में है। 2011 से, यह जोनल काउंसिल के सचिवालय के रूप में भी काम कर रहा है।
संविधान के तहत, प्रत्येक राज्य का अधिकार क्षेत्र उसके अपने क्षेत्र तक ही सीमित है। इसलिए, यह संभव है कि एक राज्य के कृत्यों और रिकॉर्ड को दूसरे राज्य में मान्यता नहीं दी जाए। इस तरह की किसी भी कठिनाई को दूर करने के लिए, संविधान में "पूर्ण विश्वास और ऋण" खंड शामिल है जो नीचे दिए गए हैं:
(i) केंद्र और प्रत्येक राज्य के सार्वजनिक कृत्यों, अभिलेखों और न्यायिक कार्यवाहियों के लिए पूरे भारत में पूरे विश्वास और श्रेय दिया जाना है।
(ii) अभिव्यक्ति "सार्वजनिक कृत्यों" में सरकार के विधायी और कार्यकारी कार्य शामिल हैं।
(iii) अभिव्यक्ति "सार्वजनिक रिकॉर्ड" में किसी सरकारी कर्मचारी द्वारा अपने आधिकारिक कर्तव्यों के निर्वहन में कोई आधिकारिक पुस्तक, रजिस्टर या रिकॉर्ड शामिल है।

122.  Nai Udaan Scheme is for :-
A. Minorities
B. Women
C. Children between ages 6-14 years
D. Differently abled

(A) Minorities अल्पसंख्यक
Explanation :-
Government of India, Ministry of Minority Affairs is implementing the scheme Nai Udaan- Scheme for Support to Minority Students for preparation of Main Examination, who clear Prelims conducted by Union Public Service Commission, Staff Selection Commission, State Public Service Commissions, etc.
The objective of the Scheme is to provide financial support to the minority candidates clearing prelims conducted by Union Public Service Commission, Staff Selection Commission and State Public Service Commissions to adequately equip them to compete for appointment to Civil Services in the Union and the State Governments and to increase the representation of the minority in the Civil Services.
Taking forward the concept of Digital India the Ministry also launched Nai Udan Portal.
भारत सरकार, अल्पसंख्यक मामलों के मंत्रालय ने मुख्य परीक्षा की तैयारी के लिए अल्पसंख्यक छात्रों को सहायता के लिए स्कीम नई उदय- योजना लागू कर रही है, जो संघ लोक सेवा आयोग, कर्मचारी चयन आयोग, राज्य लोक सेवा आयोगों, आदि द्वारा आयोजित प्रारंभिक परीक्षाओं को पास करती है।
योजना का उद्देश्य संघ लोक सेवा आयोग, कर्मचारी चयन आयोग और राज्य लोक सेवा आयोगों द्वारा आयोजित अल्पसंख्यकों के उम्मीदवारों को वित्तीय सहायता प्रदान करना है, जो उन्हें संघ और राज्य सरकारों में सिविल सेवाओं में नियुक्ति के लिए प्रतिस्पर्धा करने के लिए पर्याप्त रूप से लैस कर सकें। सिविल सेवा में अल्पसंख्यक का प्रतिनिधित्व बढ़ाने के लिए।
डिजिटल इंडिया की अवधारणा को आगे बढ़ाते हुए मंत्रालय ने नाइ उदान पोर्टल भी लॉन्च किया।

123. Which of the following are the financial powers of the president of India? निम्नलिखित में से कौन भारत के राष्ट्रपति की वित्तीय शक्तियाँ हैं?
(1) Money bills can be introduced in the Parliament only with his prior recommendation. धन विधेयक संसद में उसकी पूर्व अनुशंसा के साथ ही प्रस्तुत किए जा सकते हैं।
(2) He lays the reports of the Comptroller and Auditor General before the Parliament. वह संसद के समक्ष नियंत्रक एवं महालेखा परीक्षक की रिपोर्ट देता है।
(3) He causes to be laid before the Parliament the annual financial statement. वह संसद के वार्षिक वित्तीय विवरण के समक्ष रखे जाने का कारण बनता है।

Code :-
A. 1, 3
B. 1, 2
C. 2, 3
D. All of the above

(A) 1, 3
Explanation :-
The financial powers and functions of the President are :-
Money bills can be introduced in the Parliament only with his prior recommendation.
He causes to be laid before the Parliament the annual financial statement (ie, the Union Budget).
No demand for a grant can be made except on his recommendation.
He can make advances out of the contingency fund of India to meet any unforeseen expenditure.
He constitutes a finance commission after every five years to recommend the distribution of revenues between the Centre and the states.
राष्ट्रपति की वित्तीय शक्तियाँ और कार्य हैं: -
धन विधेयक संसद में उसकी पूर्व सिफारिश के साथ ही प्रस्तुत किए जा सकते हैं।
वह संसद के समक्ष वार्षिक वित्तीय विवरण (अर्थात केंद्रीय बजट) के समक्ष रखे जाने का कारण बनता है।
उसकी सिफारिश के अलावा अनुदान की कोई मांग नहीं की जा सकती।
वह किसी भी अप्रत्याशित व्यय को पूरा करने के लिए भारत के आकस्मिक निधि से अग्रिम कर सकता है।
वह केंद्र और राज्यों के बीच राजस्व के वितरण की सिफारिश करने के लिए हर पांच साल के बाद एक वित्त आयोग का गठन करता है।

124.  Which of the following is/are true? निम्नलिखित में से कौन-सा सत्य है / हैं?
(1) Just like an ordinance cannot amend the Constitution, it cannot alter or amend a tax law. जैसे अध्यादेश संविधान में संशोधन नहीं कर सकता, वह कर कानून में बदलाव या संशोधन नहीं कर सकता।
(2) Successive re-promulgation of ordinances with the same text without any attempt to get the bills passed by the assembly would amount to violation of the Constitution. विधानसभा द्वारा पारित बिलों को प्राप्त करने के लिए बिना किसी प्रयास के एक ही पाठ के साथ अध्यादेशों का फिर से प्रचार संविधान के उल्लंघन की राशि होगी।

Code :-
A. Only 1
B. Only 2
C. Both 1 and 2
D. Neither 1 nor 2

(B) Only 2
Explanation :-
An ordinance like any other legislation, can be retrospective, that is, it may come into force from a back date.
It may modify or repeal any act of Parliament or another ordinance. It can alter or amend a tax law also. However, it cannot be issued to amend the Constitution.
The Supreme Court ruled that successive re-promulgation of ordinances with the same text without any attempt to get the bills passed by the assembly would amount to violation of the Constitution and the ordinance so re-promulgated is liable to be struck down. It held that the exceptional power of law-making through ordinance cannot be used as a substitute for the legislative power of the state legislature.
किसी भी अन्य कानून की तरह अध्यादेश, पूर्वव्यापी हो सकता है, अर्थात यह एक पिछली तारीख से लागू हो सकता है।
यह संसद या किसी अन्य अध्यादेश के किसी भी अधिनियम को संशोधित या निरस्त कर सकता है। यह कर कानून में भी बदलाव या संशोधन कर सकता है। हालाँकि, यह संविधान में संशोधन करने के लिए जारी नहीं किया जा सकता है।
उच्चतम न्यायालय ने फैसला दिया कि विधानसभा द्वारा पारित बिलों को प्राप्त करने के लिए बिना किसी प्रयास के एक ही पाठ के साथ अध्यादेशों का क्रमिक पुनर्मूल्यांकन संविधान के उल्लंघन के लिए होगा और अध्यादेश को फिर से लागू करने के लिए उत्तरदायी है। यह माना गया कि अध्यादेश के माध्यम से कानून बनाने की असाधारण शक्ति का इस्तेमाल राज्य विधानमंडल की विधायी शक्ति के विकल्प के रूप में नहीं किया जा सकता है।

125. Which of the following is/are true? निम्नलिखित में से कौन-सा सत्य है / हैं?
(1) Ordinance ceases to operate on the expiry of two months from the reassembly of Parliament. अध्यादेश संसद के पुनर्मूल्यांकन से दो महीने की समाप्ति पर संचालित होता है।
(2) The maximum life of an ordinance can be six months. अध्यादेश का अधिकतम जीवन छह महीने हो सकता है।

Code :-
A. Only 1
B. Only 2
C. Both 1 and 2
D. Neither 1 nor 2

(D) Neither 1 nor 2
Explanation :-
Every ordinance issued by the President during the recess of Parliament must be laid before both the Houses of Parliament when it reassembles. If the ordinance is approved by both the Houses, it becomes an act. If Parliament takes no action at all, the ordinance ceases to operate on the expiry of six weeks from the reassembly of Parliament.
The ordinance may also cease to operate even earlier than the prescribed six weeks, if both the Houses of Parliament pass resolutions disapproving it.
If the Houses of Parliament are summoned to reassemble on different dates, the period of six weeks is calculated from the later of those dates. This means that the maximum life of an ordinance can be six months and six weeks, in case of non-approval by the Parliament (six months being the maximum gap between the two sessions of Parliament).
संसद के अवकाश के दौरान राष्ट्रपति द्वारा जारी किए गए प्रत्येक अध्यादेश को संसद के दोनों सदनों के समक्ष रखा जाना चाहिए, जब यह आश्वस्त हो। यदि अध्यादेश को दोनों सदनों द्वारा अनुमोदित किया जाता है, तो यह एक अधिनियम बन जाता है। यदि संसद कोई कार्रवाई नहीं करती है, तो अध्यादेश संसद के पुनर्मूल्यांकन से छह सप्ताह की समाप्ति पर काम करना बंद कर देता है।
यदि संसद के दोनों सदनों ने इसे अस्वीकार कर दिया, तो अध्यादेश निर्धारित छह सप्ताह से पहले भी काम करना बंद कर सकता है।
यदि संसद के सदनों को अलग-अलग तारीखों पर फिर से इकट्ठा करने के लिए बुलाया जाता है, तो छह सप्ताह की अवधि की गणना उन तारीखों के बाद से की जाती है। इसका मतलब यह है कि संसद द्वारा गैर-अनुमोदन के मामले में अध्यादेश का अधिकतम जीवन छह महीने और छह सप्ताह हो सकता है (संसद के दो सत्रों के बीच छह महीने का अधिकतम अंतराल)।

126. Which of the following is/are true? निम्नलिखित में से कौन-सा सत्य है / हैं?
(1) The new IPR policy does not see IPRs as a marketable financial asset. नई IPR नीति IPRs को विपणन योग्य वित्तीय संपत्ति के रूप में नहीं देखती है।
(2) The new policy makes no changes to the sections which prevent ever-greening of drug patents and grants compulsory licenses.  नई नीति से उन वर्गों में कोई बदलाव नहीं होता है जो ड्रग पेटेंट और अनुदान के अनिवार्य लाइसेंस के कभी भी हरियाली को रोकते हैं।

Code :-
A. Only 1
B. Only 2
C. Both 1 and 2
D. Neither 1 nor 2

(A) Only 1
Explanation :-
The new policy aims to pushes IPRs as a marketable financial asset, promote innovation and entrepreneurship, while protecting public interest.
It will ensure availability of essential and life-saving drugs at affordable prices.
The policy also states that India shall remain committed to WTO’s Doha Declaration on WTO’s TRIPS Agreement and Public Health.
नई नीति का उद्देश्य सार्वजनिक हित की रक्षा करते हुए, आईपीआर को एक विपणन योग्य वित्तीय संपत्ति के रूप में स्थापित करना, नवाचार और उद्यमिता को बढ़ावा देना है।
यह सस्ती कीमतों पर आवश्यक और जीवनरक्षक दवाओं की उपलब्धता सुनिश्चित करेगा।
नीति में यह भी कहा गया है कि भारत डब्ल्यूटीओ के ट्रिप्स समझौते और सार्वजनिक स्वास्थ्य पर डब्ल्यूटीओ के दोहा घोषणा के लिए प्रतिबद्ध रहेगा।

127. As per proposers of decriminalizing law on defamation, which article is attacked by the law on criminal defamation? मानहानि पर कानून को कमजोर करने वाले प्रस्तावकों के अनुसार, आपराधिक मानहानि पर कानून द्वारा किस अनुच्छेद पर हमला किया जाता है?
(1) 19
(2) 21
(3) 13

Code :-
A. 2, 3
B. 1, 2
C. 1
D. All of the above

(B) 1, 2
Explanation :-
The SC noted that reputation of an individual is an equally important right and stood on the same pedestal as free speech.
The criminalising defamation does not attack freedom of speech and expression.
One has a right to freedom of speech and expression. One is also required to maintain the idea of fraternity that assures the dignity of the individual.
SC ने कहा कि किसी व्यक्ति की प्रतिष्ठा एक समान महत्वपूर्ण अधिकार है और स्वतंत्र भाषण के रूप में एक ही पीठ पर खड़ा है।
आपराधिक मानहानि बोलने और अभिव्यक्ति की स्वतंत्रता पर हमला नहीं करता है।
किसी को बोलने और अभिव्यक्ति की स्वतंत्रता का अधिकार है। भाईचारे के विचार को बनाए रखने के लिए भी एक की आवश्यकता होती है जो व्यक्ति की गरिमा को सुनिश्चित करता है।

128. Which sections of Macaulay’s IPC relate to the law on criminal defamation? मैकाले के आईपीसी के कौन से वर्ग आपराधिक मानहानि पर कानून से संबंधित हैं?
A. 65, 67, 69
B. 499 and 500
C. 512, 515, 516
D. 123 and 124

(B) 499 and 500
Explanation :-
Sections 499 and 500 of Macaulay’s Indian Penal Code of 1860 prescribes two years’ imprisonment for a person found guilty of defamation.
The SC upheld the constitutional validity of this law recently.
1860 के मैकाले के भारतीय दंड संहिता की धारा 499 और 500 में मानहानि का दोषी पाए गए व्यक्ति के लिए दो साल की कैद का प्रावधान है।
SC ने हाल ही में इस कानून की संवैधानिक वैधता को बरकरार रखा।

129.  Which of the following is/are true? निम्नलिखित में से कौन-सा सत्य है / हैं?
(1) Now foreign firms cannot fund political parties and NGOs in India. अब विदेशी फर्म भारत में राजनीतिक दलों और गैर सरकारी संगठनों को निधि नहीं दे सकते।
(2) The Representation of People Act bars political parties from receiving foreign funds. जनप्रतिनिधि अधिनियम राजनीतिक दलों को विदेशी धन प्राप्त करने से रोक देता है।

Code :-
A. Only 1
B. Only 2
C. Both 1 and 2
D. Neither 1 nor 2

(B) Only 2
Explanation :-
The amended Foreign Contribution Regulation Act (FCRA), 2010 which was brought in through the Finance Bill route, will help foreign origin companies to fund NGOs here as well has enable them to give donations to political parties.
Such funding will now bypass government scrutiny.
संशोधित विदेशी अंशदान विनियमन अधिनियम (FCRA), 2010 जो वित्त विधेयक मार्ग के माध्यम से लाया गया था, विदेशी मूल कंपनियों को गैर-सरकारी संगठनों को निधि देने में मदद करेगा और साथ ही उन्हें राजनीतिक दलों को दान देने में सक्षम बनाता है।
इस तरह की फंडिंग अब सरकारी जांच को दरकिनार कर देगी।

130.  How many states in India have a bicameral legislature? भारत में कितने राज्यों में द्विसदनीय विधायिका है?
A. 7
B. 5
C. 6
D. 9

(A) 7
Explanation :-
Seven Indian States, Andhra Pradesh, Telangana, Bihar, Jammu-Kashmir, Karnataka, Maharashtra and Uttar Pradesh, have bicameral Legislatures.
The newly formed Telangana has 40 members.
सात भारतीय राज्यों, आंध्र प्रदेश, तेलंगाना, बिहार, जम्मू-कश्मीर, कर्नाटक, महाराष्ट्र और उत्तर प्रदेश में द्विसदनीय विधानसभाएं हैं।
नवगठित तेलंगाना में 40 सदस्य हैं।

131. The actual strength of a Legislative Council is fixed by :- विधान परिषद की वास्तविक शक्ति किसके द्वारा तय की जाती है: -
A. State Government राज्य सरकार
B. Constitution of India भारत का संविधान
C. The Parliament संसद
D. Election Commission चुनाव आयोग

(C) The Parliament संसद
Explanation :-
Though the Constitution has fixed the maximum and the minimum limits, the actual strength of a Council is fixed by Parliament. हालांकि संविधान ने अधिकतम और न्यूनतम सीमा तय कर दी है, लेकिन संसद द्वारा एक परिषद की वास्तविक ताकत तय की जाती है।

132. The smallest Legislative Council in terms of numbers is of :- संख्या की दृष्टि से सबसे छोटी विधान परिषद निम्नलिखित में से है: -
A. Jammu and Kashmir
B. Bihar
C. Karnataka
D. Telangana

(A) Jammu and Kashmir
Explanation :-
The maximum strength of the council is fixed at one-third of the total strength of the assembly and the minimum strength is fixed at 40.
However Jammu and Kashmir is an exception to this as this rule does not apply to it. It has 36 members in its Council
It means that the size of the council depends on the size of the assembly of the concerned state. This is done to ensure the predominance of the directly elected House (assembly) in the legislative affairs of the state.
परिषद की अधिकतम ताकत विधानसभा की कुल ताकत का एक तिहाई और न्यूनतम ताकत 40 पर तय की गई है।
हालाँकि, जम्मू और कश्मीर इसके लिए एक अपवाद है क्योंकि यह नियम इस पर लागू नहीं होता है। इसकी परिषद में 36 सदस्य हैं
इसका अर्थ है कि परिषद का आकार संबंधित राज्य की विधानसभा के आकार पर निर्भर करता है। यह राज्य के विधायी मामलों में सीधे निर्वाचित सदन (विधानसभा) की प्रधानता सुनिश्चित करने के लिए किया जाता है।

133. President’s Rule :- राष्ट्रपति शासन :-
A. Can be revoked by the President anytime without Parliament’s Approval. संसद की स्वीकृति के बिना कभी भी राष्ट्रपति द्वारा रद्द किया जा सकता है।
B. Can be revoked by the President on suggestion of the Election Commission. चुनाव आयोग के सुझाव पर राष्ट्रपति द्वारा निरस्त किया जा सकता है।
C. Can be revoked by the President anytime with Parliament’s Approval. संसद की स्वीकृति के साथ राष्ट्रपति द्वारा कभी भी निरस्त किया जा सकता है।
D. Can be revoked with the Parliament passing such a proclamation making it binding for the President to revoke the rule. इस तरह की उद्घोषणा को पारित करने के लिए संसद को रद्द किया जा सकता है, जिससे राष्ट्रपति शासन को रद्द कर सकते हैं।

(A) Can be revoked by the President anytime without Parliament’s Approval. संसद की स्वीकृति के बिना कभी भी राष्ट्रपति द्वारा निरस्त किया जा सकता है।
Explanation :-
A proclamation of President’s Rule may be revoked by the President at any time by a subsequent proclamation. Such a proclamation does not require the parliamentary approval.
Since the President is supposed to act as per aid and advice of the Union Council of ministers, it is they who suggest for imposing or removal of the President’s rule to the President.
राष्ट्रपति के नियम की घोषणा बाद में किसी भी समय राष्ट्रपति द्वारा रद्द की जा सकती है। इस तरह की उद्घोषणा को संसदीय अनुमोदन की आवश्यकता नहीं होती है।
चूँकि राष्ट्रपति को केंद्रीय मंत्रिपरिषद की सहायता और सलाह के अनुसार कार्य करना चाहिए, यह वह है जो राष्ट्रपति शासन लागू करने या हटाने का सुझाव राष्ट्रपति को देता है।

134.  Who of the following participate in election of members to Legislative Council? निम्नलिखित में से कौन विधान परिषद के सदस्यों के चुनाव में भाग लेता है?
(1) Graduates
(2) Teachers
(3) Members of District Boards

Code :-
A. 2, 3
B. 1, 3
C. 1, 2
D. All of the above

(D) All of the above
Explanation :-
Of the total number of members of a legislative council :-
1/3 are elected by the members of local bodies in the state like municipalities, district boards, etc.
1/12 are elected by graduates of three years standing and residing within the state,
1/12 are elected by teachers of three years standing in the state, not lower in standard than secondary school,
1/3 are elected by the members of the legislative assembly of the state from amongst persons who are not members of the assembly, and
The remainder are nominated by the governor from amongst persons who have a special knowledge or practical experience of literature, science, art, cooperative movement and social service.
एक विधान परिषद के कुल सदस्यों की संख्या: -
1/3 राज्य में स्थानीय निकायों के सदस्यों द्वारा चुने जाते हैं जैसे नगरपालिका, जिला बोर्ड आदि।
1/12 राज्य के भीतर खड़े और निवास करने वाले तीन साल के स्नातकों द्वारा चुने जाते हैं,
1/12 राज्य में तीन साल के शिक्षकों द्वारा चुने जाते हैं, माध्यमिक विद्यालय की तुलना में मानक से कम नहीं,
1/3 राज्य के विधान सभा के सदस्यों द्वारा उन व्यक्तियों में से चुने जाते हैं जो विधानसभा के सदस्य नहीं हैं, और
शेष लोगों को राज्यपाल द्वारा साहित्य, विज्ञान, कला, सहकारी आंदोलन और सामाजिक सेवा के विशेष ज्ञान या व्यावहारिक अनुभव वाले व्यक्तियों द्वारा नामित किया जाता है।

135. What is the task of Gate Mitras?
A. Man unmanned rail level crossings. मानव रहित रेल लेवल क्रॉसिंग।
B. Help passengers with automated ticketing booths. स्वचालित टिकटिंग बूथ वाले यात्रियों की सहायता करें।
C. Regulate the proper entry and exit of passengers from trains to avoid accidents.  दुर्घटनाओं से बचने के लिए गाड़ियों से यात्रियों के उचित प्रवेश और निकास को विनियमित करें।
D. All of the above

(A) Man unmanned rail level crossings. मानव रहित रेल स्तरीय क्रॉसिंग
Explanation :-
As a precursor to removing all the unmanned level crossings (UMLCs) across the country by 2020, a beginning is being made by deploying a ‘gate mitras’ - literally friend at the gate- at the crossings.
This is to avoid accidents at unmanned level crossings. The railways would provide them with a uniform, along with a high-power light, a flag and a whistle to enable them warn oncoming road traffic of an approaching train.
2020 तक देश भर में सभी मानवरहित स्तर की क्रॉसिंग (UMLCs) को हटाने के लिए एक अग्रदूत के रूप में, एक शुरुआत एक ras गेट माइट्रास ’की तैनाती के द्वारा की जा रही है - जिसका अर्थ है गेट पर- क्रॉसिंग पर दोस्त।
यह मानव रहित स्तर के क्रॉसिंग पर दुर्घटनाओं से बचने के लिए है। रेलवे उन्हें एक वर्दी, एक उच्च शक्ति प्रकाश, एक झंडा और एक सीटी के साथ प्रदान करेगा ताकि उन्हें आने वाली ट्रेन के आने वाले सड़क यातायात को चेतावनी दी जा सके।

136. There is a new division for Centre-State Relations in MEA. It is called :- एमईए में केंद्र-राज्य संबंधों के लिए एक नया विभाजन है। यह कहा जाता है :-
A. States and UTs
B. States and UTs Division
C. Federal Division संघीय प्रभाग
D. States Division

(D) States Division
Explanation :-
Ministry of External Affairs has set up a new division – “States Division” - in the Ministry in October 2014 to coordinate with States and Union Territories for further facilitation of their efforts to promote their exports and tourism and attract more overseas investments and expertise.
The functions of the States Division are to coordinate facilitation of efforts of states in promoting their exports, tourism and attract more overseas investment and expertise.
The objective of the Division is to coordinate facilitation of efforts in these areas between our Mission/Post(s) and State/Union Territories Governments as well as foreign diplomatic and trade missions in India.
It aims to assist States and Union Territories by sharing with them, where required, this Ministry's experience and expertise through training and capacity building in areas relating to external linkages relating to trade, investment, cultural and other such areas.
विदेश मंत्रालय ने अपने निर्यात और पर्यटन को बढ़ावा देने और अपने विदेशी निवेश और विशेषज्ञता को आकर्षित करने के लिए अपने प्रयासों को और अधिक सुविधाजनक बनाने के लिए राज्यों और केंद्र शासित प्रदेशों के साथ समन्वय करने के लिए अक्टूबर 2014 में मंत्रालय में एक नया डिवीजन - "स्टेट्स डिवीजन" स्थापित किया है।
स्टेट्स डिवीजन के कार्य अपने निर्यात, पर्यटन को बढ़ावा देने और अधिक विदेशी निवेश और विशेषज्ञता को आकर्षित करने के लिए राज्यों के प्रयासों की सुविधा का समन्वय करना है।
प्रभाग का उद्देश्य हमारे मिशन / पोस्ट (एस) और राज्य / केंद्र शासित प्रदेश सरकारों के साथ-साथ भारत में विदेशी राजनयिक और व्यापार मिशनों के बीच इन क्षेत्रों में प्रयासों की सुविधा का समन्वय करना है।
इसका उद्देश्य राज्यों, केंद्रशासित प्रदेशों को उनके साथ साझा करके सहायता करना है, जहां आवश्यक हो, इस मंत्रालय के अनुभव और विशेषज्ञता व्यापार, निवेश, सांस्कृतिक और अन्य ऐसे क्षेत्रों से संबंधित बाहरी संपर्कों से संबंधित क्षेत्रों में प्रशिक्षण और क्षमता निर्माण के माध्यम से।

137. Which of the following laws mandate only a year’s judicial separation before applying for dissolution of marriage? निम्नलिखित में से कौन सा कानून विवाह के विघटन के लिए आवेदन करने से पहले केवल एक वर्ष के न्यायिक पृथक्करण को अनिवार्य करता है?
(1) The Hindu Marriage Act
(2) Parsi Marriage and Divorce Act
(3) Christian Law

Code :-
A. 2, 3
B. 1, 3
C. 1, 2
D. All of the above

(C) 1, 2
Explanation :-
The Hindu Marriage Act, the Parsi Marriage and Divorce Act and even the Special Marriage Act mandate only a year’s judicial separation.
The government is going ahead with a proposal to amend a condition in a 147-year-old divorce law mandating Christian couples to live separately for two years before they can apply for dissolution of marriage. The Centre’s move is triggered by the Supreme Court’s comments that this condition, called judicial separation, “makes no sense” today.
The Law and Justice Ministry is proposing to amend the Divorce Act, 1869 to reduce by half — from two years to one — the waiting period for Christians who have already decided to divorce.
हिंदू विवाह अधिनियम, पारसी विवाह और तलाक अधिनियम और यहां तक ​​कि विशेष विवाह अधिनियम में केवल एक वर्ष का न्यायिक पृथक्करण है।
सरकार 147 साल पुराने तलाक कानून में एक शर्त में संशोधन करने के प्रस्ताव पर आगे बढ़ रही है ताकि ईसाई जोड़ों को शादी के विघटन के लिए आवेदन करने से पहले दो साल तक अलग रह सकें। सेंट्रे के इस कदम से सुप्रीम कोर्ट की टिप्पणी से भड़की है कि इस स्थिति को, न्यायिक पृथक्करण, "आज कोई मतलब नहीं है" कहा जाता है।
कानून और न्याय मंत्रालय तलाक अधिनियम, 1869 को आधे से कम करने के लिए संशोधन कर रहा है - दो साल से एक तक - उन ईसाइयों के लिए प्रतीक्षा अवधि जो पहले ही तलाक का फैसला कर चुके हैं।

138.  If the President’s Rule in a state is temporarily revoked for a floor test, which of the following sentence is true? यदि किसी राज्य में राष्ट्रपति शासन अस्थायी रूप से फ्लोर टेस्ट के लिए निरस्त किया जाता है, तो निम्नलिखित में से कौन सा वाक्य सही है?
A. The previous Chief Minister will not be restored as soon as the President’s Rule is revoked (even if temporarily) as he is yet to prove his numbers. पिछले मुख्यमंत्री को बहाल नहीं किया जाएगा क्योंकि राष्ट्रपति के नियम को रद्द कर दिया गया है (भले ही अस्थायी रूप से) क्योंकि वह अभी तक अपनी संख्या साबित करने के लिए नहीं है।
B. The previous Chief Minister will be restored as soon as the President’s Rule is revoked (even if temporarily) even though floor test is yet to be completed. पूर्व मुख्यमंत्री को बहाल किया जाएगा जैसे ही राष्ट्रपति के नियम को रद्द कर दिया जाता है (भले ही अस्थायी रूप से) भले ही मंजिल परीक्षण पूरा होना बाकी है।
C. The cabinet is dissolved completely कैबिनेट पूरी तरह से भंग है
D. None of the above is true उपरोक्त में से कोई भी सत्य नहीं है
Answer  Explanation
ANSWER: The previous Chief Minister will not be restored as soon as the President’s Rule is revoked (even if temporarily) as he is yet to prove his numbers.

Explanation :-
The previous CM would not be automatically restored as Chief Minister when President’s rule is lifted for the duration of the floor test. This is because the test is for him/her to prove his/her majority.
In case former Chief Minister is unable to prove his majority on the floor of the Assembly, the baton would go to the Governor to invite the leader of the majority group to form a new government.
The floor test can also be held for a limited purpose like allowing a chance to the previous Chief Minister to prove his majority in the House. It can also be a “composite” one between two rival parties.
फ्लोर टेस्ट की अवधि के लिए राष्ट्रपति शासन हटाए जाने पर पूर्व मुख्यमंत्री को मुख्यमंत्री के रूप में स्वचालित रूप से बहाल नहीं किया जाएगा। ऐसा इसलिए है क्योंकि परीक्षण उसके / उसके बहुमत साबित करने के लिए है।
यदि पूर्व मुख्यमंत्री विधानसभा के पटल पर अपना बहुमत साबित नहीं कर पाता है, तो बैटन राज्यपाल के पास बहुमत समूह के नेता को आमंत्रित करने के लिए एक नई सरकार बनाने के लिए जाएगा।
पिछले मुख्यमंत्री को सदन में बहुमत साबित करने का मौका देने जैसे सीमित उद्देश्य के लिए भी फ्लोर टेस्ट आयोजित किया जा सकता है। यह दो प्रतिद्वंद्वी दलों के बीच एक "समग्र" एक भी हो सकता है।

139.   "Status quo ante" in law and polity means? कानून और विनम्रता में "यथास्थिति" का मतलब है?
A. State of things subsequent to the conflict संघर्ष के बाद की चीजों की स्थिति
B. The way things were before जिस तरह से चीजें पहले थीं
C. A new state of polity एक नई राजव्यवस्था
D. State of things as prescribed by the judiciary न्यायपालिका द्वारा निर्धारित चीजों की स्थिति

(B) The way things were before जिस तरह से चीजें पहले थीं
Explanation :-
Status quo ante is Latin for "the way things were before" and incorporates the term status quo.
In law, it refers to the objective of a temporary restraining order or a rescission in which the situation is restored to "the state in which previously" it existed.
यथास्थिति "जिस तरह से चीजों से पहले थे" के लिए लैटिन है और यथास्थिति शब्द को शामिल करता है।
कानून में, यह एक अस्थायी निरोधक आदेश या एक बचाव के उद्देश्य को संदर्भित करता है जिसमें स्थिति "जिस राज्य में पहले मौजूद थी" को बहाल किया जाता है।

140. Which of the schemes have focus or targeted interventions for girls and women? लड़कियों और महिलाओं के लिए किस योजना पर ध्यान केंद्रित या लक्षित हस्तक्षेप है?
(1) Saakshar Bharat साक्षार भारत
(2) Sarva Shiksha Abhiyan सर्व शिक्षा अभियान
(3) Kasturba Gandhi Balika Vidyalayas कस्तूरबा गांधी बालिका विद्यालय

Code :-
A. 1, 3
B. 2, 3
C. 1, 2
D. All of the above

(D) All of the above
Explanation :-
In order to improve the female literacy rate in the country, “Saakshar Bharat”, a Centrally Sponsored Scheme is being implemented in rural areas of States and UT that had adult female literacy rate of 50 per cent and below as per Census 2001, including left wing extremism affected districts, irrespective of their literacy rates, with special focus on women and other disadvantaged groups.
In order to ensure greater participation of girls in elementary education, Sarva Shiksha Abhiyan (SSA) has targeted interventions for girls which include opening of schools in the neighborhood to make access easier for girls, appointment of additional teachers including women teachers, separate toilets for girls, teachers’ sensitization programme to promote girls participation, gender sensitive teaching learning materials including textbooks.
In addition, Kasturba Gandhi Balika Vidyalayas have been opened in Educationally Backward Blocks where the female rural literacy is below the national average to provide for residential upper primary schools for girls.
Under Rashtriya Madhyamik Shiksha Abhiyan (RMSA) several provisions viz. construction of separate toilets for girls, construction of residential quarters for teachers, curriculum reforms, conducting self-defence training for the girls, special programme for empowerment of girls, stipend for girls with disabilities, vocationalisation of secondary education, girls hostel, etc. have been made for improving girls education and effectively addressing female literacy rate.
देश में महिला साक्षरता दर में सुधार के लिए, “भारतवर्ष”, राज्यों और केंद्र शासित प्रदेशों के ग्रामीण क्षेत्रों में एक केंद्र प्रायोजित योजना लागू की जा रही है, जिसमें वयस्क महिला साक्षरता दर 50 प्रतिशत थी और 2001 की जनगणना के अनुसार, इसमें सुधार भी शामिल था। महिलाओं और अन्य वंचित समूहों पर विशेष ध्यान देने के साथ, विंग अतिवाद प्रभावित जिलों, उनकी साक्षरता दर के बावजूद प्रभावित।
प्रारंभिक शिक्षा में लड़कियों की अधिक भागीदारी सुनिश्चित करने के लिए, सर्व शिक्षा अभियान (एसएसए) ने लड़कियों के लिए हस्तक्षेप को लक्षित किया है, जिसमें लड़कियों की पहुंच आसान बनाने के लिए, महिला शिक्षकों सहित अतिरिक्त शिक्षकों की नियुक्ति, लड़कियों के लिए अलग शौचालय की व्यवस्था शामिल है। , शिक्षक भागीदारी, लड़कियों की भागीदारी को बढ़ावा देने के लिए, पाठ्यपुस्तकों सहित लिंग संवेदनशील शिक्षण शिक्षण सामग्री।
इसके अलावा, कस्तूरबा गांधी बालिका विद्यालय शैक्षिक रूप से पिछड़े ब्लॉकों में खोले गए हैं जहाँ लड़कियों के लिए आवासीय उच्च प्राथमिक स्कूलों की व्यवस्था करने के लिए महिला ग्रामीण साक्षरता राष्ट्रीय औसत से कम है।
राष्ट्रीय मध्यम शिक्षा अभियान (RMSA) के तहत कई प्रावधान अर्थात। लड़कियों के लिए अलग शौचालय का निर्माण, शिक्षकों के लिए आवासीय क्वार्टर का निर्माण, पाठ्यक्रम सुधार, लड़कियों के लिए आत्मरक्षा प्रशिक्षण, लड़कियों के सशक्तीकरण के लिए विशेष कार्यक्रम, विकलांग लड़कियों के लिए वजीफा, माध्यमिक शिक्षा का व्यवसायीकरण, गर्ल्स हॉस्टल, आदि। लड़कियों की शिक्षा में सुधार और महिला साक्षरता दर को प्रभावी ढंग से संबोधित करने के लिए बनाया गया है।

141.  Smarac Citizen is an app developed for which of the following reasons? Smarac Citizen निम्नलिखित में से किस कारण से विकसित किया गया ऐप है?
A. To grant construction permission near monuments. स्मारकों के पास निर्माण अनुमति देने के लिए।
B. To help tourists as a guide for information about monuments. स्मारकों के बारे में जानकारी के लिए एक गाइड के रूप में पर्यटकों की मदद करना।
C. A map highlighting monuments for tourists. पर्यटकों के लिए स्मारकों को उजागर करने वाला एक मानचित्र।
D. None of the above उपरोक्त में से कोई नहीं

(A) To grant construction permission near monuments. स्मारकों के पास निर्माण अनुमति देने के लिए।
Explanation :-
Architects and real estate developers who plan to construct buildings close to national monuments can soon download a new mobile app named Smarac Citizen, which will grant them construction permits within a few minutes.
Based on a proposal from the National Monuments Authority (NMA), the app has been developed by the Indian Space Research Organisation (ISRO), containing the maps of 3,686 Centrally-protected monuments across the country.
Earlier, the NMA had to send teams to the areas close to historic monuments, where developers sought construction permits.
The teams had to ensure that permits were granted only to projects beyond 300 metres from the outer boundary of a monument.
Apart from easing the process of obtaining construction permits, the app will help the NMA keep a tab on encroachments around monuments.
The app will tell you how far new constructions are from any particular protected monument.
It will also give the elevation — the height of the site with respect to mean sea level. It will help architects not only for the NMA approval, but also for the approval of the Airports Authority of India.
आर्किटेक्ट और रियल एस्टेट डेवलपर्स जो राष्ट्रीय स्मारकों के करीब इमारतों का निर्माण करने की योजना बना रहे हैं, जल्द ही स्मार्क सिटीजन नाम से एक नया मोबाइल ऐप डाउनलोड कर सकते हैं, जो उन्हें कुछ ही मिनटों में निर्माण परमिट प्रदान करेगा।
राष्ट्रीय स्मारक प्राधिकरण (NMA) के एक प्रस्ताव के आधार पर, भारतीय अंतरिक्ष अनुसंधान संगठन (ISRO) द्वारा ऐप विकसित किया गया है, जिसमें देश भर में 3,686 केंद्र-संरक्षित स्मारकों के नक्शे हैं।
इससे पहले, NMA को ऐतिहासिक स्मारकों के करीब के क्षेत्रों में टीमों को भेजना था, जहां डेवलपर्स ने निर्माण परमिट की मांग की थी।
टीमों को यह सुनिश्चित करना था कि स्मारक की बाहरी सीमा से केवल 300 मीटर की दूरी तक की परियोजनाओं के लिए ही अनुमति दी गई थी।
निर्माण परमिट प्राप्त करने की प्रक्रिया को आसान बनाने के अलावा, एप्लिकेशन NMA को स्मारकों के आसपास के अतिक्रमणों पर एक टैब रखने में मदद करेगा।
ऐप आपको बताएगा कि किसी विशेष संरक्षित स्मारक से कितने नए निर्माण हैं।
यह समुद्र तल से संबंधित ऊंचाई के साथ साइट की ऊंचाई भी देगा। यह न केवल NMA अनुमोदन के लिए, बल्कि भारतीय विमानपत्तन प्राधिकरण के अनुमोदन के लिए भी वास्तुकारों की मदद करेगा।

142. Which is the most important place to determine the majority of a government? किसी सरकार का बहुमत निर्धारित करने के लिए सबसे महत्वपूर्ण स्थान कौन सा है?
A. Floor of the house
B. The state governor
C. The President and state governor
D. The Supreme Court of India

(D) Floor of the house
Explanation :-
Bommai case in 1994, made it clear that the only place to ascertain the majority of a government was the floor of the House.
All the other avenues are either secondary or means of redressal.
1994 में बोम्मई मामले ने, यह स्पष्ट कर दिया कि सरकार के बहुमत का पता लगाने का एकमात्र स्थान सदन का तल था।
अन्य सभी रास्ते या तो गौण हैं या निवारण के साधन हैं।

143. Which of the following is/are true? निम्नलिखित में से कौन-सा सत्य है / हैं?
(1) The court does not have the power to restore the state government to office in case it finds the proclamation of president’s rule to be unconstitutional. न्यायालय के पास राज्य सरकार को कार्यालय में बहाल करने की शक्ति नहीं है, क्योंकि वह राष्ट्रपति शासन की घोषणा को असंवैधानिक मानता है।
(2) If the Proclamation is invalid, it does not stand validated merely because it is approved of by the Parliament. यदि उद्घोषणा अमान्य है, तो यह केवल इसलिए मान्य नहीं है क्योंकि यह संसद द्वारा अनुमोदित है।

Code :-
A. Only 1
B. Only 2
C. Both 1 and 2
D. Neither 1 nor 2

(B) Only 2
Explanation :-
In the Bommai case, the Supreme Court held valid the power of the court to restore the government to office in case it finds the proclamation of president’s rule to be unconstitutional.
If the Proclamation is invalid, it does not stand validated merely because it is approved of by the Parliament. The grounds for challenging the validity of the Proclamation may be different from those challenging the validity of legislation. However, that does not make any difference to the vulnerability of the Proclamation on the limited grounds available. And therefore the validity of the Proclamation issued under Article 356(1) can be challenged even after it has been approved by both Houses of Parliament under Article 356(3).
बोम्मई मामले में, सर्वोच्च न्यायालय ने सरकार को पद पर बहाल करने के लिए अदालत की शक्ति को वैध ठहराया, यदि वह राष्ट्रपति शासन की घोषणा को असंवैधानिक मानती है।
यदि उद्घोषणा अमान्य है, तो यह मान्य नहीं है क्योंकि यह संसद द्वारा अनुमोदित है। उद्घोषणा की वैधता को चुनौती देने का आधार कानून की वैधता को चुनौती देने वालों से अलग हो सकता है। हालाँकि, इससे सीमित आधार पर उद्घोषणा की भेद्यता पर कोई फर्क नहीं पड़ता है। और इसलिए अनुच्छेद 356 (1) के तहत जारी उद्घोषणा की वैधता को संसद के दोनों सदनों द्वारा अनुच्छेद 356 (3) के तहत अनुमोदित किए जाने के बाद भी चुनौती दी जा सकती है।

144. Unnat Bharat Abhiyan is for :-
A. Adult literacy in rural areas. ग्रामीण क्षेत्रों में वयस्क साक्षरता।
B. Connecting higher educational institutions to communities. उच्च शिक्षण संस्थानों को समुदायों से जोड़ना।
C. Research in higher educational institutions. उच्च शिक्षण संस्थानों में अनुसंधान।
D. Use of technology in agriculture. कृषि में प्रौद्योगिकी का उपयोग।


(B) Connecting higher educational institutions to communities. उच्च शिक्षण संस्थानों को समुदायों से जोड़ना।
Explanation :-
The Ministry of Human Resource Development (MHRD) has launched a programme called Unnat Bharat Abhiyan with an aim to connect institutions of higher education, including Indian Institutes of Technology (IITs), National Institutes of Technology (NITs) and Indian Institutes of Science Education & Research (IISERs) etc. with local communities to address the development challenges through appropriate technologies.
The objectives of Unnat Bharat Abhiyan are broadly two-fold:
i) Building institutional capacity in Institutes of higher education in research & training relevant to the needs of rural India.
ii) Provide rural India with professional resource support from institutes of higher education, especially those which have acquired academic excellence in the field of Science, Engineering & Technology and Management.
मानव संसाधन विकास मंत्रालय (MHRD) ने भारतीय प्रौद्योगिकी संस्थानों (IIT), राष्ट्रीय प्रौद्योगिकी संस्थानों (NIT) और भारतीय विज्ञान शिक्षा संस्थानों सहित उच्च शिक्षा संस्थानों को जोड़ने के उद्देश्य से Unnat Bharat Abhiyan नामक एक कार्यक्रम शुरू किया है। उपयुक्त प्रौद्योगिकियों के माध्यम से विकास की चुनौतियों का समाधान करने के लिए स्थानीय समुदायों के साथ अनुसंधान (IISER) आदि।
उन्नाव भारत अभियान के उद्देश्य मोटे तौर पर दो गुना हैं:
i) ग्रामीण भारत की जरूरतों के लिए प्रासंगिक अनुसंधान और प्रशिक्षण में उच्च शिक्षा संस्थानों में संस्थागत क्षमता का निर्माण।
ii) उच्च शिक्षा के संस्थानों से पेशेवर संसाधन सहायता के साथ ग्रामीण भारत प्रदान करें, विशेष रूप से उन लोगों को जिन्होंने विज्ञान, इंजीनियरिंग और प्रौद्योगिकी और प्रबंधन के क्षेत्र में अकादमिक उत्कृष्टता हासिल की है।

145. Under which of the following situations a Rajya Sabha member vacates his/her seat? निम्नलिखित में से किस परिस्थिति में एक राज्यसभा सदस्य अपनी सीट खाली करता है?
(1) Resignation त्यागपत्र
(2) Disqualification on Ground of Defection अवज्ञा के आधार पर अयोग्यता
(3) Double Membership दोहरी सदस्यता

Code :-
A. 2, 3
B. 1, 2
C. 1, 3
D. All of the above

(D) All of the above उपरोक्त सभी
Explanation :-
In the following cases, a member of Parliament vacates his seat :-
Double Membership
Disqualifications of all types including defection
Resignation
Absence
Other cases A member has to vacate his seat in the Parliament:
(a) if his election is declared void by the court;
(b) if he is expelled by the House;
(c) if he is elected to the office of President or Vice-President; and
(d) if he is appointed to the office of governor of a state.
निम्नलिखित मामलों में, संसद का सदस्य अपनी सीट खाली करता है: -
डबल सदस्यता
दलबदल सहित सभी प्रकार की अयोग्यता
इस्तीफा
अनुपस्थिति
अन्य मामलों में एक सदस्य को संसद में अपनी सीट खाली करनी होती है:
(क) यदि न्यायालय द्वारा उसका चुनाव शून्य घोषित किया जाता है;
(ख) यदि वह सदन द्वारा निष्कासित किया जाता है;
(ग) यदि वह राष्ट्रपति या उपराष्ट्रपति के पद के लिए चुना जाता है; तथा
(घ) यदि वह किसी राज्य के राज्यपाल के पद पर नियुक्त किया जाता है।

146. Which of the following states use Hindi along with English in their High Courts for proceedings as well as judgements, etc.? निम्नलिखित में से कौन-सा राज्य अपने उच्च न्यायालयों में अंग्रेजी के साथ-साथ कार्यवाही और निर्णयों के लिए हिंदी का उपयोग करता है?
(1) Rajasthan
(2) New Delhi
(3) Bihar

Code :-
A. 1, 2
B. 1, 3
C. 2, 3
D. All of the above

(B) 1, 3
Explanation :-
The provision of optional use of Hindi in proceedings has already been made in the High Courts of Rajasthan, Uttar Pradesh, Madhya Pradesh and Bihar.
The Official Language Act of 1963 lays down that Hindi translation of acts, ordinances, orders, regulations and bye-laws published under the authority of the president are deemed to be authoritative texts. Further, every bill introduced in the Parliament is to be accompanied by a Hindi translation.
Similarly, there is to be a Hindi translation of state acts or ordinances in certain cases.
The act also enables the governor of a state, with the previous consent of the president, to authorise the use of Hindi or any other official language of the state for judgements, decrees and orders passed by the high court of the state but they should be accompanied by an English translation.
राजस्थान, उत्तर प्रदेश, मध्य प्रदेश और बिहार के उच्च न्यायालयों में कार्यवाही में हिंदी के वैकल्पिक उपयोग का प्रावधान पहले ही किया जा चुका है।
1963 का राजभाषा अधिनियम यह कहता है कि राष्ट्रपति के अधिकार के तहत प्रकाशित कृत्यों, अध्यादेशों, आदेशों, विनियमों और उपनियमों का हिंदी अनुवाद आधिकारिक ग्रंथ माना जाता है। इसके अलावा, संसद में पेश किए जाने वाले हर बिल का हिंदी अनुवाद करना होता है।
इसी तरह, कुछ मामलों में राज्य कृत्यों या अध्यादेशों का हिंदी अनुवाद होना है।
अधिनियम राज्य के राज्यपाल को राष्ट्रपति की पिछली सहमति के साथ, राज्य के उच्च न्यायालय द्वारा पारित निर्णयों, आदेशों और आदेशों के लिए हिंदी या राज्य की किसी अन्य आधिकारिक भाषा के उपयोग को अधिकृत करने के लिए भी सक्षम बनाता है, लेकिन उन्हें होना चाहिए एक अंग्रेजी अनुवाद के साथ।

147. Which of the following is/are true? निम्नलिखित में से कौन सा सत्य है / हैं?
(1) The Governor along with the Chief Justice of India can authorize the use of the Hindi language in the proceedings of any High Court. भारत के मुख्य न्यायाधीश के साथ राज्यपाल किसी भी उच्च न्यायालय की कार्यवाही में हिंदी भाषा के प्रयोग को अधिकृत कर सकते हैं।
(2) The Supreme Court hears only those petitions or appeals which are made in English. सर्वोच्च न्यायालय केवल उन याचिकाओं या अपीलों को सुनता है जो अंग्रेजी में की जाती हैं।

Code :-
A. Only 1
B. Only 2
C. Both 1 and 2
D. Neither 1 nor 2

(B) Only 2
Explanation ;-
Article 348 (1) of the Constitution of India provides that all proceedings in the Supreme Court and in every High court shall be in English Language until Parliament by law otherwise provides.
Under Article 348 (2), the Governor of the State may, with the previous consent of the President, authorize the use of the Hindi language or any other language used for any official purpose of the State, in the proceedings of the High Court having its principal seat in that State provided that decrees, judgments or orders passed by such High Courts shall be in English.
Section 7 of the Official Languages Act, 1963, provides that the use of Hindi or official language of a State in addition to the English language may be authorized, with the consent of the President of India, by the Governor of the State for purpose of judgments, etc. made by the High Court for that State.
Government had taken up with the Supreme Court of India the proposals of the Governments of Chhattisgarh, Gujarat, Tamil Nadu and Karnataka seeking consent of the President of India for allowing regional languages in the proceedings of their High Courts. However, the Full Court of the Supreme Court disapproved these proposals.
भारत के संविधान के अनुच्छेद 348 (1) में यह प्रावधान है कि सर्वोच्च न्यायालय और प्रत्येक उच्च न्यायालय में सभी कार्यवाही अंग्रेजी भाषा में होगी जब तक कि संसद कानून द्वारा अन्यथा प्रदान नहीं करती है।
अनुच्छेद 348 (2) के तहत, राज्य के राज्यपाल, राष्ट्रपति की पिछली सहमति के साथ, उच्च न्यायालय की कार्यवाही में हिंदी भाषा या राज्य के किसी भी आधिकारिक उद्देश्य के लिए इस्तेमाल की जाने वाली किसी अन्य भाषा को अधिकृत कर सकते हैं। उस राज्य में इसकी प्रमुख सीट, बशर्ते कि ऐसे उच्च न्यायालयों द्वारा पारित निर्णय, निर्णय या आदेश अंग्रेजी में होंगे।
आधिकारिक भाषा अधिनियम, 1963 की धारा 7 में यह प्रावधान है कि अंग्रेजी भाषा के अलावा किसी राज्य की हिंदी या आधिकारिक भाषा का उपयोग भारत के राष्ट्रपति की सहमति से, राज्य के राज्यपाल द्वारा किया जा सकता है। उस राज्य के लिए उच्च न्यायालय द्वारा निर्णय, आदि।
सरकार ने भारत के सर्वोच्च न्यायालय के साथ छत्तीसगढ़, गुजरात, तमिलनाडु और कर्नाटक की सरकारों के प्रस्तावों को अपने उच्च न्यायालयों की कार्यवाही में क्षेत्रीय भाषाओं को अनुमति देने के लिए भारत के राष्ट्रपति की सहमति लेने की मांग की थी। हालांकि, सुप्रीम कोर्ट के पूर्ण न्यायालय ने इन प्रस्तावों को अस्वीकार कर दिया।

148. Which is the first Municipal Council to attain the status of "integrated with digital locker" for the issuance of all essential Documents? सभी आवश्यक दस्तावेजों को जारी करने के लिए "डिजिटल लॉकर के साथ एकीकृत" की स्थिति प्राप्त करने वाली पहली नगर परिषद कौन-सी है?
A. Rahuri
B. Pune
C. New Delhi
D. Solapur

(A) Rahuri
Explanation :-
Rahuri Municipal Council of Ahmednagar district in Maharashtra today became the first Municipal Council to attain the status of integrated with digital locker for the issuance of all essential Documents. This service here is a part of Digital India Programme.
The Website is integrated as an issuer and Requester with DigiLocker where the Citizen will have access to all these documents at any time.
They will have freedom to access them from anywhere in the world at any time. The citizen of the area can now apply for the certificates like Birth, Marriage and Death Certificates online without going to Municipal Council and these documents will be available in the Digilocker signed to the citizens.
This will also ensure them the peace of mind provided by the easy security of their documents forever.
महाराष्ट्र में अहमदनगर जिले की राहुरी नगर परिषद आज सभी आवश्यक दस्तावेज जारी करने के लिए डिजिटल लॉकर के साथ एकीकृत की स्थिति प्राप्त करने वाली पहली नगर परिषद बन गई। यहां यह सेवा डिजिटल इंडिया प्रोग्राम का एक हिस्सा है।
वेबसाइट डिजीलॉकर के साथ एक जारीकर्ता और अनुरोधकर्ता के रूप में एकीकृत है, जहां नागरिक को इन सभी दस्तावेजों तक किसी भी समय पहुंच प्राप्त होगी।
उन्हें किसी भी समय दुनिया में कहीं से भी उपयोग करने की स्वतंत्रता होगी। क्षेत्र के नागरिक अब जन्म, विवाह और मृत्यु प्रमाण पत्र जैसे प्रमाण पत्रों के लिए नगर परिषद में जाए बिना ऑनलाइन आवेदन कर सकते हैं और ये दस्तावेज नागरिकों को हस्ताक्षरित डिजिलॉकर में उपलब्ध होंगे।
यह उन्हें हमेशा के लिए उनके दस्तावेजों की आसान सुरक्षा द्वारा प्रदान की गई मन की शांति भी सुनिश्चित करेगा।

149. Which of the following is/are true? निम्नलिखित में से कौन-सा सत्य है / हैं?
(1) From 1st January 2017, all mobile phones will be required to have the facility of identifying the location through satellite based GPS. 1 जनवरी 2017 January से, सभी मोबाइल फोन में उपग्रह आधारित जीपीएस के माध्यम से स्थान की पहचान करने की सुविधा होनी आवश्यक है।
(2) From 1st January 2018 all feature phones will have the facility of panic button.

Code :-
A. Only 1
B. Only 2
C. Both 1 and 2
D. Neither 1 nor 2

(D) Neither 1 nor 2
Explanation :-
The Department of Telecommunications has notified the rules on panic button under section 10 of the Indian Wireless Telegraph Act 1933.
Under these rules, w.e.f. 1.1.2017, all feature phones will have the facility of panic button configured to the numeric key 5 or 9 and all smart phones will have the panic button configured to three times short pressing of the on-off button.
Further, w.e.f. 1.1.2018, all mobile phones will be required to have the facility of identifying the location through satellite based GPS.
The Ministry of Women and Child Development had taken up the issue of installation of a physical panic button on mobile phones. It was observed that in order to provide safety to women in distress situation, it is important to enable them to send out distress signal to a family member or the police authorities so that they can be rescued.
The Ministry had discussed this issue with a number of stakeholders as well as Department of Telecommunications and had insisted that a physical panic button is much superior to having an App on the mobile phone.
It was argued that a woman in distress does not have more than a second or two to send out a distress message as a perpetrator will often reach out to her mobile phone in the event of a physical/sexual assault.
दूरसंचार विभाग ने भारतीय वायरलेस टेलीग्राफ अधिनियम 1933 की धारा 10 के तहत पैनिक बटन पर नियमों को अधिसूचित किया है।
इन नियमों के तहत, w.e.f. 1.1.2017, सभी फीचर फोन में संख्यात्मक कुंजी 5 या 9 में कॉन्फ़िगर किए गए पैनिक बटन की सुविधा होगी और सभी स्मार्ट फोन में ऑन-ऑफ बटन के तीन बार कम दबाने पर पैनिक बटन कॉन्फ़िगर किया जाएगा।
इसके अलावा, w.e.f. 1.1.2018, सभी मोबाइल फोन के लिए उपग्रह आधारित जीपीएस के माध्यम से स्थान की पहचान करने की सुविधा होना आवश्यक होगा।
महिला और बाल विकास मंत्रालय ने मोबाइल फोन पर एक शारीरिक आतंक बटन की स्थापना का मुद्दा उठाया था। यह देखा गया कि संकट की स्थिति में महिलाओं को सुरक्षा प्रदान करने के लिए, उन्हें परिवार के सदस्य या पुलिस अधिकारियों को संकट संकेत भेजने में सक्षम बनाना महत्वपूर्ण है ताकि उन्हें बचाया जा सके।
मंत्रालय ने इस मुद्दे पर कई हितधारकों के साथ-साथ दूरसंचार विभाग के साथ चर्चा की थी और इस बात पर जोर दिया था कि मोबाइल फोन पर ऐप रखने के लिए एक भौतिक पैनिक बटन बहुत बेहतर है।
यह तर्क दिया गया कि संकट में रहने वाली महिला के पास एक संकट संदेश भेजने के लिए एक या दो से अधिक नहीं है क्योंकि एक अपराधी अक्सर शारीरिक / यौन हमले की स्थिति में अपने मोबाइल फोन तक पहुंच जाएगा।

150. Which of the following is/are true? निम्नलिखित में से कौन-सा सत्य है / हैं?
(1) There is now provision of relief for offences of rape and gang rape. अब बलात्कार और सामूहिक बलात्कार के अपराधों के लिए राहत का प्रावधान है।
(2) There is delinking requirement of medical examination for getting relief amount for non-invasive kind of offences against women. महिलाओं के खिलाफ गैर-इनवेसिव प्रकार के अपराधों के लिए राहत राशि प्राप्त करने के लिए चिकित्सकीय जांच की आवश्यकता है।

Code :-
A. Only 1
B. Only 2
C. Both 1 and 2
D. Neither 1 nor 2

(C) Both 1 and 2
Explanation :-
The Ministry of Social Justice and Empowerment has notified the Scheduled Castes and the Scheduled Tribes (Prevention of Atrocities) Amendment Rules, 2016 on 14 April 2016, the birthday of Babasaheb Ambedkar.
Rules have now been framed to give effect to the amended provisions. These rules will speed up the process of dispensation of justice to victims of atrocities, are strongly sensitive in cases of offences against women, and liberalize and expedite access to relief for the members of Scheduled Castes and Scheduled Tribes who have been victims of atrocities.
Completion of investigation and filing of charge sheet in the court within sixty days.
Provision of relief for offences of rape and gang rape (This provision has been introduced for the first time.)
Delinking requirement of medical examination for getting relief amount for non-invasive kind of offences against women like sexual harassment, gestures or acts intended to insult the modesty of women, assault or use of criminal force with intent todisrobe, voyeurism, stalking.
Provision of admissible relief amount to SC/ST women for offences of grievous nature, on conclusion of trial, even though not ending in conviction.
Increase in the existing quantum of relief amount from between Rs. 75,000/- to Rs. 7, 50,000/-to between Rs. 85,000/- to Rs. 8, 25,000/-, depending upon the nature of the offence, while linking it with the Consumer Price Index for Industrial Workers for the month of January, 2016.
Provision of admissible relief in cash or in kind or both within seven days to the victims of atrocity, their family members and dependents.
Rationalization of the phasing of payment of relief amount to victims for various offences of atrocities.
Regular Reviews of the scheme for the rights and entitlements of victims and witnesses in accessing justice at the State, District and Sub-Division Level Committees in their respective meetings.
सामाजिक न्याय और अधिकारिता मंत्रालय ने 14 अप्रैल 2016 को बाबासाहेब अम्बेडकर के जन्मदिन पर अनुसूचित जाति और अनुसूचित जनजाति (अत्याचार निवारण) संशोधन नियम, 2016 को अधिसूचित किया है।
संशोधित प्रावधानों को प्रभावी करने के लिए अब नियम बनाए गए हैं। ये नियम अत्याचार के शिकार लोगों को न्याय दिलाने की प्रक्रिया को तेज करेंगे, महिलाओं के खिलाफ अपराधों के मामलों में दृढ़ता से संवेदनशील हैं और अनुसूचित जाति और अनुसूचित जनजाति के सदस्यों के लिए राहत के लिए उदारता और शीघ्रता से उपयोग करते हैं जो अत्याचारों के शिकार हुए हैं।
अदालत में साठ दिनों के भीतर जांच और आरोप पत्र दाखिल करने का काम पूरा।
बलात्कार और सामूहिक बलात्कार के अपराधों के लिए राहत का प्रावधान (यह प्रावधान पहली बार पेश किया गया है।)
महिलाओं के खिलाफ गैर-आक्रामक तरह के अपराधों के लिए राहत राशि प्राप्त करने के लिए चिकित्सीय परीक्षा की आवश्यकता, जैसे यौन उत्पीड़न, इशारों या काम के लिए महिलाओं के शील का अपमान करना, मारपीट करना या आपराधिक बल का इस्तेमाल करने के इरादे से टॉड्रिस, वॉयरायरिज़्म, पीछा करना।
एससी / एसटी महिलाओं को गंभीर प्रकृति के अपराधों के लिए स्वीकार्य राहत राशि का प्रावधान, मुकदमे के निष्कर्ष पर, भले ही दोषसिद्धि में समाप्त न हो।
रुपये के बीच से राहत राशि की मौजूदा मात्रा में वृद्धि। 75,000 / - से रु। 7, 50,000 / - रुपये के बीच। 85,000 / - से रु। 8, 25,000 / -, अपराध की प्रकृति पर निर्भर करता है, जबकि इसे जनवरी, 2016 के महीने के लिए औद्योगिक श्रमिकों के लिए उपभोक्ता मूल्य सूचकांक के साथ जोड़ा जाता है।
अत्याचार के शिकार लोगों, उनके परिवार के सदस्यों और आश्रितों को सात दिनों के भीतर नकद या तरह या दोनों में स्वीकार्य राहत का प्रावधान।
अत्याचार के विभिन्न अपराधों के लिए पीड़ितों को राहत राशि के भुगतान की चरणबद्धता का युक्तिकरण।
राज्य, जिला और उप-मंडल स्तर की समितियों में अपनी-अपनी बैठकों में पीड़ितों और गवाहों के अधिकारों और अधिकारों के लिए योजना की नियमित समीक्षा।

151. Which of the following is/are true? निम्नलिखित में से कौन-सा सत्य है / हैं?
(1) Uchhatar Avishkar Yojana (UAY) has been launched for promoting research that is socially relevant. उच्च्तर अविष्कार योजना (यूएई) सामाजिक रूप से प्रासंगिक अनुसंधान को बढ़ावा देने के लिए शुरू की गई है।
(2) It will be implemented in all universities and colleges of India. यह भारत के सभी विश्वविद्यालयों और कॉलेजों में लागू किया जाएगा।

Code :-
A. Only 1
B. Only 2
C. Both 1 and 2
D. Neither 1 nor 2

(A)  Only 1
Explanation :-
Uchhatar Avishkar Yojana (UAY) has been launched for promoting research that is socially relevant and is of use to the end users like the Industry.
The UAY promotes industry sponsored, outcome-oriented research projects.
The main aim of launching the UAY scheme is to make students more accustomed with the outer world and give them a market oriented mindset.
The Scheme is to be implemented first in premier institutes like IITs only.
Uchhatar Avishkar Yojana (UAY) अनुसंधान को बढ़ावा देने के लिए शुरू की गई है जो सामाजिक रूप से प्रासंगिक है और उद्योग जैसे अंतिम उपयोगकर्ताओं के लिए उपयोग की है।
UAY उद्योग प्रायोजित, परिणाम-उन्मुख अनुसंधान परियोजनाओं को बढ़ावा देता है।
यूएई योजना शुरू करने का मुख्य उद्देश्य बाहरी दुनिया के छात्रों को अधिक आदी बनाना और उन्हें बाजार उन्मुख मानसिकता देना है।
इस योजना को पहले IIT जैसे प्रीमियर संस्थानों में ही लागू किया जाना है।

152. Failing of Money Bill obliges the Chief Ministers to resign, but if they don’t resign then which of the following is/are the course/s of action most appropriate? धन विधेयक का विफल होना मुख्यमंत्रियों को इस्तीफा देने के लिए बाध्य करता है, लेकिन यदि वे इस्तीफा नहीं देते हैं, तो निम्नलिखित में से कौन सा कार्य / क्रिया सबसे उपयुक्त है?
(1) Floor test
(2) No-Confidence Motion
(3) President’s Rule as per article 356 and 365

Code :-
A. 1, 2
B. 1, 3
C. 2, 3
D. All of the above

(A) 1, 2
Explanation :-
Emphasising the importance of a floor test for upholding democracy, a High Court Bench said that failing of Money Bill obliges the Chief Ministers to resign, but if they don’t resign then you either bring a no confidence motion or the Chief Minister is asked to seek confidence through a floor test.
The Bench observed that on many occasions governments don’t resign. They’ve developed a thick skin. If they don’t resign then that’s not a matter to be seen by the President.
Though president’s Rule can be applied. It may be termed unconstitutional if the reasons are not strong enough.
लोकतंत्र को बनाए रखने के लिए एक फ्लोर टेस्ट के महत्व पर जोर देते हुए, एक उच्च न्यायालय की बेंच ने कहा कि मनी बिल का विफल होना मुख्यमंत्रियों को इस्तीफा देने के लिए बाध्य करता है, लेकिन यदि वे इस्तीफा नहीं देते हैं, तो आप या तो अविश्वास प्रस्ताव लाते हैं या मुख्यमंत्री से पूछा जाता है। एक फ्लोर टेस्ट के माध्यम से आत्मविश्वास की तलाश करें।
बेंच ने कहा कि कई मौकों पर सरकारें इस्तीफा नहीं देती हैं। उन्होंने एक मोटी त्वचा विकसित की है। यदि वे इस्तीफा नहीं देते हैं तो राष्ट्रपति द्वारा देखा जाने वाला मामला नहीं है।
हालांकि राष्ट्रपति के नियम को लागू किया जा सकता है इसे असंवैधानिक करार दिया जा सकता है यदि कारण पर्याप्त मजबूत न हों।

153. Which of the following is/are true regarding the Factories Act 1948 and revisions proposed? निम्नलिखित में से कौन-सा फैक्ट्री अधिनियम 1948 के बारे में सच है / और संशोधन प्रस्तावित हैं?
(1) The present Factories Act is applicable to factories (with electricity connection) with 20 workers and factories, without electricity, with 10 workers. वर्तमान कारखानों अधिनियम 10 श्रमिकों के साथ, 20 श्रमिकों और कारखानों के बिना, बिजली के साथ कारखानों पर लागू है।
(2) The proposed law will apply to all factories that employ at least 40 workers. प्रस्तावित कानून उन सभी कारखानों पर लागू होगा जो कम से कम 40 श्रमिकों को रोजगार देते हैं।

Code :-
A. Only 1
B. Only 2
C. Both 1 and 2
D. Neither 1 nor 2

(B) Only 2
Explanation :-
The Act applies to establishments with 10 or more workers, if the premise is using power and to establishments with 20 or more workers, without electricity connection.
The Factories Act is a legislation that deals with safety, health and welfare of workers.
All factories that manufacture or deals with hazardous substance and processes and dangerous operations will be covered under this Act even if they employ a single worker. The sectors that manufacture hazardous processes include coal, gas, iron and steel, petroleum, cement and leather.
In a bid to promote start-ups, the government wants to speed up the process of setting up a factory.
An employer’s proposal to treat different departments of his factory as separate companies will have to be approved or rejected in a month.
Registration of factories will also be made online and entrepreneurs will only have to submit a self-certified declaration on the safety, health and welfare standards of the factory to get approvals for setting up a factory.
The trade unions said this would allow more factories to stay out of the ambit of the Factories Act.
The issue is that government have made factories with less than 40 workers out of purview of this Act. This will encourage factories to divide their department into separate factories.
Apart from speeding up registration and compliance processes to help new entrepreneurs and start-ups, the proposed new Factories Act seeks to do away with the "inspector raj".
अधिनियम 10 या अधिक श्रमिकों के साथ प्रतिष्ठानों पर लागू होता है, यदि आधार बिजली कनेक्शन के बिना बिजली और 20 या अधिक श्रमिकों के साथ प्रतिष्ठानों का उपयोग कर रहा है।
फैक्ट्रीज एक्ट एक ऐसा कानून है जो श्रमिकों की सुरक्षा, स्वास्थ्य और कल्याण से संबंधित है।
वे सभी कारखाने जो खतरनाक पदार्थ और प्रक्रियाओं और खतरनाक कार्यों से निर्माण या लेनदेन करते हैं, वे इस अधिनियम के तहत कवर किए जाएंगे, भले ही वे एक ही कर्मचारी को नियुक्त करते हों। खतरनाक प्रक्रियाओं का निर्माण करने वाले क्षेत्रों में कोयला, गैस, लोहा और इस्पात, पेट्रोलियम, सीमेंट और चमड़ा शामिल हैं।
स्टार्ट-अप को बढ़ावा देने के लिए, सरकार एक कारखाना स्थापित करने की प्रक्रिया को गति देना चाहती है।
एक नियोक्ता को अपने कारखाने के विभिन्न विभागों को अलग-अलग कंपनियों के रूप में मानने के प्रस्ताव को एक महीने में अनुमोदित या अस्वीकार करना होगा।
कारखानों का पंजीकरण भी ऑनलाइन किया जाएगा और उद्यमियों को केवल कारखाना स्थापित करने के लिए अनुमोदन प्राप्त करने के लिए कारखाने की सुरक्षा, स्वास्थ्य और कल्याण मानकों पर स्व-प्रमाणित घोषणा प्रस्तुत करनी होगी।
ट्रेड यूनियनों ने कहा कि इससे कारखानों को फैक्ट्रीज एक्ट के दायरे से बाहर रहना होगा।
मुद्दा यह है कि सरकार ने 40 से कम श्रमिकों वाले कारखानों को इस अधिनियम के दायरे से बाहर कर दिया है। यह कारखानों को अपने कारखानों को अलग-अलग कारखानों में विभाजित करने के लिए प्रोत्साहित करेगा।
नए उद्यमियों और स्टार्ट-अप की मदद के लिए पंजीकरण और अनुपालन प्रक्रियाओं में तेजी लाने के अलावा, प्रस्तावित नए कारखानों अधिनियम "इंस्पेक्टर राज" के साथ दूर करना चाहता है।

154.  Which of the following is/are true? निम्नलिखित में से कौन-सा सत्य है / हैं?
(1) President decision to impose President’s Rule is open to judicial review. राष्ट्रपति शासन लगाने का राष्ट्रपति का निर्णय न्यायिक समीक्षा के लिए खुला है।
(2) Judges are not open to judicial review. न्यायाधीश न्यायिक समीक्षा के लिए खुले नहीं हैं।

Code :-
A. Only 1
B. Only 2
C. Both 1 and 2
D. Neither 1 nor 2

(A) Only 1
Explanation :-
Judges are open to judicial review.
The judiciary can revise its own verdicts.
Judicial review is one of the basic structures of the constitution.
In Bommai case (1994), the following propositions have been laid down by the Supreme Court on imposition of President’s Rule in a state under Article 356:
1. The presidential proclamation imposing President’s Rule is subject to judicial review.
2. The satisfaction of the President must be based on relevant material. The action of the president can be struck down by the court if it is based on irrelevant or extraneous grounds or if it was found to be malafide or perverse.
3. Burden lies on the Centre to prove that relevant material exist to justify the imposition of the President’s Rule.
4. The court cannot go into the correctness of the material or its adequacy but it can see whether it is relevant to the action.
5. If the court holds the presidential proclamation to be unconstitutional and invalid, it has power to restore the dismissed state government and revive the state legislative assembly if it was suspended or dissolved.
6. The state legislative assembly should be dissolved only after the Parliament has approved the presidential proclamation. Until such approval is given, the president can only suspend the assembly. In case the Parliament fails to approve the proclamation, the assembly would get reactivated.
7. Secularism is one of the ‘basic features’ of the Constitution. Hence, a state government pursuing anti-secular politics is liable to action under Article 356.
8. The question of the state government losing the confidence of the legislative assembly should be decided on the floor of the House and until that is done the ministry should not be unseated.
9. Where a new political party assumes power at the Centre, it will not have the authority to dismiss ministries formed by other parties in the states.
10. The power under Article 356 is an exceptional power and should be used only occasionally to meet the requirements of special situations.
न्यायिक समीक्षा के लिए न्यायाधीश खुले हैं।
न्यायपालिका अपने स्वयं के फैसले को संशोधित कर सकती है।
न्यायिक समीक्षा संविधान की बुनियादी संरचनाओं में से एक है।
बोम्मई मामले (1994) में, अनुच्छेद 356 के तहत एक राज्य में राष्ट्रपति शासन लगाने पर सुप्रीम कोर्ट द्वारा निम्नलिखित प्रस्ताव रखे गए हैं:
1. राष्ट्रपति शासन लगाने वाला राष्ट्रपति उद्घोषणा न्यायिक समीक्षा के अधीन है।
2. राष्ट्रपति की संतुष्टि प्रासंगिक सामग्री पर आधारित होनी चाहिए। यदि राष्ट्रपति अप्रासंगिक या असंगत आधारों पर आधारित है या यदि वह कुकृत्य या विकृत पाया गया तो राष्ट्रपति की कार्रवाई को अदालत द्वारा विफल किया जा सकता है।
3. बर्डन केंद्र पर यह साबित करने के लिए निहित है कि राष्ट्रपति के नियम को लागू करने के लिए प्रासंगिक सामग्री मौजूद है।
4. अदालत सामग्री या उसकी पर्याप्तता की शुद्धता में नहीं जा सकती है लेकिन यह देख सकती है कि यह कार्रवाई के लिए प्रासंगिक है या नहीं।
5. यदि न्यायालय राष्ट्रपति की उद्घोषणा को असंवैधानिक और अमान्य ठहराता है, तो उसके पास बर्खास्त राज्य सरकार को बहाल करने और राज्य विधानसभा को पुनर्जीवित करने या उसे भंग करने की शक्ति है।
6. संसद के राष्ट्रपति पद की घोषणा को मंजूरी देने के बाद ही राज्य विधान सभा को भंग कर दिया जाना चाहिए। जब तक ऐसी मंजूरी नहीं दी जाती है, तब तक अध्यक्ष केवल विधानसभा को निलंबित कर सकता है। यदि संसद उद्घोषणा को मंजूरी देने में विफल रहती है, तो विधानसभा पुनः सक्रिय हो जाएगी।
7. धर्मनिरपेक्षता संविधान की 'बुनियादी विशेषताओं' में से एक है। इसलिए, धर्मनिरपेक्ष राजनीति का अनुसरण करने वाली राज्य सरकार अनुच्छेद 356 के तहत कार्रवाई के लिए उत्तरदायी है।
8. राज्य सरकार का विधान सभा के विश्वास को खोने का प्रश्न सदन के पटल पर तय किया जाना चाहिए और जब तक ऐसा नहीं किया जाता तब तक मंत्रालय को एकजुट नहीं होना चाहिए।
9. जहां एक नया राजनीतिक दल केंद्र में सत्ता ग्रहण करता है, उसे राज्यों में अन्य दलों द्वारा गठित मंत्रालयों को खारिज करने का अधिकार नहीं होगा।
10. अनुच्छेद 356 के तहत शक्ति एक असाधारण शक्ति है और विशेष परिस्थितियों की आवश्यकताओं को पूरा करने के लिए कभी-कभी ही इसका उपयोग किया जाना चाहिए।

155. Which of the following is/are true regarding Jal Gram scheme? जल ग्राम योजना के बारे में निम्नलिखित में से कौन-सा सच है / हैं?
(1) Under the Jal Gram scheme, 100 villages dominated by tribal population will be selected. जल ग्राम योजना के तहत, आदिवासी आबादी के प्रभुत्व वाले 100 गांवों का चयन किया जाएगा।
(2) Jal Gram is a scheme under Jal Kranti Abhiyan of Ministry of Water Resources. जल ग्राम जल संसाधन मंत्रालय के जल क्रांति अभियान के तहत एक योजना है।

Code :-
A. Only 1
B. Only 2
C. Both 1 and 2
D. Neither 1 nor 2

(B) Only 2
Explanation :-
100 villages dominated by SCs in the country will be selected under Jal Gram Scheme for water conservation.
Jal Gram is a scheme under Jal Kranti Abhiyan of Ministry of Water Resources, River Development and Ganga Rejuvenation, in which two water starved villages will be selected from every district in the country for all round development of water resources.
देश में एससी के वर्चस्व वाले 100 गांवों को जल संरक्षण के लिए जल ग्राम योजना के तहत चुना जाएगा।
जल ग्राम जल संसाधन, नदी विकास और गंगा कायाकल्प मंत्रालय के जल क्रांति अभियान के तहत एक योजना है, जिसमें जल संसाधनों के सर्वांगीण विकास के लिए देश के हर जिले से दो जल भूखे गांवों का चयन किया जाएगा।

156. Which of the following is/are true? निम्नलिखित में से कौन-सा सत्य है / हैं?
(1) If a witness does not believe in God, he/she can swear in the name of constitution.  यदि कोई गवाह ईश्वर में विश्वास नहीं करता है, तो वह संविधान के नाम पर शपथ ले सकता है।
(2) Currently there are only two ways for a witness to swear – in name of God or in the name of the Indian constitution. वर्तमान में गवाह के लिए केवल दो ही तरीके हैं - भगवान के नाम पर या भारतीय संविधान के नाम पर।

Code :-
A. Only 1
B. Only 2
C. Both 1 and 2
D. Neither 1 nor 2

(D) Neither 1 nor 2
Explanation :-
Currently, as per the Oath Act, 1969, any person who deposes in the court can swear in the name of God or by placing a hand on a religious book.
Petitions have been filed in Bombay High Court seeking for people who practice atheism the liberty to take the oath in courts the name of Constitution instead of swearing in the name of God.
वर्तमान में, शपथ अधिनियम, 1969 के अनुसार, कोई भी व्यक्ति जो न्यायालय में जमा करता है, वह भगवान के नाम पर या किसी धार्मिक पुस्तक पर हाथ रखकर शपथ ले सकता है।
बॉम्बे हाई कोर्ट में याचिका दायर की गई है, जो ऐसे लोगों की तलाश कर रहे हैं जो भगवान की कसम खाने के बजाय संविधान के नाम पर अदालतों में शपथ लेने की स्वतंत्रता का अभ्यास करते हैं।

157. Which of the following states have recently (in 2016) been subjected to the President’s Rule? हाल ही में (2016 में) निम्नलिखित में से कौन-सा राज्य राष्ट्रपति शासन के अधीन है?
(1) Jammu Kashmir
(2) Arunachal Pradesh
(3) Uttarakhand

Code :-
A. 2, 3
B. 1, 3
C. 1, 2
D. All of the above

(D) All of the above
Explanation :-
In Jammu and Kashmir it was implemented twice in since 2015. First from 9, January 2015 to 1, March 2015 - 51 days - Failure of Government formation after fractured verdict in Assembly elections. BJP & PDP reached an understanding to form an alliance to form Government in J&K.
Second from 8 January 2016 to 4 April 2016 - 101 days - Death of Chief minister Mufti Mohammad Sayeed.
Arunachal Pradesh had it from 25 January 2016 to 19 February 2016 - 26 days - 21 Congress MLAs joined hands with 11 of the BJP and two Independents, making the current government a minority government.
Uttarakhand has it from 27 March 2016
Collapse of CM Harish Rawat's ministry following a split in the state unit of Congress party. (The Rule was imposed one day before the scheduled floor test sparking controversy).
जम्मू और कश्मीर में इसे दो बार 2015 से लागू किया गया था। 9 से पहले, जनवरी 2015 से 1, मार्च 2015 तक - 51 दिन - विधानसभा चुनावों में खंडित फैसले के बाद सरकार के गठन में विफलता। बीजेपी और पीडीपी जम्मू-कश्मीर में सरकार बनाने के लिए गठबंधन बनाने के लिए एक समझ में पहुंच गए।
दूसरा 8 जनवरी 2016 से 4 अप्रैल 2016 तक - 101 दिन - मुख्यमंत्री मुफ्ती मोहम्मद सईद का निधन।
अरुणाचल प्रदेश में 25 जनवरी 2016 से 19 फरवरी 2016 तक 26 दिन थे - 21 कांग्रेस विधायकों ने भाजपा के 11 और दो निर्दलीय उम्मीदवारों के साथ हाथ मिलाया, जिससे वर्तमान सरकार अल्पमत की सरकार बन गई।
उत्तराखंड में यह 27 मार्च 2016 से है
कांग्रेस पार्टी की राज्य इकाई में विभाजन के बाद सीएम हरीश रावत के मंत्रालय का पतन। (नियत फ्लोर टेस्ट स्पार्किंग विवाद से एक दिन पहले नियम लागू किया गया था)।

158.  "Locus Standi", a term heard often related judiciary, means :- "Locus Standi", एक शब्द जिसे अक्सर संबंधित न्यायपालिका में सुना जाता है, का अर्थ है: -
A. Interest and relevance of a person in a case किसी मामले में किसी व्यक्ति की रुचि और प्रासंगिकता
B. A person filing a case  मामला दर्ज करने वाला व्यक्ति
C. A person appointed by court for guiding the person filing the case मामला दायर करने वाले व्यक्ति का मार्गदर्शन करने के लिए अदालत द्वारा नियुक्त व्यक्ति
D. Friend of court

(A) Interest and relevance of a person in a case
Explanation :-
In law, standing or locus standi is the term for the ability of a party to demonstrate to the court sufficient connection to and harm from the law or action challenged to support that party's participation in the case.
The court generally asks people their locus standi, which means what is your interest or role In filing this case or raising the issue at hand.
Locus standi means a person who approached the court should show himself that he suffered a legal injury.
Mere interest is generally not sufficient to file the writ. The person should have more interest than that of an ordinary member.
कानून में, स्थायी या लोकल स्टैंडिंग एक ऐसी पार्टी की क्षमता है, जो अदालत में उस मामले में पार्टी की भागीदारी का समर्थन करने के लिए चुनौती दी गई कानून या कार्रवाई से पर्याप्त संबंध रखती है।
अदालत आम तौर पर लोगों से उनके ठिकाने के बारे में पूछती है, जिसका मतलब है कि आपकी रुचि या भूमिका इस मामले को दायर करने या मुद्दे को उठाने में है।
Locus standi का मतलब है कि एक व्यक्ति जिसने अदालत का दरवाजा खटखटाया उसे खुद को दिखाना चाहिए कि उसे कानूनी चोट लगी है।
रिट दाखिल करने के लिए आम तौर पर ब्याज पर्याप्त नहीं है। व्यक्ति को एक साधारण सदस्य की तुलना में अधिक रुचि होनी चाहिए।

159. Recently a governance drive was launched by the Union Government. It was linked to which personality of India? हाल ही में केंद्र सरकार द्वारा एक गवर्नेंस ड्राइव शुरू किया गया था। यह भारत के किस व्यक्तित्व से जुड़ा था?
A. Pandit Deen Dayal Upadhyay
B. Pandit Madan Mohan Malviya
C. Mahatma Gandhi
D. Dr. BR Ambedkar

(D) Dr. BR Ambedkar
Explanation :-
Aimed at improving rural livelihoods and strengthening the Panchayati Raj across the country, the Centre, in collaboration with States and panchayats, will launch an 11-day, nation-wide village self-governance campaign.
The "Gram Uday Se Bharat Uday Abhiyan" will begin on April 14 from Mhow in Madhya Pradesh, on the occasion of 125th birth anniversary of Dr. B.R. Ambedkar.
During the campaign between April 14 and April 16, a "social harmony programme" will be conducted in gram panchayats, through which villagers will honour Dr. Ambedkar.
Between April 17 and April 20, "village farmer assemblies" will be held in gram panchayats, focussing on agriculture and farmers. In these assemblies, information regarding agriculture schemes will be shared with farmers and their suggestions for improvements in agriculture will be solicited.
ग्रामीण आजीविका में सुधार लाने और देश भर में पंचायती राज को मजबूत करने के उद्देश्य से केंद्र, राज्यों और पंचायतों के सहयोग से, 11-दिवसीय, देशव्यापी ग्राम स्व-शासन अभियान शुरू करेगा।
"ग्राम उदय से भारत उदय अभियान" 14 अप्रैल को मध्य प्रदेश के महू से शुरू होगा, डॉ। बी आर की 125 वीं जयंती के अवसर पर। अम्बेडकर।
14 अप्रैल और 16 अप्रैल के बीच अभियान के दौरान, ग्राम पंचायतों में एक "सामाजिक समरसता कार्यक्रम" आयोजित किया जाएगा, जिसके माध्यम से ग्रामीण डॉ। अंबेडकर को सम्मानित करेंगे।
17 अप्रैल से 20 अप्रैल के बीच, "ग्राम किसान सभाओं" को ग्राम पंचायतों में आयोजित किया जाएगा, कृषि और किसानों पर ध्यान केंद्रित किया जाएगा। इन विधानसभाओं में, किसानों के साथ कृषि योजनाओं के बारे में जानकारी साझा की जाएगी और कृषि में सुधार के लिए उनके सुझावों को हल किया जाएगा।

160. Gurgaon is being renamed as :-
A. Gudgaon
B. Gurgram
C. Gurugram
D. GuruDron

(C) Gurugram
Explanation :-
The decision to change the name of Gurgaon was taken on the basis of several representations recommending “Gurugram” as the appropriate name for the city.
Its neighbouring district Mewat has been renamed “Nuh”.
Haryana was a historic land of the Mahabharata period and Gurgaon was a great centre of learning where Guru Dronacharya taught the Pandavas and Kauravas.
The town derived its name from Guru Dronacharya, who was gifted the village as “gurudakshina” by his students, the Pandavas.
It came to be known as “Gurugram”, which in the course of time got distorted to Gurgaon.
For Mewat, it is said that it was a geographical and cultural unit and not a town. It is spread beyond Haryana in adjoining Uttar Pradesh and Rajasthan.
The headquarters of Mewat district is at Nuh town. The people of the area and the elected representatives had been demanding that the name of Mewat be changed to Nuh.
The proposal for the name change will now be sent to the Union government for its approval and come into force after a gazette notification. Following the notification, Gurgaon district will also be known as Gurugram.
गुड़गांव का नाम बदलने का निर्णय "गुरुग्राम" शहर के लिए उपयुक्त नाम की सिफारिश करने वाले कई अभ्यावेदन के आधार पर लिया गया था।
इसके पड़ोसी जिले मेवात का नाम "नूंह" रखा गया है।
हरियाणा महाभारत काल का एक ऐतिहासिक देश था और गुड़गांव शिक्षा का एक बड़ा केंद्र था जहाँ गुरु द्रोणाचार्य ने पांडवों और कौरवों को शिक्षा दी थी।
इस शहर का नाम गुरु द्रोणाचार्य के नाम पर पड़ा, जो अपने छात्रों, पांडवों द्वारा गाँव को "गुरुदक्षिणा" के रूप में भेंट किया गया था।
यह "गुरुग्राम" के रूप में जाना जाता है, जो समय के साथ गुड़गांव में विकृत हो गया।
मेवात के लिए, यह कहा जाता है कि यह एक भौगोलिक और सांस्कृतिक इकाई थी, न कि शहर। यह उत्तर प्रदेश और राजस्थान से सटे हरियाणा से बाहर फैला हुआ है।
मेवात जिले का मुख्यालय नूंह शहर में है। इलाके के लोग और चुने हुए प्रतिनिधि मांग कर रहे थे कि मेवात का नाम बदलकर नूंह कर दिया जाए।
नाम बदलने का प्रस्ताव अब इसकी मंजूरी के लिए केंद्र सरकार को भेजा जाएगा और गजट नोटिफिकेशन के बाद लागू होगा। अधिसूचना के बाद, गुड़गांव जिले को गुरुग्राम के नाम से भी जाना जाएगा।

161.  Which of the following is/are true? निम्नलिखित में से कौन सा सत्य है / हैं?
(1) Due to the Freedom of Religion and management of religious affairs provided in the Fundamental Rights, The State cannot reform Hindu religious institutions of a public character. मौलिक अधिकारों में प्रदत्त धार्मिक स्वतंत्रता और धार्मिक मामलों के प्रबंधन के कारण, राज्य एक सार्वजनिक चरित्र के हिंदू धार्मिक संस्थानों में सुधार नहीं कर सकता है।
(2) Article 25 and 26 are subject to public order, morality and health but not to other provisions relating to fundamental rights owing to their provisions regarding freedom of religion. अनुच्छेद 25 और 26 सार्वजनिक आदेश, नैतिकता और स्वास्थ्य के अधीन हैं लेकिन धर्म की स्वतंत्रता के संबंध में उनके प्रावधानों के कारण मौलिक अधिकारों से संबंधित अन्य प्रावधानों के लिए नहीं।

Code :-
A. Only 1
B. Only 2
C. Both 1 and 2
D. Neither 1 nor 2

(D) Neither 1 nor 2
Explanation :-
Article 25 - Freedom of Conscience and Free Profession, Practice and Propagation of Religion.
Article 25 covers not only religious beliefs (doctrines) but also religious practices (rituals). Moreover, these rights are available to all persons—citizens as well as non-citizens.
However, these rights under article 25 are subject to public order, morality, health and other provisions relating to fundamental rights.
Further, the State is permitted to :-
(A) Regulate or restrict any economic, financial, political or other secular activity associated with religious practice; and
(B) Provide for social welfare and reform or throw open Hindu religious institutions of a public character to all classes and sections of Hindus.
Like the rights under Article 25, the rights under Article 26 are also subject to public order, morality and health but not subject to other provisions relating to the Fundamental Rights.
अनुच्छेद 25 - विवेक की स्वतंत्रता और धर्म के मुक्त पेशे, अभ्यास और प्रसार।
अनुच्छेद 25 में न केवल धार्मिक विश्वास (सिद्धांत), बल्कि धार्मिक प्रथाएं (अनुष्ठान) शामिल हैं। इसके अलावा, ये अधिकार सभी व्यक्तियों-नागरिकों के साथ-साथ गैर-नागरिकों के लिए भी उपलब्ध हैं।
हालांकि, अनुच्छेद 25 के तहत ये अधिकार सार्वजनिक आदेश, नैतिकता, स्वास्थ्य और मौलिक अधिकारों से संबंधित अन्य प्रावधानों के अधीन हैं।
इसके अलावा, राज्य को इसकी अनुमति है: -
(ए) धार्मिक अभ्यास से जुड़े किसी भी आर्थिक, वित्तीय, राजनीतिक या अन्य धर्मनिरपेक्ष गतिविधि को विनियमित या प्रतिबंधित करना; तथा
(बी) सामाजिक कल्याण और सुधार के लिए प्रदान करें या हिंदुओं के सभी वर्गों और वर्गों के लिए एक सार्वजनिक चरित्र के खुले हिंदू धार्मिक संस्थानों को फेंक दें।
अनुच्छेद 25 के तहत अधिकारों की तरह, अनुच्छेद 26 के तहत अधिकार भी सार्वजनिक व्यवस्था, नैतिकता और स्वास्थ्य के अधीन हैं, लेकिन मौलिक अधिकारों से संबंधित अन्य प्रावधानों के अधीन नहीं हैं।

162. Article 15 promotes ending of discrimination. Which of the following articles support article 15 in the issue of women entry into the temple? अनुच्छेद 15 भेदभाव को समाप्त करने को बढ़ावा देता है। निम्नलिखित में से कौन सा लेख मंदिर में महिलाओं के प्रवेश के मुद्दे पर अनुच्छेद 15 का समर्थन करता है?
A. 14, 25, 26
B. 17, 25, 26, 28
C. 14, 25, 26, 27
D. 14, 25

(D) 14, 25
Explanation :-
Equality before law and equal protection of laws (Article 14).
Prohibition of discrimination on grounds of religion, race, caste, sex or place of birth (Article 15).
Abolition of untouchability and prohibition of its practice (Article 17).
Freedom of conscience and free profession, practice and propagation of religion (Article 25).
Freedom to manage religious affairs (Article 26).
Freedom from payment of taxes for promotion of any religion (Article 27).
Freedom from attending religious instruction or worship in certain educational institutions (Article 28).
Thus articles 14, 15 and 25 are closely related to ending of discrimination against women on the issue of temple entry.
कानून के समक्ष समानता और कानूनों की समान सुरक्षा (अनुच्छेद 14)।
धर्म, जाति, जाति, लिंग या जन्म स्थान के आधार पर भेदभाव का निषेध (अनुच्छेद 15)।
अस्पृश्यता का उन्मूलन और इसके अभ्यास का निषेध (अनुच्छेद 17)।
अंतरात्मा की स्वतंत्रता और मुक्त पेशा, अभ्यास और धर्म का प्रचार (अनुच्छेद 25)।
धार्मिक मामलों का प्रबंधन करने की स्वतंत्रता (अनुच्छेद 26)।
किसी भी धर्म के प्रचार के लिए करों के भुगतान से स्वतंत्रता (अनुच्छेद 27)।
कुछ शिक्षण संस्थानों में धार्मिक शिक्षा या पूजा में भाग लेने की स्वतंत्रता (अनुच्छेद 28)।
इस प्रकार 14, 15 और 25 के लेख मंदिर प्रवेश के मुद्दे पर महिलाओं के साथ भेदभाव को समाप्त करने से संबंधित हैं।

163.  Dr. Ambedkar’s birth anniversary will be observed for the first time in :-
A. ASEAN
B. UN
C. IMF
D. UNESCO

(B) UN
Explanation :-
The birth anniversary of BR Ambedkar, father of the Indian Constitution, will be observed on April 13 at the United Nations for the first time with focus on combating inequalities to achieve Sustainable Development Goals.
The landmark 2030 Agenda for Sustainable Development recognises that combating inequality within and among countries, creating sustained, inclusive and sustainable growth and fostering inclusion are interdependent.
The vision of Dr. Ambedkar, the architect of the Indian Constitution, to achieve social justice and equality also finds resonance in the core message of the 2030 Agenda.
भारतीय संविधान के पिता बीआर अंबेडकर की जयंती 13 अप्रैल को संयुक्त राष्ट्र में पहली बार देखी जाएगी, जिसमें सतत विकास लक्ष्यों को प्राप्त करने के लिए असमानताओं का मुकाबला करने पर ध्यान केंद्रित किया जाएगा।
सतत विकास के लिए लैंडमार्क 2030 एजेंडा मानता है कि देशों के भीतर और बीच असमानता का मुकाबला करना, निरंतर, समावेशी और टिकाऊ विकास और समावेश को बढ़ावा देना अन्योन्याश्रित हैं।
सामाजिक न्याय और समानता को प्राप्त करने के लिए भारतीय संविधान के निर्माता डॉ। अम्बेडकर की दृष्टि भी 2030 के एजेंडा के मूल संदेश में प्रतिध्वनि पाती है।

164. Which of the following initiatives are launched by the Health Ministry? स्वास्थ्य मंत्रालय ने निम्नलिखित में से कौन-सी पहल शुरू की है?
(1) ANMOL
(2) India Fights Zika
(3) India Fights Dengue

Code :-
A. 1, 3
B. 2, 3
C. 1, 2
D. All of the above

(A) 1, 3
Explanation :-
‘ANM Online application-ANMOL’ - ANMOL is a tablet-based application that allows ANMs (auxiliary nursing midwives) to enter and update data for beneficiaries of their jurisdiction.
This will ensure more prompt entry and updating of data as well as improve the data quality since the data will be entered “at source” by providers of health services themselves.
The application is Aadhaar-enabled so it will help in authentication of the records of field workers and beneficiaries.
Launching another mobile application ‘India Fights Dengue’, Health ministry said that this application will find wide use. As the fight against Vector Borne Diseases, especially Dengue, can be won only with effective community participation, this App empowers the community members how to contribute towards prevention of Dengue. ‘एएनएम ऑनलाइन एप्लीकेशन-एएनएमओएल’ - एएनएमओएल एक टैबलेट-आधारित एप्लिकेशन है जो एएनएम (सहायक नर्सिंग मिडवाइव्स) को उनके अधिकार क्षेत्र के लाभार्थियों के लिए डेटा दर्ज करने और अपडेट करने की अनुमति देता है।
इससे डेटा की गुणवत्ता में सुधार के साथ ही डेटा की गुणवत्ता में सुधार के साथ-साथ डेटा की गुणवत्ता में सुधार के लिए और अधिक त्वरित प्रवेश और अद्यतन सुनिश्चित किया जा सकेगा।
आवेदन आधार-सक्षम है, इसलिए यह क्षेत्र के श्रमिकों और लाभार्थियों के रिकॉर्ड के प्रमाणीकरण में मदद करेगा।
एक अन्य मोबाइल एप्लिकेशन F इंडिया फाइट्स डेंगू ’लॉन्च करते हुए, स्वास्थ्य मंत्रालय ने कहा कि इस एप्लिकेशन का व्यापक उपयोग होगा। वेक्टर बोर्न डिजीज, विशेष रूप से डेंगू के खिलाफ लड़ाई के रूप में, केवल प्रभावी सामुदायिक भागीदारी के साथ जीता जा सकता है, यह ऐप समुदाय के सदस्यों को यह अधिकार देता है कि डेंगू की रोकथाम के लिए कैसे योगदान दिया जाए।

165. Which of the following initiatives are launched by the Health Ministry? स्वास्थ्य मंत्रालय ने निम्नलिखित में से कौन सी पहल शुरू की है?
(1) Swasth Bharat Mobile application
(2) E-RaktKosh initiative
(3) E-Vaccine

Code :-
A. 1, 3
B. 2, 3
C. 1, 2
D. All of the above

(C) 1, 2
Explanation :-
Health Ministry launched 'E-RaktKosh initiative.' It is an integrated Blood Bank Management Information System that has been conceptualized and developed after multiple consultations with all stakeholders.
This web-based mechanism interconnects all the Blood Banks of the State into a single network.
The Integrated Blood Bank MIS refers the acquisition, validation, storage and circulation of various live data and information electronically regarding blood donation and transfusion service.
Such system is able to assemble heterogeneous data into legible reports to support decision making from effective donor screening to optimal blood dissemination in the field.
This initiative will be of great use to persons and families in need of blood transfusion.
The application will enable not only information of the nearest blood bank on a mobile but also of the availability of the particular blood group in a given radius. स्वास्थ्य मंत्रालय ने 'ई-राकॉश पहल' शुरू की। यह एक एकीकृत ब्लड बैंक प्रबंधन सूचना प्रणाली है जिसे सभी हितधारकों के साथ कई परामर्शों के बाद अवधारणा और विकसित किया गया है।
यह वेब-आधारित तंत्र राज्य के सभी ब्लड बैंकों को एक ही नेटवर्क में इंटरकनेक्ट करता है।
इंटीग्रेटेड ब्लड बैंक MIS रक्त दान और आधान सेवा के संबंध में विभिन्न लाइव डेटा और सूचना के अधिग्रहण, सत्यापन, भंडारण और संचलन को संदर्भित करता है।
इस तरह की प्रणाली क्षेत्र में इष्टतम रक्त प्रसार के लिए प्रभावी दाता स्क्रीनिंग से निर्णय लेने का समर्थन करने के लिए सुस्पष्ट रिपोर्ट में विषम डेटा को इकट्ठा करने में सक्षम है।
यह पहल रक्त आधान की आवश्यकता वाले व्यक्तियों और परिवारों के लिए बहुत काम की होगी।
एप्लिकेशन को न केवल एक मोबाइल पर निकटतम ब्लड बैंक की जानकारी दी जाएगी, बल्कि किसी दिए गए दायरे में विशेष ब्लड ग्रुप की उपलब्धता भी होगी।

166.  In which state has the government launched schemes for distributing energy efficient fans and agriculture pumps? किस राज्य में सरकार ने ऊर्जा कुशल पंखे और कृषि पंप वितरित करने के लिए योजनाएं शुरू की हैं?
a. New Delhi
b. Tamil Nadu
c. Punjab
d. Andhra Pradesh

(D) Andhra Pradesh
Explanation :-
In order to make country more energy efficient, the Union Government today launched two schemes namely National Energy Efficient Agriculture Pumps Programme and National Energy Efficient Fan Programme in Vijayawada, Andhra Pradesh.
These Schemes will be implemented by Energy Efficiency Services Limited (EESL), a JV of PSUs under Ministry of Power.
Under the National Energy Efficient Agriculture Pumps Programme, farmers can replace their inefficient pumps free of cost with the new BEE star rated energy efficient agricultural pump-sets.
These pumps will come enabled with smart control panel and a SIM card, giving farmers the flexibility to remotely control these pumps from their mobile phones and from the comfort of their homes.
With the usage of these 50 Watts BEE 5 Star rated ceiling fans, to be distributed under the National Energy Efficient Fan Programme, it is estimated that consumer’s electricity bills will reduce by about Rs. 700-730 per year.
These fans are30% more energy efficient as compared to conventional fans, which range from 75-80 Watts.
At present, two energy efficient fans will be provided to each consumer at Rs 60 a month per fan on EMI basis. The EMI amount will be added to the consumers’ electricity bills for two years. This scheme will be available to the consumer on providing a copy of latest electricity bill along with a copy of residence proof at the designated distribution center. The consumer can also purchase the fan by paying Rs. 1250/- upfront.
देश को अधिक ऊर्जा कुशल बनाने के लिए, केंद्र सरकार ने आज आंध्र प्रदेश के विजयवाड़ा में राष्ट्रीय ऊर्जा कुशल कृषि पंप कार्यक्रम और राष्ट्रीय ऊर्जा कुशल फैन कार्यक्रम नाम से दो योजनाएं शुरू की हैं।
ये योजनाएं ऊर्जा दक्षता सेवा लिमिटेड (ईईएसएल) द्वारा लागू की जाएंगी, जो बिजली मंत्रालय के अधीन सार्वजनिक उपक्रमों के एक जेवी हैं।
राष्ट्रीय ऊर्जा कुशल कृषि पंप कार्यक्रम के तहत, किसान अपने अकुशल पंपों को नए बीईई स्टार रेटेड ऊर्जा कुशल कृषि पंप-सेटों के साथ मुफ्त में बदल सकते हैं।
ये पंप स्मार्ट कंट्रोल पैनल और एक सिम कार्ड के साथ सक्षम होंगे, जिससे किसानों को अपने मोबाइल फोन से और अपने घरों के आराम से इन पंपों को दूर से नियंत्रित करने की सुविधा मिलेगी।
इन 50 वाट्स बीईई 5 स्टार रेटेड सीलिंग प्रशंसकों के उपयोग के साथ, राष्ट्रीय ऊर्जा कुशल फैन प्रोग्राम के तहत वितरित किए जाने के लिए, यह अनुमान लगाया जाता है कि उपभोक्ता के बिजली के बिल में लगभग रु। 700-730 प्रति वर्ष।
ये पंखे पारंपरिक प्रशंसकों की तुलना में 30% अधिक ऊर्जा कुशल हैं, जो 75-80 वॉट से हैं।
वर्तमान में, प्रत्येक उपभोक्ता को ईएमआई के आधार पर प्रति माह 60 रुपये में दो ऊर्जा कुशल पंखे प्रदान किए जाएंगे। दो साल के लिए उपभोक्ताओं के बिजली बिलों में ईएमआई राशि जोड़ी जाएगी। यह योजना निर्धारित वितरण केंद्र पर निवास प्रमाण की एक प्रति के साथ नवीनतम बिजली बिल की एक प्रति प्रदान करने पर उपभोक्ता को उपलब्ध होगी। उपभोक्ता रुपये देकर पंखा भी खरीद सकता है। 1250 / - अपफ्रंट।